Final Exam All Material, Final Exam New Material, Exam 4 All Questions, NUR 112 Exam 3 ALL QUESTIONS

¡Supera tus tareas y exámenes ahora con Quizwiz!

While performing a physical assessment, the student nurse tells her instructor that she cannot palpate her patient's bladder. Which statement by the instructor is best? A) "Try to palpate it again; it takes practice but you will locate it." B) "Palpate the patient's bladder only when it is distended by urine." C) "Document this abnormal finding on the patient's chart." D) "Immediately notify the nurse assigned to the care of your patient."

2) "Palpate the patient's bladder only when it is distended by urine."

Using. Maslow's hierarchy of needs, rank the following nursing diagnoses in order of importance, beginning with the highest-priority diagnosis (1-4) (Enter the number of each step in the proper sequence, using comma and space format, such as: 1, 2, 3, 4.) 1. Anxiety 2. Risk for infection 3. Disturbed body image 4. Sleep deprivation

4,2,1,3

The school nurse is teaching a group of middle school students how to prevent tinea pedis . Which remark by a student provides evidence of learning ? A ) " I can contract the infection by walking barefoot in the gymnasium's showers . " B ) " The best way to avoid contracting the infection is to use good hand washing . " C ) " Wearing unventilated shoes prevents the fungus from gaining contact with my feet . " D ) " There is really no way to prevent its spread ; it's highly contagious . "

A ) " I can contract the infection by walking barefoot in the gymnasium's showers . "

The nurse reviews contact lens care with a client . Which statement indicates that teaching has been effective ? A ) " I will use saline to clean my lenses . " B ) " I will place my lenses on the counter . " C ) " I will keep my lenses in when using eye drops . " D ) " I will wash my hands after removing my lenses . "

A ) " I will use saline to clean my lenses . "

For which patient problem is it most important to provide frequent perineal care ? A ) Active lower gastrointestinal bleeding B ) Recovering from diabetic ketoacidosis C ) Has a circumcised penis D ) History of acute asthma

A ) Active lower gastrointestinal bleeding

The nurse prepares to change a client's bed . Which actions will the nurse take to minimize the effects of the environment during this task ? Select all that apply . A ) Avoid shaking the linen B ) Place pillows on the sink C ) Place used linen on the floor D ) Hold contaminated linen away from the body E ) Fold and place bedspread on the roommate's chair

A ) Avoid shaking the linen D ) Hold contaminated linen away from the body

Which statement best describes the procedure used to assess capillary refill ? A ) Briefly press the tip of the nail with firm , steady pressure , then release and observe for changes in color . B ) Press firmly with your fingertip for 5 seconds over a bony area , release pressure , and observe the skin for the reaction . C ) Tap on the skin with short strokes from your fingers . D ) Lift a fold of skin and allow it to return to its normal position .

A ) Briefly press the tip of the nail with firm , steady pressure , then release and observe for changes in color .

Which area should the nurse inspect when assessing for cyanosis in a dark - skinned patient ? Select all that apply . A ) Buccal mucosal B ) Around the lips C ) Palms D ) Tongue E ) Nailbeds

A ) Buccal mucosal C ) Palms D ) Tongue

The left pupil of a patient fails to accommodate . This finding may reflect an abnormality in which cranial nerve ? A ) CN III B ) CN V C ) CN VIII D ) CN X

A ) CN III

A patient is brought to the emergency department after inhaling mercury . The nurse should be alert for which acute adverse effects associated with mercury inhalation ? A ) Chest pain , pneumonitis , and inflammation of the mouth B ) Intestinal obstruction and numbness of the hands C ) Hypotension , oliguria , and tingling of the feet D ) Tachycardia , hematuria , and diaphoresis

A ) Chest pain , pneumonitis , and inflammation of the mouth

The nurse assists a surgeon with central venous catheter insertion . Which action is necessary to help maintain sterile technique ? A ) Closing the patient's door to limit room traffic while preparing the sterile field B ) Using clean procedure gloves to handle sterile equipment C ) Placing the nonsterile syringes containing flush solution on the sterile field D ) Remaining 6 inches away from the sterile field during the procedure

A ) Closing the patient's door to limit room traffic while preparing the sterile field

Which disorder ( s ) might limit a patient's visual field ? Select all that apply . A ) Diabetes B ) Advanced glaucoma C ) Peripheral vascular disease D ) Cataracts E) Macular degeneration

A ) Diabetes B ) Advanced glaucoma D ) Cataracts E) Macular degeneration

The nurse reviews the proper way to dispose of solid waste with a group of factory workers . What should be included in this teaching ? A ) Donate used items . B ) Discard yard trimmings . C ) Discard used containers and bags . D ) Purchase products with extra packaging .

A ) Donate used items .

What should the nurse use to assess skin temperature ? A ) Dorsum of the hand B ) Pad of the fingertip C ) Palm of the hand D ) Dorsum of the wrist

A ) Dorsum of the hand

Which action protects the body against infection ? Select all that apply A ) Eating a healthy , well - balanced diet B ) Being an older adult or an infant C ) Engaging in leisure activities three times a week D ) Exercising for 30 minutes 5 days a week E ) Receiving recommended immunizations

A ) Eating a healthy , well - balanced diet C ) Engaging in leisure activities three times a week D ) Exercising for 30 minutes 5 days a week E ) Receiving recommended immunizations

The nurse suspects a 3 - year - old child who is coughing vigorously has aspirated a small object . Which action should the nurse take first ? A ) Encourage the child to continue coughing . B ) Deliver upward abdominal thrusts with a fisted hand . C ) Deliver five rapid back blows between the shoulder blades . D ) Perform a blind finger sweep of the child's mouth .

A ) Encourage the child to continue coughing .

A patient becomes infected with oral candidiasis ( thrush ) while receiving intravenous antibiotics to treat a systemic infection . Which type of infection has the patient developed ? A ) Endogenous healthcare - related B ) Exogenous healthcare - related C ) Latent D ) Primary

A ) Endogenous healthcare - related

The nurse prepares an educational program on safety for staff members . Which reason should the nurse emphasize that is the number one cause for accidental death in individuals over the age of 652 A ) Falls . B ) Fires C ) Poisoning D ) Motor vehicle crashes

A ) Falls .

patient . unconscious position to Which item is best for providing mouth care for an unconscious patient ? A ) Foam swabs B ) Lemon - glycerin swabs C ) Hydrogen peroxide D ) Cotton - tipped applicator soaked in mouthwash

A ) Foam swabs

While assessing a patient , the nurse notes that the patient's nails are excessively brittle . What does this finding suggest ? A ) Inadequate dietary intake B ) Normal aging process C ) Fungal infection D ) Excessive use of silver salts

A ) Inadequate dietary intake

The nurse asks a client with an infection that cannot be identified about recent travel . Why did the nurse assess about travel ? Select all that apply . A ) Infected travelers serve as reservoirs for pathogens . B ) It determines which immunizations may be required at this time . C ) Closed spaces in airplanes provide an environment to transmit pathogens . D ) It aids in determining the microorganism to which the client was exposed . E ) It increases the ability to infect many people headed for widespread locations ,

A ) Infected travelers serve as reservoirs for pathogens . C ) Closed spaces in airplanes provide an environment to transmit pathogens . E ) It increases the ability to infect many people headed for widespread locations ,

What is the body's first line of defense against bacteria ? 1 A ) Intact skin B ) White blood cells C ) Lymph glands D ) Inflammatory response

A ) Intact skin

A patient develops urticaria and pruritus 5 days after beginning phenytoin for treatment of seizures . Which type of reaction is the patient most likely experiencing ? A ) Mild adverse reaction B ) Dose - related adverse reaction C ) Toxic reaction D ) Anaphylactic reaction

A ) Mild adverse reaction

A client asks why the nurse needs so much time to complete a physical assessment . What should the nurse explain as the purposes for this type of assessment ? Select all that apply . A ) Obtain baseline data B ) Identify nursing diagnoses C ) Screen for health problems D ) Evaluate teaching provided E ) Monitor previously identified problems

A ) Obtain baseline data B ) Identify nursing diagnoses C ) Screen for health problems E ) Monitor previously identified problems

Which action should the nurse take when preparing a patient for a bed bath ? A ) Place the nurse call device within reach for safety B ) Cover the patient with the top linens from the bed C ) Have the patient completely bathe himself to promote independence D ) Wash the patient's body without assistance from the patient .

A ) Place the nurse call device within reach for safety

Teratogenic drugs should be avoided in which patient population ? A ) Pregnant women B ) Elderly C ) Children D ) Adolescents

A ) Pregnant women

The Joint Commission's national Speak Up® campaign encourages patients to become active and informed participants on the healthcare team . What is the goal of this campaign ? A ) Prevent healthcare errors B ) Control the cost of healthcare C ) Reduce the number of automobile accidents D ) Provide a forum for people without health insurance

A ) Prevent healthcare errors

Physiological changes associated with aging place the older adult especially at risk for which nursing diagnosis ? A ) Risk for Falls B ) Risk for Ineffective Airway Clearance ( choking ) C ) Risk for Poisoning D ) Risk for Suffocation ( drowning )

A ) Risk for Falls

A patient is admitted with an acute exacerbation of chronic obstructive pulmonary disease . Which finding might the nurse expect when assessing the patient's nails ? A ) Soft , boggy nails B ) Brittle nails C ) Thickened nails D ) Thick nails with yellowing

A ) Soft , boggy nails

The school nurse prepares a teaching session for school - age children . What should be included about firearm safety ? Select all that apply . A ) Stop . B ) Tell an adult . C ) Don't touch it . D ) Leave the area . E ) Check for bullets .

A ) Stop . B ) Tell an adult . C ) Don't touch it . D ) Leave the area .

The nurse is performing an otoscopic examination on an adult patient . After having the patient tilt the head to the side not being examined and looking into the ear canal to make sure a foreign body is not present , what should the nurse do next ? A ) Straighten the ear canal by pulling the pinna up and back . B ) Insert the speculum into the ear canal slowly . C ) Test the mobility of the tympanic membrane . D ) Straighten the ear canal by pulling the pinna down and back .

A ) Straighten the ear canal by pulling the pinna up and back .

A patient asks the nurse why there is no vaccine available for the common cold . Which response by the nurse is correct ? A ) The virus mutates too rapidly to develop a vaccine B ) Vaccines are developed only for very serious illnesses C ) Researchers are focusing efforts on an HIV vaccine D ) The virus for the common cold has not been identified .

A ) The virus mutates too rapidly to develop a vaccine

For which patient should the nurse avoid using back massage ? Select all that apply . A ) Underwent heart surgery 3 days ago B ) Sustained rib fractures from a fall C ) Recovering from a lumbar laminectomy D ) Sustained a leg fracture in a sledding accident E ) Receiving treatment for full - thickness burns

A ) Underwent heart surgery 3 days ago B ) Sustained rib fractures from a fall E ) Receiving treatment for full - thickness burns

The nurse manager is concerned that an epidemic might be occurring . What did the nurse assess to make this clinical determination ? A ) Unusual disease pattern B ) Number of clients prescribed antibiotics C ) Increase in clients prescribed the pneumonia vaccination D ) Limited number of clients with the influenza vaccination

A ) Unusual disease pattern

The patient takes anticoagulants . Which instruction is most important for the nurse to include on the patient's care plan ? A ) Use an electric razor for shaving B ) Apply skin moisturizer . C ) Use less soap when bathing . D ) Floss teeth daily .

A ) Use an electric razor for shaving

The nurse is teaching a group of newly hired nursing assistive personnel ( NAP ) about proper hand washing . Which NAP action indicates that teaching has been effective ? Select all that apply . A ) Uses a paper towel to turn off the faucet B ) Holds fingertips above the wrists while rinsing off the soap C ) Removes all rings and watch before washing hands D ) Cleans underneath each fingernail E ) Rubs hands vigorously for 15 seconds

A ) Uses a paper towel to turn off the faucet C ) Removes all rings and watch before washing hands D ) Cleans underneath each fingernail E ) Rubs hands vigorously for 15 seconds

During a thermometer exchange program at a local hospital , a person drops a mercury thermometer on the floor . How should the nurse intervene ? Select all that apply . A ) Using gloves and a paper towel , place the mercury in a plastic bag , and dispose of it . B ) Notify the hazardous material management team immediately . C ) Evacuate the area immediately . D ) After putting on a gown , gloves , and a mask , clean up the mercury . E ) Wash her hands well after removing the spill . F ) Ventilate the area well for several days .

A ) Using gloves and a paper towel , place the mercury in a plastic bag , and dispose of it . E ) Wash her hands well after removing the spill . F ) Ventilate the area well for several days .

Alcohol - based solutions for hand hygiene can be used to combat which types of organisms ? Select all that apply . A ) Virus B ) Bacterial spores C ) Yeast D ) Mold E ) Protozoa

A ) Virus C ) Yeast D ) Mold E ) Protozoa

Which activity is of highest priority for maintaining medical asepsis ? A ) Washing hands B ) Donning gloves C ) Applying sterile drapes D ) Wearing a gown

A ) Washing hands

The nurse prepares to assess whether a client has a risk for falling . Which question should the nurse ask if using the Morse fall scale ? Select all that apply . A ) What is the client's gait ? B ) Does the client use an ambulatory aid ? C ) Does the client have a history of falling ? D ) Does the client have an intravenous infusion ? E ) Does the client need help walking to the bathroom ?

A ) What is the client's gait ? B ) Does the client use an ambulatory aid ? C ) Does the client have a history of falling ? D ) Does the client have an intravenous infusion ?

A patient reports experiencing gas, abdominal bloating, and diarrhea after consuming milk or cheese. Lactose intolerance might immediately suspected if the patient is of which heritage? A African American B Mexican American C European American D Arab American

A African American

The preceptor reviews collaborative problems with a new graduate. Which statement should the preceptor use to describe this type of client problem? A) A collaborative problem is always a potential problem. B) A collaborative problem cannot be monitored or controlled. C) A collaborative problem is unrelated to a medical diagnosis. D) A collaborative problem can be prevented by an individual nursing intervention.

A collaborative problem is always a potential problem.

The nurse is counseling a 17-year-old girl on smoking cessation. What should the nurse include when teaching this client? Select all that apply. A) "Keep healthy snacks or gum available to chew instead of smoking a cigarette." B) "Don't tell your friends and I family you are trying to quit, until you feel confident that you'll be SUccessful." C.) "Plan a time to quit when you will not have many other demands or stressors in your life." D) "Reward yourself with an activity you enjoy when you quit smoking." E) "Spend time with friends who do not smoke."

A) "Keep healthy snacks or gum available to chew instead of smoking a cigarette." C.) "Plan a time to quit when you will not have many other demands or stressors in your life." D) "Reward yourself with an activity you enjoy when you quit smoking." E) "Spend time with friends who do not smoke."

A mother brings her 4-month-old infant for a well-baby checkup. The mother tells the nurse that she would like to start bottle feeding her baby because she cannot keep up with the demands of breastfeeding since returning to work. Which response by the nurse is appropriate? A) "Make sure you give your baby an iron-fortified formula to supplement any stored breast milk you have." B) "You really need to continue breastfeeding baby." C) "Give your baby formula until he is 6 months old; then you can introduce whole milk." D) "Your baby weighs 14 pounds, so he will require about 36 ounces of formula a day."

A) "Make sure you give your baby an iron-fortified formula to supplement any stored breast milk you have."

The nursing instructor is teaching the student about occurrence reports. Which statement by the student indicates an understanding of the purpose of occurrence reports? A) Occurrence reports track problems and identify areas for quality improvement B) Occurrence reports are required by the Food and Drug Administration to report drug errors C) The Joint Commission requires occurrence reports for all client falls D) Occurrence reports provide legal information should the patient seek legal action after an unusual occurrence.

A) "Occurrence reports track problems and identify areas for quality improvement."

A 17-year-old patient sustained facial fractures and a 6-inch laceration on the left side of her face in a motor vehicle accident. The patient tells the nurse that she does not want anyone to see her "looking this way." Which statement by the nurse is most appropriate? A) "Tell me what you mean by "looking this way." B) "Okay, I'll restrict your visitors until your face heals." C) "Your friends and family love you no matter what." D) "You're young; your face will heal quickly."

A) "Tell me what you mean by "looking this way."

The nurse prepares to assess a client from a non-English-speaking culture. Which question should the nurse include to assess language? A) "Who is in your family?" B) "How long have you lived here?" C) "Who makes most of the decisions?" D) "Are you comfortable speaking English?"

A) "Who is in your family?"

A patient who has been hospitalized for weeks becomes angry and says, "I hate this place; nobody knows how to take care of me or I'd be home by now." Which response by the nurse is best in this situation? A) "You seem angry; what's going on that makes you hate this place?" B) "I'm sorry that we aren't caring for you according to your expectations." C) "You were very sick; don't be angry; you're lucky to be alive. D) "You shouldn't be angry with us; we're trying to help you."

A) "You seem angry; what's going on that makes you hate this place?"

What is the deadline after admission for using the Minimum Data Set to evaluate a newly admitted resident of a long-term care facility? A) 14 days B) 3 days C) 2 days D) 24 hours

A) 14 days

A client's average normal temperature is 98°F. Which temperature would be expected during the night in a healthy young adult client who does not have a fever, inflammatory process, or underlying health problems? A) 97.2°F B) 98.0°F C) 98.6°F D) 99.2°F

A) 97.2°F

What makes a nursing history different from a medical history when collecting assessment data from the patient? A) A nursing history focuses on the patient's responses to the health problem. B) The same information is gathered; the difference is in who obtains the information. C) A nursing history is gathered using a specific format. D) A medical history collects more in-depth information.

A) A nursing history focuses on the patient's responses to the health problem.

Which term refers to the movement of a drug from the site of administration to the bloodstream ? A ) Absorption B ) Distribution C ) Metabolism D ) Excretion

A) Absorption

Which nursing activities represent direct care? Choose all that apply A) Administering a medication B) Documenting an assessment C) Participating in a care conference D) Making work assignments for the shift

A) Administering a medication

Which is considered a "practice "? A) Always drinking water after exercise B) Thinking often about cleanliness C) Emphasizing success D) Maintaining youth

A) Always drinking water after exercise

A nurse admits a patient to the unit after completing a comprehensive interview and physical examination. Which action must the nurse implement to develop a nursing diagnosis? A) Analyze the assessment data B) Consult standards of care C) Decide which interventions are appropriate D) Ask the client's perceptions of her health problem

A) Analyze the assessment data

Which portion of a nutritional assessment must the registered nurse complete? A) Analyzing the data B) Obtaining intake and output C) Weighing the patient D) Obtaining the history

A) Analyzing the data

The nurse is admitting a Roman Catholic adult patient who is critically ill. Based on her knowledge of the patient's religion, for which religious rite should she expect to notify the hospital chaplain? A) Anointing of the Sick B) Baptism C) Eucharist D) Sacrament of Reconciliation

A) Anointing of the Sick

A patient with severe hemorrhoids is incontinent of liquid stool. Which intervention is contraindicated for this patient? A) Apply an indwelling fecal drainage device. B) Apply an external fecal collection device. C) Place an incontinence garment on the patient. D) Place a waterproof pad under the patient's buttocks.

A) Apply an indwelling fecal drainage device.

When assisting with bedside central venous catheter (CVC) placement, which nursing intervention is appropriate? Select all that apply. A) Apply sterile gloves and mask (and possibly gown). B) Scrub the insertion site with antibacterial soap for 1 min. C) Verify that informed consent has been obtained. D) Place the patient in low Fowler's position. E) Prepare an infusion bag with dextrose 10% and water.

A) Apply sterile gloves and mask (and possibly gown). C) Verify that informed consent has been obtained.

The nurse is teaching a client how to use a portable blood pressure device to monitor blood pressure at home. What is the most important action for the nurse to take? A) Ask the client to demonstrate the use of the blood pressure device. B) Explain the importance of frequent calibration of the device. C) Give the client a chart to record his blood pressure readings. D) Provide written instructions of the information taught.

A) Ask the client to demonstrate the use of the blood pressure device

At last measurement, the client's vital signs were: oral temperature 98°F (36.7°C), heart rate 76 beats/min, respiratory rate 16 breaths/min, and blood pressure (BP) 118/60 mm Hg. Four hours later, the vital signs were: oral temperature 103.2°F (38.5°C), heart rate 76 beats/min, respiratory rate 14 breaths/min, and blood pressure 120/66 mm Hg. Which should be the nurse's first intervention at this time? A) Ask the client whether he has had a warm drink in the last 30 minutes. B) Notify the primary care provider of the client's temperature. C) Ask the client whether he is feeling chilled. D) Take the temperature by a different route.

A) Ask the client whether he has had a warm drink in the last 30 minutes.

The nurse has just been assigned to the clinical care of a newly admitted patient. To know how to best care for the patient, the nurse uses the nursing process. Which step would the nurse probably do first? A) Assessment B) Diagnosis C) Plan outcomes D) Plan interventions

A) Assessment

The nurse has just been assigned to the clinical care of a newly admitted patient. To know how to best care for the patient, the nurse uses the nursing process. Which step would the nurse probably do first? A) Assessment B.) Diagnosis C.) Plan outcomes D) Plan interventions

A) Assessment

A patient who was prescribed furosemide (Lasix) is deficient in potassium. Therefore, the patient should be instructed to increase his intake of which foods? A) Bananas, peaches, molasses, and potatoes B) Eggs, baking soda, and baking powder C) Wheat bran, chocolate, eggs, and sardines D) Egg yolks, nuts, and sardines

A) Bananas, peaches, molasses, and potatoes

The nurse is instructing a patient about performing home testing for fecal occult blood. What food should the patient state to avoid eating for 3 days before the test? A) Beef B) Milk C) Eggs D) Oatmeal

A) Beef

Which is an example of what traditional medicine and complementary and alternative medicine therapies have in common? A) Both can produce adverse effects in some patients. B) Both use prescription medications. C) Both are usually reimbursed by insurance programs. D) Both are regulated by the FDA.

A) Both can produce adverse effects in some patients.

The left pupil of a patient fails to accommodate. This finding may reflect an abnormality in which cranial nerve? A) CN III B) CN V C) CN VIII D) CN X

A) CN III

Which phrase differentiates a short-term goal from a long-term goal? A) Can be met within a few hours or a few days B) Are developed from the problem side of the nursing diagnosis C) Must have target times/dates D) Specify desired client responses to interventions

A) Can be met within a few hours or a few days

For a patient with Risk for Imbalanced Nutrition: Less Than Body Requirements related to Impaired Swallowing, which nursing intervention is appropriate? Select all that apply. A) Check inside the mouth for pocketing of food after eating. B) Provide a full liquid diet that is easy to swallow. C) Remind the patient to raise the chin slightly to prepare for swallowing. D) Keep the head of the bed elevated for 30 to 45 minutes after feeding. E) Provide with a straw for drinking fluids.

A) Check inside the mouth for pocketing of food after eating. D) Keep the head of the bed elevated for 30 to 45 minutes after feeding.

The nurse is assessing vital signs for a client after a surgical procedure on the left leg. IV fluids are infusing. Which action is the most important for the nurse at this time? A) Compare the left pedal pulse with the right pedal pulse. B) Count the client's respiratory rate for 1 full minute. C) Take the blood pressure in the arm without an IV. D) Take an oral temperature with an electronic thermometer.

A) Compare the left pedal pulse with the right pedal pulse

A mother of a school-age child seeks healthcare because her child has had diarrhea after being ill with a viral infection. The patient states that after vomiting for 24 hours, his appetite has returned. Which recommendation should the nurse make to this mother? A) Consume a diet consisting of bananas, white rice, applesauce, and toast. B) Drink large quantities of water regularly to prevent dehydration. C) Take loperamide (an antidiarheal) as needed to control diarrhea. D) Increase the consumption of raw fruits and vegetables.

A) Consume a diet consisting of bananas, white rice, applesauce, and toast.

While driving home from work, the nurse stops to help victims of a motor vehicle crash before an ambulance arrives. Which action ensures the nurse is functioning within the scope of the Good Samaritan laws? A) Contacted 911 immediately upon arriving to the scene B) Received victim's credit card information for payment C) Stayed with the victim after emergency personnel arrived D) Walked a block away from the site to get paper towels and water

A) Contacted 911 immediately upon arriving to the scene

After observing a conversation, the nurse manager determines that a new nurse's communication technique was effective. Which aspects of communication did the manager observe? Select all that apply. A) Correct channel B) Effective message C) Minimized concerns D) Encouraged feedback E) Appropriate encoding

A) Correct channel B) Effective message D) Encouraged feedback E) Appropriate encoding

Before entering the room of a patient who is angry and yelling, the nurse removes her stethoscope from around her neck. Why did the nurse perform this action? A) Could be used by the patient to hurt her B) Might cause the patient not to trust her C) Would distract her from focusing on the patient D) Will function as another stressor for the patient

A) Could be used by the patient to hurt her

Which procedure technique has the most effect on the accuracy of an apical pulse count? A) Counting the rate for 1 full minute B) Exposing only the left side of the chest C) Determining why assessment of apical pulse is indicated D) Using your ring finger to palpate the intercostal spaces

A) Counting the rate for 1 full minute

A patient who underwent a total abdominal hysterectomy is assisted out of bed as soon as her vital signs are stable. Which item is directing this intervention? A) Critical pathway B) Nursing care plan C) Case manager D) Traditional care model

A) Critical pathway

The nurse suspects that a client is overusing a defense mechanism. What behavior did the client demonstrate to the nurse? A) Criticized spouse for health problems B) Requested strategies to prevent overeating C) Asked for information about a prescribed medication D) Discussed ways to react when an adolescent daughter acts out

A) Criticized spouse for health problems

Which statement about culture is true? Select all that apply. A) Culture exists on both material and nonmaterial levels. B) Culture mainly influences food choices and special holidays. C) Cultural customs change over time at different rates. D) Culture is learned through life experiences shared by other cultural members. E) Cultural assumptions and habits unconscious.

A) Culture exists on both material and nonmaterial levels. C) Cultural customs change over time at different rates. D) Culture is learned through life experiences shared by other cultural members. E) Cultural assumptions and habits unconscious.

An adult patient is diagnosed with lung cancer, and surgery to remove the right lung is recommended. The patient is uncertain about whether he should consent to the surgery because of the risks involved. Which nursing diagnosis is most appropriate for this patient? A) Decisional Conflict B) Death Anxiety C) Powerlessness D) Ineffective Denial

A) Decisional Conflict

At the end of an imagery session, which physical assessment finding suggests that the relaxation technique was successful? A) Decreased blood pressure B) Decreased peripheral skin temperature C) Increased heart rate D) Increased respiratory rate

A) Decreased blood pressure

The nurse assesses a client's vital signs. Which client situation should be reported to the primary care provider? A) Decreased blood pressure (BP) after standing up B) Decreased temperature after a period of diaphoresis C) Increased heart rate after walking down the hall D) Increased respiratory rate when the heart rate increases

A) Decreased blood pressure (BP) after standing up

What is the most significant change in kidney function that occurs with aging? A) Decreased glomerular filtration rate B) Proliferation of micro-blood vessels to renal cortex C) Formation of urate crystals D) Increased renal mass

A) Decreased glomerular filtration rate

The nurse is caring for a patient with unresolved anger. For which associated complication should the nurse assess? A) Depression B) Hypochondriasis C) Somatization D) Malingering

A) Depression

The nurse prepares an educationai program on irritable bowel syndrome for a group of clients. What should the nurse emphasize as the role of the small intestine? Select all that apply. A) Digests lipids B) Secretes mucus C) Absorbs vitamins D) Processes chyme E) Absorbs carbohydrates

A) Digests lipids C) Absorbs vitamins D) Processes chyme E) Absorbs carbohydrates

The nurse prepares an educationai program on irritable bowel syndrome for a group of clients. What should the nurse emphasize as the role of the small intestine? Select all that apply. A) Digests lipids B) Secretes mucus C) Absorbs vitamins D) Processes chyme E) Absorbs carbohydrates

A) Digests lipids C) Absorbs vitamins D) Processes chyme E) Absorbs carbohydrates

The nurse gives a patient the wrong medication and documents that the correct medication was given. The next staff member recognizes the error and reports the nurse to the state board of nursing. Which action can the state board of nursing take against the nurse in this situation? A) Disciplinary action against the nurse's license to practice B) Criminal misdemeanor charges against the nurse C) Medical malpractice lawsuit against the nurse D) Employment release from the institution

A) Disciplinary action against the nurse's license to practice

Which statement best describes theology? A) Discussions and theories related to God and His relation to the world B) Doctrines about the human soul and its relation to eternal life C) A lifelong journey involving accumulation of experience and understanding D) Codes of conduct that integrate beliefs and values

A) Discussions and theories related to God and His relation to the world

The nurse caring for a patient admitted with severe depression identifies a nursing diagnosis of Hopelessness on the care plan. Which outcome is appropriate for this diagnosis? A) Displays stabilization and control of mood B) Sleeps 6 to 8 hours per night with report of feeling rested C) Does not engage in risky, self-injurious behavior D) Eats a well-balanced diet to prevent weight change

A) Displays stabilization and control of mood

Which instruction should the nurse give to the patient experiencing constipation? Select all that apply. A) Drink at least eight glasses of water or noncaffeinated fluid per day. B) Include a minimum of four servings of meat per day. C) Consume a high-fiber diet. D) Exercise as you feel necessary. E) Eat meals on a regular schedule.

A) Drink at least eight glasses of water or noncaffeinated fluid per day. minimum C) Consume a high-fiber diet. E) Eat meals on a regular schedule.

During an assessment, the nurse suspects that a client utilizes maladaptive coping methods to handle stress. What did the nurse assess in this client? A) Drinks with friends after work B) Attends a poetry class at the library C) Runs in the park several times a week D) Tends a vegetable garden in the summer

A) Drinks with friends after work

The nurse cares for a patient who suddenly experiences a cardiac arrest. As the nurse responds to the emergency, which substance in the nurse's body will be secreted in large amounts to help the nurse react in this situation? A) Epinephrine B) Corticotrophin-releasing hormone C) Aldosterone D) Antidiuretic hormone

A) Epinephrine

The nurse is writing a nursing diagnosis for a client. Which part of the statement indicates the interventions required to address the problem? A) Etiology B) Problem C) Frequency D) Completion date

A) Etiology

A client's radial pulse is full and bounding. Which nursing diagnosis should the nurse select to address this clinical finding? A) Excess fluid volume B) Deficient fluid volume C) Decreased cardiac output D) Ineffective tissue perfusion

A) Excess fluid volume

A mother tells the nurse at an annual well-child checkup that her 6-year-old son occasionally "wets himself." Which response by the nurse is appropriate? A) Explain that occasional wetting is normal in children of this age. B) Tell the mother to restrict her child's activities to avoid wetting. C) Suggest "time-out" to reinforce the importance of staying dry D) Inform the mother that medication is commonly used to control wetting.

A) Explain that occasional wetting is normal in children of this age.

What should the nurse understand as being advantages of an electronic health record (EHR)?Select all that apply. A) Facilitate evidence-based nursing practice B) Promote efficient use of the nurse's documentation time C) Reduce the opportunity for interdisciplinary collaboration D) Ensure improved client safety and outcomes E) Reduce cost for documenting care electronically

A) Facilitate evidence-based nursing practice B) Promote efficient use of the nurse's documentation time D) Ensure improved client safety and outcomes

abnormal potassium level of 6.8; communication, What does this scenario demonstrate? Select all that apply. however, he fails to notify the physician The nurse notes that a patient with renal failure has an and does not relate the value during hand-off A) Failure to inmplement a plan of care. B) Failure to evaluate C) Malpractice D) Failure to assess and diagnose E) Failure to follow a standard of care

A) Failure to inmplement a plan of care. B) Failure to evaluate D) Failure to assess and diagnose

The staff development coordinator prepares the Nurse's Bill of Rights for new nursing orientation. What rights should the coordinator emphasize when reviewing this document? Select all that apply. A) Fair compensation for work B) Negotiate terms of employment C) Freely and openly advocate for clients D) Walk off a care area for unsafe conditions E) Work in an environment that supports ethical practice

A) Fair compensation for work B) Negotiate terms of employment C) Freely and openly advocate for clients E) Work in an environment that supports ethical practice

The patient verbalizes an overwhelming lack of energy stating, " I still feel exhausted even after I sleep. I feel guilty when I can't keep up with my usual daily activities or sleeping during the day. I've been a little depressed lately, too. The patient seems to have difficulty concentrating but has no apparent physical problems. Which diagnosis best describes his health status? A) Fatigue related to depression B) Fatigue related to difficulty concentrating C) Guilt related to lack of energy D) Chronic confusion related to lack of energy

A) Fatigue related to depression

Which findings are specific indicators of hypoxia? Select all that apply. A) Feelings of anxiety B) Crackles in the lung bases C) Increased heart rate D) Improved breathing in upright position E) Cyanosis of the tongue

A) Feelings of anxiety C) Increased heart rate E) Cyanosis of the tongue

The nurse is providing nutrition counseling for a patient planning pregnancy. The nurse should emphasize the importance of consuming which nutrient to prevent neural tube defects? A) Folic acid B) Calcium C) Protein D) Vitamin D

A) Folic acid

In caring for a patient with comorbidities, the nurse draws upon her knowledge of diabetes and skin integrity. In a spirit of inquiry, she looks up the latest guidelines for providing skin care and includes them in the plan of care. The nurse provides skin care according to the procedural guidelines and begins regular monitoring to evaluate the effectiveness of the interventions. Which best describes these activities? A) Full-spectrum nursing B) Critical thinking C) Nursing process D) Nursing knowledge

A) Full-spectrum nursing

The nurse measures the urine output of a patient who requires a bedpan to void. Which action should the nurse take after applying gloves? A) Have the patient void directly into the bedpan. B) Pour the urine into a graduated container. C) Read the volume with the container on a flat surface at eye level. D) Observe the color and clarity of the urine in the bedpan.

A) Have the patient void directly into the bedpan.

The nurse notes an S3 heart sound while performing an assessment on a patient admitted with an acute myocardial infarction . What should this finding indicate to the nurse ? A) Heart failure B ) Coronary artery disease C ) Hypertension D ) Pulmonic stenosis

A) Heart failure

The nurse notes an Si 3 heart sound while performing an assessment on a patient admitted with acute myocardial infarction. What should this finding indicate to the nurse? A) Heart failure B) Coronary artery disease C) Hypertension D) Pulmonic stenosis

A) Heart failure

The nurse requests an older client be prescribed suicide precautions. What assessment finding caused the nurse to make this request? Select all that apply. A) History of depression B) Stated the desire to die C) ) Prior attempt at suicide D) Asked to speak to a pastor E) Explained a plan for suicide

A) History of depression B) Stated the desire to die C) ) Prior attempt at suicide E) Explained a plan for suicide

A client from a non-American culture says that to maintain health there needs to be a balance between the body and nature. Which health belief system does this client follow? A) Holistic B) Scientific C) Biomedical D) Magico-religious

A) Holistic

In a healthy adult, which regulates body fluids? Select all that apply. A) Hormone levels B) Fluid intake C) Oxygen saturation D) Kidney function E) Neurological function

A) Hormone levels B) Fluid intake D) Kidney function

Which questions would be effective for obtaining information from a patient during the nursing assessment? Select all that apply. A) How did this happen to you? B) What was your first symptom? C) Why didn't you seek healthcare earlier? D) When did you start having symptoms ? E) Are you hungry or thirsty?

A) How did this happen to you? B) What was your first symptom? D) When did you start having symptoms ?

Which aspects of healthcare are affected by a client's culture? Select all that apply. A) How the client views healthcare B) How the client views illness C) How the client will pay for healthcare services D) The types of treatments the client will accept E) When the client will seek healthcare services

A) How the client views healthcare B) How the client views illness D) The types of treatments the client will accept E) When the client will seek healthcare services

A patient with a history of alcoholism is disoriented and vacillates between being calm and disruptive and loud. Vital signs are BP 138/84 mm Hg: pulse 135 beats/min, regular and strong; respiratory rate 22 breaths/min; temperature 37.1°C (98.1°F). What electrolyte imbalance might the nurse suspect this patient is experiencing? A) Hypomagnesemia B) Hypocalcemia C) Hyperkalemia D) Hypernatremia

A) Hypomagnesemia

An older patient has been vomiting for 2 days and has been unable to eat or drink anything during that time. Current vital signs are T = 99.6°F (37.6°C) orally; P = 110 beats/min weak and thready; BP = 80/52 mm Hg. Skin and mucous membranes are dry, there is decreased skin turgor, and the patient is experiencing weakness., The most recent laboratory results are as follows: Sodium 138 mEq/L Potassium 3.7 mEq/L Calcium 9.2 mg/dL Magnesium 1.8 mg/dL Chloride 99 mEq/L BUN 29 mg/dL Which health problem should the nurse realize this patient is experiencing? A) Hypovolemia B) Hypervolemia C) Hypernatremia D) Hyponatremia

A) Hypovolemia

Which statement by the new graduate nurse indicates a need for further instruction about documentation? Select all thatNapply. A) I can wait until the end of the shift to document my care B) Charting every 2 hours is the most appropriate way to document nursing care C) I find it easier to chart before I go to lunch and then after my shift my shift report D) I should chart as soon as possible after nursing care is given E) I document medications given before providing them to the patient.

A) I can wait until the end of the shift to document my care B) Charting every 2 hours is the most appropriate way to document nursing care C) I find it easier to chart before I go to lunch and then after my shift my shift report E) I document medications given before providing them to the patient.

The nursing instructor asks students how they would assess the fifth vital sign. Which student would be correct? A) I would have the client rate her pain on a scale of 0 to 10. B) I would ask the client when she had her last bowel movement C) I would take the client's pulse oximetry reading. D) I would interview the client about history of tobacco use.

A) I would have the client rate her pain on a scale of 0 to 10.

A patient comes to the emergency department complaining of severe substernal chest pain. Which statement by the nurse appropriately offers reassurance? A) I'll give you some medication to help relieve the pain. B) If you lie still and relax, you'll be fine in a little while. C) Please try not to think about the pain as best as you can D) Don't worry; we're going to take good care of you

A) I'll give you some medication to help relieve the pain.

The nurse is individualizing the plan of care for a nursing diagnosis of Anxiety. Why should the nurse write goals/outcomes on the plan of care? A) Identifies desired changes in the patient's health status B) Lists specific patient responses to medical interventions C) Highlights specific nursing behaviors to improve a patient's health D) Provides criteria to evaluate the appropriateness of a nursing diagnosis

A) Identifies desired changes in the patient's health status

A patient's arterial blood gas results are pH = 7.30; PCO2 =40, HCO3=19 mEq/L:;PO2 =80. What would be appropriate nursing diagnosis for the patient ? A) Impaired Gas Exchange B) Metabolic Acidosis C) Risk for Impaired Gas Exchange D) Risk for Acid-Base Imbalance

A) Impaired Gas Exchange

The nurse determines goals for a client's plan of care. What should the nurse ensure is included in each of the goals? Select all that apply. A) Indicators B) Outcome label C) Health promotion D) Disease prevention E) Measurement scale

A) Indicators B) Outcome label E) Measurement scale

Which nursing diagnosis is categorized as a psychosocial, rather than a self-concept, diagnosis? A) Ineffective Coping B) Situational Low Self-Esteem C) Disturbed Personal Identity D) Disturbed Body Image

A) Ineffective Coping

Three days after abdominal surgery, the nurse notes 4-cm periwound erythema and swelling at the distal end of a client's incision. The area is tender and warm to the touch. Staples are intact along the incision, and there is no obvious drainage. Heart rate is 96 beats/min and oral temperature is 100.8°F (38.2°C). What kind of complication should the nurse suspect this client is experiencing? A) Infection at the incisional site B) Dehiscence of the wound C) Hematoma under the skin D) Formation of granulation tissue

A) Infection at the incisional site

Which assessment finding indicates that a client has low self-esteem and requires more in-depth assessment? Select all that apply. A) Infrequent eye contact B) Straight posture C) Overly critical of others D) Careful grooming E) Low motivation to learn

A) Infrequent eye contact C) Overly critical of others E) Low motivation to learn

Which body fluid lies in the spaces between the body cells? A) Interstitial B) Intracellular C) Intravascular D) Transcellular

A) Interstitial

The nurse notices that a patient has spoon-shaped, brittle nails. This suggests that the patient is experiencing Imbalanced Nutrition: Less Than Body Requirements related to deficiency of which of the following nutrients? A) Iron B) Vitamin A C) Protein D) Vitamin C

A) Iron

A patient recovering from abdominal surgery has had a urine output greater than 60 mL/hr for the past 2 hours; however, the output suddenly drops to nothing. What should the nurse do first? A) Irrigate the catheter with 30 mL of sterile solution. B) Replace the patient's indwelling urinary catheter. C) Infuse 500 mL of normal saline solution IV over 1 hour. D) Notify the surgeon immediately.

A) Irrigate the catheter with 30 mL of sterile solution.

An older patient is tearful, shaky, and withdrawn and admits to worrying about losing her aging spouse. Why should the nurse recognize this patient's reaction as being Anxiety instead of Fear? Select all that apply. A) It concerns future or anticipated events. B) It concerns anticipation of danger rather than a present danger. C) There is no shakiness or tearfulness present. D) There is a psychological rather than a physical threat. E) The source can be identified.

A) It concerns future or anticipated events. B) It concerns anticipation of danger rather than a present danger. D) There is a psychological rather than a physical threat.

Which is the primary reason for the use of a diagnostic label, or patient problem? A) It is used to develop client goals. B) It is used to develop cue clusters. C) It is used to develop interventions. D) It is the etiology.

A) It is used to develop client goals.

The nurse reviews standards of professional practice every 3 to 4 months. Why does the nurse complete this review? Select all that apply. A) Judge own performance B) Develop an improvement plan C) Incorporate into evaluationNtools D) Educate the public about nursing E) Understand employer expectations

A) Judge own performance B) Develop an improvement plan E) Understand employer expectations

The nurse includes a diagnostic statement in the client's plan Physical Mobility related to laziness and not having appropriate shoes. What is wrong with the diagnostic statement? A) Judgmental B) Too complex C) Legally questionable D) Without supportive data

A) Judgmental

The nurse collects biographical information on a client from Central America. Which information provided by the client represents ethnicity? A) Latino B) Catholic C) White D) Teacher

A) Latino

Which action by the nurse breaches patient confidentiality? Select all that apply. A) Leaving patient data displayed on a computer screen where others are able to view them B) Remaining logged on to the computer system after documenting patient care C) Faxing a patient report to the nurses' station at the facility where the patient is being transferred D) Informing the nurse manager of a change in the patient's condition E) Reviewing the medical record of a next-door neighbor admitted for a health problem

A) Leaving patient data displayed on a computer screen where others are able to view them B) Remaining logged on to the computer system after documenting patient care E) Reviewing the medical record of a next-door neighbor admitted for a health problem

The nurse needs to relocate an intravenous site in a patient with an infiltrated IV in the right forearm. In which site is it appropriate for the nurse to insert the new IV line? Select all that apply. A) Left hand B) Right wrist C) Right antecubital area D) Right saphenous vein E) Left jugular vein

A) Left hand C) Right antecubital area

The nurse is preparing an educational program on nutrition for a group of community members. What should the nurse include as "bad stuff" when discussing dietary guidelines? Select all that apply. A) Limit salt intake. B) Wash fruit before eating. C) Rinse meat before cooking. D) Wash hands before preparing food. E) Ingest alcohol in moderation.

A) Limit salt intake. B) Wash fruit before eating. D) Wash hands before preparing food. E) Ingest alcohol in moderation.

The new nurse is struggling with identifying nursing diagnoses to guide a client's care. What should the preceptor encourage the nurse to do? A) Look at assessment data. B) Ask the client for her goals for care. C) Analyze the results of care provided. D) Identify actions to address specific problems.

A) Look at assessment data.

During an admission assessment the nurse learns that a client has an allergy to penicillin. Where should the nurse document this information after including it in a note in the medical record? A) MAR B) Kardex® C) Bedside clipboard D) Above the client's bed

A) MAR

A practicing registered nurse wants to become a staff development educator. Which type of education should this nurse investigate? A) Master's degree B) Doctoral degree C) Baccalaureate degree D) Continuing education

A) Master's degree

The patient's medical record contains the following documentation: 06/05/17 0200 Received patient from the E.D. BP 80/52, HR 118, RR 24, temp 104°F. Arouses to verbal stimuli but drifts off to sleep. Normal saline infusing in left arm via 18-gauge IV catheter at 250 mL/hr. Urinary catheter draining scant dark amber urine. Pt receiving O2 at 6 L/min via nasal cannula. Lungs with coarse crackles at the left base. Loose cough present. Pt unable to expectorate secretions.— Ann Davids, RN Which type of charting has the nurse used? A) Narrative B) Focus C) SOAP D) PIE

A) Narrative

Which outcome is related to Decreased Cardiac Output? Select all that apply. A) No dyspnea or shortness of breath with exertion B.)Normal skin color C.) Respiratory rate less than 16 breaths/min D.) Brisk capillary refill E.) Urine output > 50 mL/hr

A) No dyspnea or shortness of breath with exertion B.)Normal skin color D.) Brisk capillary refill E.) Urine output > 50 mL/hr

A client is admitted for total hip replacement surgery. What information is essential for the nurse to collect when planning for this client's discharge. Select all that apply. A) Number of steps to get into the home B) Location of the bathroom in the home C) Number of hours the client sleeps per night D) Number of steps to walk inside of the home E) Individuals available to transport the client for care

A) Number of steps to get into the home B) Location of the bathroom in the home D) Number of steps to walk inside of the home E) Individuals available to transport the client for care

Nurses have the potential to be very influential in shaping healthcare policy. Which factor contributes most to nurses' influence? A) Nurses are the largest health professional group. B) Nurses have a long history of serving the public. C) Nurses have achieved some independence from physicians in recent years. D) Political involvement has helped refute negative images portrayed in the media.

A) Nurses are the largest health professional group.

A 52-year-old man has a triceps skinfold thickness of 18 mm, and his weight exceeds the ideal body weight for his height by 23%. Which nursing diagnosis should the nurse identify for this patient? A) Obesity B) Risk for Imbalanced Nutrition: More Than Body Requirements C) Imbalanced Nutrition: Less Than Body Requirements D) Readiness for Enhanced Nutrition

A) Obesity

Which action should the nurse take when beginning bladder training using scheduled voiding? A) Offer the patient a bedpan every 2 hours while she is awake. B) Increase the voiding interval by 30 to 60 minutes each week. C) Frequently ask the patient whether she has the urge to void. D) Increase the frequency between voiding even if urine leakage occurs.

A) Offer the patient a bedpan every 2 hours while she is awake.

The nurse is teaching a patient about the importance of reducing saturated fats in his diet. The nurse will recognize that learning has occurred if, upon questioning, the patient replies that he should read product labels to eliminate the intake of which saturated fat? Select all that apply. 1 A) Palm oil B) Coconut oil C) Canola oil D) Peanut oil E) Olive oil

A) Palm oil B) Coconut oil

The nurse assesses a patient's abdomen 4 days after abdominal surgery and notes that bowel sounds are absent. This finding most likely suggests which postoperative complication? A) Paralytic ileus B) Small bowel obstruction C) Diarrhea D) Constipation

A) Paralytic ileus

A patient with quadriplegia has an ischial wound that extends through the epidermis into the dermis. How should the nurse document the depth of this wound? A) Partial-thickness wound B) Penetrating wound C) Superficial wound D) Full-thickness wound

A) Partial-thickness wound

A 13-year-old patient is admitted to the hospital. There is no medical restriction on visitatio To help maintain the patient's social identity while hospitalized, whom should the nurse encourage to visit? A) Peers B) Grandparents C) Siblings D) Parents

A) Peers

The nurse notes that a client has a loop colostomy. What should the nurse ensure when providing care to this client? A) Plastic rod is in place. B) Irrigations occur every day. C) Ostomy appliance is changed every 6 hours. D) Bedside commode is in place for bowel evacuation.

A) Plastic rod is in place.

A patient of Hispanic origin is frustrated because the healthcare team does not understand the importance of hot and cold therapies. Which nursing diagnosis is most appropriate for this patient? A) Powerlessness B) Impaired Verbal Communication C) Spiritual Distress D) Risk for Noncompliance

A) Powerlessness

Which action should the nurse take to assess a 2-year-old child for pinworms? A) Press clear cellophane tape against the anal opening at night to obtain a specimen. B) Collect a freshly passed stool from a diaper using a wooden specimen blade. C) Place a smear of stool on a slide and add two drops of reagent. D) Prepare the patient for a flat plate (x-ray) of the abdomen.

A) Press clear cellophane tape against the anal opening at night to obtain a specimen.

What is the function of antidiuretic hormone when released in the alarm stage of the general adaptation syndrome? A) Promotes fluid retention by increasing the reabsorption of water by kidney tubules B) Increases efficiency of cellular metabolism and fat conversion to energy for cells and muscle C) Increases the use of fats and proteins for energy and conserves glucose for use by the brain D) Promotes fluid retention by causing the kidneys to reabsorb more sodium

A) Promotes fluid retention by increasing the reabsorption of water by kidney tubules

Which contribution of Florence Nightingale had an immediate impact on improving patients' health? A) Providing a clean environment B) Improving nursing education C) Changing the delivery of care in hospitals D) Establishing nursing as a distinct profession

A) Providing a clean environment

A homeless patient is admitted with an infected leg wound. According to Maslow's hierarchy of needs, which nursing intervention meets a basic physiological need? A) Providing the patient with a dinner tray B) Administering antibiotics as prescribed C) Irigating a wound with normal saline solution D) Encouraging the patient to express his feelings

A) Providing the patient with a dinner tray

A client experiences acute shortness of breath. Which noninvasive technique should the nurse use to assess this client's arterial oxygen saturation ? A) Pulse oximetry B) Auscultate breath sounds C) Count the respiratory rate D) Arterial blood gas sampling

A) Pulse oximetry

Which intervention would be aNppropriate for a client who has a fever? Select all that apply. A) Put an ice pack on the client's neck and axillae. B) Provide the client a blanket when he is shivering. C) Offer the client fluids to drink every 1 to 2 hours. D) Measure the temperature using a tympanic thermometer. E) Lower the head of the bed.

A) Put an ice pack on the client's neck and axillae. C) Offer the client fluids to drink every 1 to 2 hours.

An older patient with dehydration is exhibiting confusion, distractibility, memory loss, and irritability. Which behaviors should lead the nurse to suspect that dementia, rather than depression or dehydration, is the source of the symptoms? A) Rambles, speaks incoherently, and answers questions inappropriately B) Speaks slowly with delayed response to questions but responds appropriately C) Awakens early in the day yet sleeps almost constantly during the day D) Sometimes has difficulty concentrating on details of the present situation

A) Rambles, speaks incoherently, and answers questions inappropriately

Which nursing diagnosis is written in correct format? A) Readiness for Enhanced Nutrition B) Pain related to stating a scale of 1 to 5, it's a 5. C) Impaired Mobility related to pain A.M.B. hip fracture D) Risk for Infection related to compromised immunity A.M.B. fever

A) Readiness for Enhanced Nutrition

The department of nursing at a local hospital is considering changing to charting by exception (CBE). Which statement provides a rationale to support making this change? A) Reduces the time nurses spend charting B) Addresses the patient's concerns holistically C) Establishes an ongoing care plan from admission D) Is most useful when constructing a timeline of events

A) Reduces the time nurses spend charting

The preceptor explains full-spectrum nursing to a new graduate nurse. What should the preceptor include when discussing patient situation? Select all that apply. A) Refers to the context for care B) Includes the client's relationships C) Addresses the resources for client care D) Ensures the same nurse provides care to the client E) Focuses on the patient's environment outside the care setting

A) Refers to the context for care B) Includes the client's relationships C) Addresses the resources for client care E) Focuses on the patient's environment outside the care setting

The nurse receives a telephone order from a primary care provider for 40 mEq potassium chloride in 100 mL of sterile water for injection to be infused over 4 hours. Which action must the nurse take to ensure the accuracy of the order? A) Repeat the order to the prescriber even if she believes she understood the order correctly. B) Immediately notify the pharmacy of the order and verify it with a pharmacist. C) Ask the unit secretary to listen to the prescriber on the phone to verify the order. D) Transcribe the order on note paper and verify the dosage in a drug handbook.

A) Repeat the order to the prescriber even if she believes she understood the order correctly.

Which is an example of a cluster of related cues? A) Reports nausea and stomach pain after eating B) Has a productive cough and states stools are loose C) Has a daily bowel movement and eats a high-fiber diet D) Respiratory rate 20 breaths/min, heart rate 85 beats/min, blood pressure 136/84

A) Reports nausea and stomach pain after eating

Which characteristic do the various definitions of critical thinking have in common? A) Requires reasoned thought B) Asks the questions why? or how? C) Is a hierarchical process D) Demands specialized thinking skills

A) Requires reasoned thought

What is the mechanism that controls acid-base balance? Select all that apply. A) Respiratory mechanisms B) Active transport mechanisms C) Renal mechanisms D) Buffer systems E) Peripheral vascular system

A) Respiratory mechanisms C) Renal mechanisms D) Buffer systems

When a patient has metabolic acidosis, which body system influences the acid-base imbalance to produce the compensatory changes in the arterial blood gases? A) Respiratory system B) Renal system C) Vascular system D) Neurological system

A) Respiratory system

For which adult client should the nurse make follow-up observations and monitor the vital signs closely? A) Resting morning blood pressure is 136/86 while the afternoon BP is 128/84 mm Hg. B) Oral temperature is 97.9°F in the morning and 99.8°F in the evening. C) Heart rate was 76 beats/min before eating and 88 beats/min after eating. D) Respiratory rate is 16 breaths/min when standing and 18 when lying down.

A) Resting morning blood pressure is 136/86 while the afternoon BP is 128/84 mm Hg.

Which is an example of invasion of privacy by nurses? Select all that apply. A) Searching a patient's belongings without permission B) Reviewing the plan for patient care in the lunchroom C) Discussing healthcare issues for an unconscious patient with his power of attorney D) Releasing patient health information to local newspaper reporters E) Taking a photo of the patient and posting it on Facebook

A) Searching a patient's belongings without permission B) Reviewing the plan for patient care in the lunchroom D) Releasing patient health information to local newspaper reporters E) Taking a photo of the patient and posting it on Facebook

Which factor places the patient at risk for constipation? Select all that apply. A) Sedentary lifestyle B) High-dose calcium therapy C) Lactose intolerance D) Consuming spicy foods E) High intake of caffeine

A) Sedentary lifestyle B) High-dose calcium therapy

Which factor places the patient at risk for constipation? Select all that apply. A) Sedentary lifestyle B) High-dose calcium therapy C) Lactose intolerance D) Consuming spicy foods E) High intake of caffeine

A) Sedentary lifestyle B) High-dose calcium therapy

A surgeon prescribes potassium chloride 20 mEq by mouth for a patient with a nasogastric ( NG ) tube for gastric drainage . How should the nurse proceed ? A) Seek clarification from the surgeon about the medication order . B) Clamp the NG tube while administering the dose by mouth . C) Instill the medication through the NG tube . D) Withhold the oral potassium chloride elixir .

A) Seek clarification from the surgeon about the medication order .

A client complains about not having enough time to complete work responsibilities and take care of family activities. What shoukd the nurse suggests to this client? Select all that apply A) Set boundaries B) Learn to say no C) Make a to-do list D) Delegate responsibilities E) Schedule time for exercise

A) Set boundaries B) Learn to say no C) Make a to-do list D) Delegate responsibilities E) Schedule time for exercise

After receiving diphenhydramine, a patient complains of having a dry mouth. Which drug effect is this patient experiencing? A) Side Effect B) Adverse Reaction C) Toxic Reaction D) Supportive Effect

A) Side Effect

The wife of an elderly patient begins crying after she is informed that he has a terminal illness. Which intervention by the nurse is best? A) Sit quietly with the patient's wife while she composes her thoughts. B) Inform his wife that a chaplain is available if she would like to speak to him. C) Remind his wife that her husband has lived a long and happy life. D) Tell his wife there are always options and suggest she not give up hope.

A) Sit quietly with the patient's wife while she composes her thoughts.

The nurse identifies the diagnosis Impaired Urinary Elimination in an older adult patient admitted after a stroke. Impaired Urinary Elimination places the patient at risk for which complication? A) Skin breakdown B) Urinary tract infection C) Bowel incontinence D) Renal calculi

A) Skin breakdown

The nurse manager wants to reduce the number of medication errors made on a care area. What can be implemented to improve the safety of medication administration? Select all that apply. A) Smart pumps B) Automated dispensing cabinets C) Unit dose administration system D) Bar code medication administration E) Computerized prescriber order entry (CPOE)

A) Smart pumps B) Automated dispensing cabinets D) Bar code medication administration E) Computerized prescriber order entry (CPOE)

Which healthcare worker should the nurse consult to counsel a patient about financial and family stressors affecting healthcare? A) Social worker B) Occupational therapist C) Physician's assistant D) Technologist

A) Social worker

A patient with leg cramps is irritable and febrile and has dry mucous membranes. based on these findings, the patient most likely has excess levels on which mineral? A) Sodium B) Potassium C) Phosphorus D) Magnesium

A) Sodium

A patient is admitted with an acute exacerbation of chronic obstructive pulmonary disease. Which finding might the nurse expect when assessing the patient's nails? A) Soft, boggy nails B) Brittle nails C) Thickened nails D) Thick nails with yellowing

A) Soft, boggy nails

The mother of a 15-year-old Mexican female is excited about planning quinceañera. With which cultural term should the nurse identify this event? A) Specific B) Universal C) Stereotype D) Archetype

A) Specific

In preparation for relocating to another state the nurse contacts the state board of nursing to obtain the necessary transfer of nursing license. Where will the nurse learn about continuing education requirements for the new state of practice? A) State board of nursing B) The Joint Commission C) School of original nursing education D) Human resource department at new place of employment

A) State board of nursing

The nurse is performing an otoscopic examination on an adult patient. After having the patient tilt the head to the side not being examined and looking into the ear canal to make sure a foreign body is not present, what should the nurse do next? A) Straighten the ear canal by pulling the pinna up and back. B) Insert the speculum into the ear canal slowly. C) Test the mobility of the tympanic membrane. D) Straighten the ear canal by pulling the pinna down and back.

A) Straighten the ear canal by pulling the pinna up and back.

Which is an example of an ongoing assessment? A) Taking the client's temperature 1 hour after giving acetaminophen (Tylenol) B) Examining the patient's mouth at the time she complains of a sore throat C) Requesting the patient to rate intensity on a pain scale with the first perception of pain D) Asking the patient in detail how he will return to his normal exercise activities

A) Taking the client's temperature 1 hour after giving acetaminophen (Tylenol)

Which statement by the student nurse indicates an understanding of the nursing Kardex®? Select all that apply. A) The Kardex® pulls data from multiple areas of the patient's chart B) The Kardex® is usually kept at the patient's bedside C) The Kardex® is used to document patient response to interventions D) The Kardex® summarizes the plan of care and guides nursing care E) The Kardex® is a portable file of patient information

A) The Kardex® pulls data from multiple areas of the patient's chart D) The Kardex® summarizes the plan of care and guides nursing care E) The Kardex® is a portable file of patient information

A client is admitted to a long-term care facility. What does the nurse have to use to adhere to federal law? A) The Minimum Data Set (MDS) for assessment B) Situation-background-assessment-recommendation (SBAR) for reporting C) Centers for Medicare and Medicaid Services (CMS) guidelines prior to surgery D) The Joint Commission guidelines for discharge planning

A) The Minimum Data Set (MDS) for assessment

The nurse identifies diagnoses appropriate for a client's care. What should the nurse recall when identifying the goals for this client? Select all that apply. A) The goals guide the assessments to complete. B) The goal is the opposite of the unhealthy response. C) Goals and assessments are guided by interventions. D) The problem is the health status that needs to be changed. E) The problem determines the outcomes to measure change.

A) The goals guide the assessments to complete. B) The goal is the opposite of the unhealthy response. D) The problem is the health status that needs to be changed. E) The problem determines the outcomes to measure change.

A nurse with a large caseload of patients needs to delegate some assessment tasks to other members of the health team. The nurse is unsure which tasks can be delegated to nursing assistive personnel (NAP) and which are appropriate for a licensed practical nurse (LPN). To which sources should the nurse turn for the answer this question? Select all that apply. A) The nurse practice act of his state B) The American Medical Association guidelines C) The Code of Ethics for Nurses D) The American Nurses's Scope and Standards of Practice E) The staff nurse who was hired with you several months earlier

A) The nurse practice act of his state D) The American Nurses's Scope and Standards of Practice

What is the primary goal that the nurse should establish for a patient with an open wound? A) The wound will remain free of infection throughout the healing process. B) Client completes antibiotic treatment as ordered. C) The wound will remain free of scar tissue at healing. D) Client increases caloric intake throughout the healing process.

A) The wound will remain free of infection throughout the healing process.

Which is a criticism of standardized nursing diagnoses developed by NANDA-I? A) There is little research to support nursing diagnoses labels. B) A perfect nursing diagnosis must be written for it to be useful. C) They are not included in all states' nurse practice acts. D) Other professions do not recognize nursing diagnoses.

A) There is little research to support nursing diagnoses labels.

What do initial, ongoing, and discharge planning have in common? A) They are based on assessment and diagnosis. B) They require input from a multidisciplinary team. C) They focus on the patient's perception of his needs. D) They have specific timelines in which to be completed.

A) They are based on assessment and diagnosis.

The student nurse asks for an indwelling urinary catheter for a hospitalized patient who is incontinent. Which response should the nurse make about the use of catheters only being absolutely necessary? A) They are the leading cause of infection. B) They are too expensive for routine use. C) They contain latex, increasing the risk for allergies. D) Insertion is painful for most patients.

A) They are the leading cause of infection.

What do the nursing assessment models have in common? A) They assess and cluster data into model categories. B) They organize assessment data according to body systems. C) They specify use of the nursing process to collect data. D) They are based on the ANA Standards of Care.

A) They assess and cluster data into model categories.

Which aspect of the full-spectrum nursing role is essential for the nurse to do in order to successfully carry out all the others? A) Thinking and reasoning about the care B) Providing hands-on client care C) Carrying out physician orders D) Delegating to assistive personnel

A) Thinking and reasoning about the care

The nurse is caring for a group of patients on the medical-surgical unit. Which patient situation exemplifies difficulty in adapting to a change in body image? A) Traumatic amputation of the left leg in an industrial accident B) Hypothyroidism with coarse, dry, thinning hair and weight gain C) Recovering from gastric bypass surgery for obesity D) Recovering from a wound graft for peripheral vascular disease

A) Traumatic amputation of the left leg in an industrial accident

The nurse prepares to conduct anthropometric measurements with a client. Where should the nurse measure skinfold thickness? A) Triceps B) Quadriceps C) Gastrocnemius D) Gluteus maximus

A) Triceps

The nurse needs to provide a medication to a child. What should the nurse recall when calculating the dosage of this medication? Select all that apply. A) Use Clark's rule. B) Convert mg to mEq. C) Refer to a nomogram. D) Recall that 1 mL is equal to 100 units. E) Convert the child's weight to kilograms.

A) Use Clark's rule. C) Refer to a nomogram. E) Convert the child's weight to kilograms.

Which statement about communication is true? Select all that apply. A) Used to meet physical and psychosocial needs B) Most basically described as talking and listening C) The process of sending and receiving information D) The basis for forming relationships E) Method to ensure a message is understood

A) Used to meet physical and psychosocial needs C) The process of sending and receiving information D) The basis for forming relationships

After completing a health history the nurse is concerned that a client is at risk for urinary tract infections. What did the nurse assess to make this clinical determination? Select all that apply. A) Uses a spermicidal contraceptive B) Diagnosed with kidney stones 2 years ago C) Takes medication for type 2 diabetes mellitus D) Treated for a urinary tract infection 6 months ago E) Drinks 8 to 10 glasses of water a day in addition to other fluids

A) Uses a spermicidal contraceptive B) Diagnosed with kidney stones 2 years ago C) Takes medication for type 2 diabetes mellitus D) Treated for a urinary tract infection 6 months ago

A patient experiences expressive aphasia after a stroke. Which expected outcome is appropriate for this patient? A) Uses alternative methods of communication B) Communicates effectively using a translator C) Interprets messages accurately D) Follows commands when asked

A) Uses alternative methods of communication

The nurse prepares a teaching tool about normal urination for a group of community members. What should the nurse include in this teaching? Select all that apply. A) Voiding can be voluntarily delayed. B) The internal sphincter keeps urine from entering the urethra. C) Stretch receptors in the bladder cause the detrusor muscle to relax. D) Stretch receptors in the bladder cause the internal sphincter to constrict. E) Stretch receptors in the bladder are activated when 200 to 450 mL of urine is present.

A) Voiding can be voluntarily delayed. B) The internal sphincter keeps urine from entering the urethra. E) Stretch receptors in the bladder are activated when 200 to 450 mL of urine is present.

Which food provides the body with no usable glucose? A) Wheat germ B) Apple C) White bread D) White rice

A) Wheat germ

The nurse is interviewing a patient who has a recent onset of migraine headaches. The patient is highly anxious and cannot seem to focus on what the nurse is saying. Which question would be best for the nurse to use to begin gathering data about the headaches? A) When did your migraines begins B) Tell me about your family history of migraines. C) What are the types of things that trigger your your headaches. D) Describe what your headaches feel like.

A) When did your migraines begins

To promote wound healing, the nurse is teaching a patient about choosing foods containing protein. The nurse will evaluate that learning has occurred if the patient recognizes which food(s) as an incomplete protein that should be consumed with a complementary protein? Select all that apply. A) Whole grain bread B) Peanut butter C) Chicken D) Eggs E) Meat

A) Whole grain bread B) Peanut butter

Which statement describes the difference between dehiscence and evisceration? A) With dehiscence, there is a separation of one or more layers of wound tissue; evisceration involves the protrusion of internal viscera from the incision site. B) Dehiscence is an urgent complication that requires surgery as soon as possible; evisceration is not as urgent. C) Dehiscence involves the protrusion of internal viscera from the incision site; with evisceration, there is a separation of one or more layers of wound tissue. D) Dehiscence involves rupture of subcutancous tissue; evisceration involves damage to dermal tissue.

A) With dehiscence, there is a separation of one or more layers of wound tissue; evisceration involves the protrusion of internal viscera from the incision site.

A patient is diagnosed with an intestinal infection after traveling abroad. The nurse should encourage the intake of which food to promote healing? A) Yogurt B) Pasta C) Oatmeal D) Broccoli

A) Yogurt

Which statement or question by the nurse manager demonstrates an assertive approach when communicating with the staff nurse about a patient care issue? A) You must assess and document pain status for every patient ? B) Why haven't you been assessing and documenting pain for every patient? C) Will you please assess and document pain status for every patient? D) Explain why you haven't been assessing and documenting pain for every patient?

A) You must assess and document pain status for every patient ?

The nurse administers heparin 5,000 units subcutaneously at 2100 and documents in the medication administration record (MAR) that the dose was administered. What other information is important for the nurse to document? A) Injection site B) Previous site of administration C) Patient response to medication D) Heart rate prior to administration

A) injection site

A health center that is interested in purchasing IV infusion pumps organizes a group of nurses to evaluate pumps provided by a variety of vendors. Which type of group has been organized? A) Short term B) Ongoing C) Self-help D) Work-related social support

A) short term

implementing full-spectrum nursing. What should the nurse manager explain to this nurse? A new graduate nurse is having difficulty understanding the importance of "caring" when Select all that apply. A. ) Caring motivates and facilitates doing. B. ) Caring ensures positive client outcomes. C. ) Caring ensures adherence to the care plan, D. ) Caring motivates and facilitates thinking. E. ) Caring replaces steps in the nursing process.

A. ) Caring motivates and facilitates doing. D. ) Caring motivates and facilitates thinking.

Which special consideration may the nurse need to make when caring for a female Rastafarian patient? A. Allow the patient to wear her own clothing. B. Provide a diet that is caffeine-free. C. Allow the patient to wear jewelry with religious symbols. D.Provide free-flowing water for bathing.

A. Allow the patient to wear her own clothing.

A Muslim client has asked the nurse to pray with her. Which item should the nurse anticipate that the patient could request before praying? A. Bathing water B.Rosary beads C.Mala beads D. Prayer cloth

A. Bathing water

A patient with gout has been treated with allopurinol (Zyloprim) for 2 months. The nurse will monitor laboratory results for which therapeutic effect? a. Decreased uric acid levels b. Decreased prothrombin time c. Decreased white blood cell count d. Increased hemoglobin and hematocrit levels

A. Decreased uric acid levels

The nurse prepares a paper on the traditional American healthcare culture to submit for publication. What should the nurse include as values of this culture? Select all that apply. A. Significance of technology B Desire to conquer disease C. Uses the problem-solving method D. Reliance on the biomedical system E. Defines health as absence of disease

A. Significance of technology B Desire to conquer disease D. Reliance on the biomedical system E. Defines health as absence of disease

During assessment of a patient with osteoarthritis pain, the nurse knows that which condition is a contraindication to the use of nonsteroidal anti-inflammatory drugs (NSAIDs)? a. Renal disease b. Diabetes mellitus c. Headaches d. Rheumatoid arthritis

A. renal disease

A patient calls the clinic to ask about taking a glucosamine-chondroitin supplement for arthritis. The nurse reviews the medication history and notes that there will be a concern for drug interactions if the patient is also taking medications for which disorder? a. Type 2 diabetes mellitus b. Hypothyroidism c. Hypertension d. Angina

A. type 2 diabetes mellitus

After receiving a nebulizer treatment with a beta agonist, the patient complains of nervous and wonders if her asthma is getting worse. What is the nurse's best response? A.) "This is an expected adverse effect. Let me take your pulse." B. ) "The next scheduled nebulizer treatment will be skipped." C.) "I will notify the physician about this adverse effect." D.)"We will hold the treatment for 24 hours."

A.) "This is an expected adverse effect. Let me take your pulse."

Which factor influences normal lung volumes and capacities? Select all that apply A.) Age B.)Race C.)Body size D.)Activity level E.) Gender

A.) Age C.)Body size D.)Activity level E.) Gender

A nurse admits a patient to the unit after completing a comprehensive interview and physical examination. Which action must the nurse implement to develop a nursing diagnosis? A.) Analyze the assessment data B.) Consult standards of care C.) Decide which interventions are appropriate D.) ) Ask the client's perceptions of her health problem

A.) Analyze the assessment data

The nurse notes that a client has a prescription for a peak expiratory flow meter. For which health problem should the nurse prepare teaching for this client? A.)Asthma B.) Pneumonia C.)Emphysema D.)Pulmonary edema

A.) Asthma

What is the nurse's obligation in ethical decision making? Select all that apply. A.) Be a patient advocate. B.) Involve institutional ethics committees. C.) Improve one's own ethical decision making. D.) Respect patient confidentiality. E.) Avoid whistleblowers.

A.) Be a patient advocate. B.) Involve institutional ethics committees. C.) Improve one's own ethical decision making. D.) Respect patient confidentiality.

activities. Which age-related change in the cardiovascular system might the client be An older client reports fatigue when walking upstairs and when performing normal household experiencing? Select all that apply. A.) Cardiac contractile strength is reduced. B.) Heart valves become more rigid. C.) Peripheral vessels lose elasticity. D.) Heart responds to increased oxygen demands. E.)There is reduced tolerance to exercise.

A.) Cardiac contractile strength is reduced. B.) Heart valves become more rigid. C.) Peripheral vessels lose elasticity. E.)There is reduced tolerance to exercise

The nurse manager notes an increase in ethical issues related to societal factors. What has the manager identified as causing these issues? Select all that apply. A.) Cost containment B.) Moral developmental stage C.) Technological advances D.) Multicultural population E.) Increased consumer awareness

A.) Cost containment C.) Technological advances D.) Multicultural population E.) Increased consumer awareness

A client experiences leg pain and edema of both ankles. Which laboratory test should the nurse anticipate being prescribed for this client? A.) Creactive protein (CRP) B.) Coagulation studies C.) Complete blood count D.) Complete metabolism panel

A.) Creactive protein (CRP)

The nurse is discussing the use of adsorbents such as bismuth subsalicylate (Pepto-Bismol) with patient who has diarrhea. The nurse will warn the patient about which possible adverse effects? A.) Dark stools and blue gums B.) Urinary hesitancy C.) Drowsiness and dizziness D.) Blurred vision and headache

A.) Dark stools and blue gums

A client recovering from an acute myocardial infarction is experiencing tachycardia, palpitations, anxiety, jugular vein distention, and fatigue. Which nursing diagnoses is most appropriate for this client? A.) Decreased Cardiac Output B.) Ineffective Peripheral Tissue Perfusion C.) Impaired Cardiac Contractility D.) Activity Intolerance

A.) Decreased Cardiac Output

A client had a myocardial infarction that damaged the sinoatrial (SA) node. For which potential complication should the nurse assess this client? Select all that apply. A.) Decreased heart rate B.) Increased heart rate C.) Decreased cardiac output D.) Decreased strength of ventricular contractions. E.) Increased cardiac output

A.) Decreased heart rate C.) Decreased cardiac output

Three days after cardiac surgery, the client's leg used for harvesting veins for the bypass is warm and tender and has a 3-cm area of erythema and swelling at the distal end of the incision. The incision staples are intact without drainage and vital signs are stable. Which type of complication is this client most likely experiencing? A.) Deep vein thrombosis (DVT) B.) Dehiscence of the wound C.) Internal bleeding D.) Infection at the incisional site

A.) Deep vein thrombosis (DVT)

A client is prescribed an ACE inhibitor. What should the nurse explain as the reason for this medication? A.) Dilate vessels to ease the workload of the heart B.) Increase venous return to raise the blood pressure C.) Decrease the heart rate and reduce heart contractility D.) Increase removal of sodium and water from the body

A.) Dilate vessels to ease the workload of the heart

The staff development instructor plans a training session on full-spectrum nursing. Which concept should the instructor include in this information? Select all that apply. A.) Doing B. ) Caring C. ) Thinking D. ) Collaboration E. ) Patient situation

A.) Doing B. ) Caring C. ) Thinking E. ) Patient situation

Which term describes the ability of nurses to base their practice on professional standards of ethical conduct and to participate in ethical decision making? A.) Ethical agency B.) Attitudes C.) Belief D.) Value neutrality

A.) Ethical agency

A patient is taking guaifenesin (Humibid) as part of treatment for a sinus infection. Which instruction will the nurse include during patient.teaching? A.) Force fluids to help loosen and liquefy secretions. B.) Report clear-colored sputum to the prescriber. C.) Avoid driving a car or operating heavy machinery because of the sedating effects. D.) Report symptoms that last longer than 2 days.

A.) Force fluids to help loosen and liquefy secretions.

In caring for a patient with comorbidities, the nurse draws upon her knowledge of diabetes and skin integrity. In a spirit of inquiry, she looks up the latest guidelines for providing skin care and includes them in the plan of care. The nurse provides skin care according to the procedural guidelines and begins regular monitoring to evaluate the effectiveness of the interventions. Which best describes these activities? A.) Full-spectrum nursing B.) Critical thinking C.) Nursing process D. ) Nursing knowledge

A.) Full-spectrum nursing

A patient with a tracheostomy has difficulty removing excessive, thick mucus from the respiratory tract. The nurse expects that which drug will be ordered to aid in the removal of mucus? A.) Guaifenesin (Humibid) B.) Benzonatate (Tessalon Perles) C.) Diphenhydramine (Benadryl) D.) Dextromethorphan (Robitussin DM)

A.) Guaifenesin (Humibid)

The nurse examines the site of a client's tuberculin skin test as being 5 mm induration and documents that the test is positive. Which information in the client's history did the nurse use to make this clinical determination? A.)HIV positive B.)Type 2 diabetes mellitus C.) Lives in a skilled nursing facility D.)Recent immigrant to the United States

A.) HIV positive

nurse A patient is receiving lactulose (Enulose) three times a day. The nurse knows that the patient is not constipated and is receiving this drug for which reason? A.) High ammonia levels due to liver failure B.) Prevention of constipation C.) Chronic renal failure D.). Chronic diarrhea

A.) High ammonia levels due to liver failure

Which aspects of healthcare are affected by a client's culture? Select all that apply. A.) How the client views healthcare B.) How the client views illness C. ) How the client will pay for healthcare services D. ) The types of treatments the client will accept E.) When the elient will seek healthcare services

A.) How the client views healthcare B.) How the client views illness D. ) The types of treatments the client will accept E.) When the elient will seek healthcare services

A patient will be taking bismuth subsalicylate (Pepto-Bismol) to control diarrhea. When reviewing the patient's other ordered medications, the nurse recognizes that which medication or medication class will interact significantly with the Pepto-Bismol? A.) Hypoglycemic drugs B.) Antibiotics C.) Acetaminophen (Tylenol) D.) Antidepressants

A.) Hypoglycemic drugs

The nurse determines that a client is experiencing mild depression. What did the nurse assess to make this clinical determination? Select all that apply. A.) Low energy B) Excessive thirst C) Feelings of guilt D) Increased sleeping E) Poor concentration

A.) Low energy C) Feelings of guilt D) Increased sleeping E) Poor concentration

A patient who has severe nausea and vomiting following a case of food poisoning comes to the urgent care center. When reviewing his medication history, the nurse notes that he has an allergy to procaine. The nurse would question an order for which antiemetic drug if ordered for this patient? A.) Metoclopramide (Reglan) B.) Promethazine (Phenergan) C.) Phosphorated carbohydrate solution (Emetrol) D.) Palonosetron (Aloxi)

A.) Metoclopramide (Reglan)

What do negligence and malpractice have in common? Select all that apply. A.) Negligence and malpractice are unintentional torts. B.) Negligence and malpractice are felonies. C.) Malpractice is the professional form of negligence. D.) Negligence and malpractice involve the intent to do harm to a patient. E.) Breach of duty must be present for negligence to occur

A.) Negligence and malpractice are unintentional torts. C.) Malpractice is the professional form of negligence.

Which term refers to the ethical questions that arise out of nursing practice? A.) Nursing ethies B.) Bioethics C.) Ethical dilemma D.) Moral distress

A.) Nursing ethies

What is rationale for wrapping petroleum gauze around a chest tube insertion ? A.) Prevents air from leaking around the size. B.) Prevents infection at the insertion site. C.) Absorbs drainage from the insertion site. D.) Protects the tube from becoming dislodged.

A.) Prevents air from leaking around the site.

A patient is receiving a tube feeding through gastrostomy. The nurse expects that which type of drug will be used to promote gastric emptying for this patient? A.) Prokinetic drugs, such as metoclopramide (Reglan) B.) . Serotonin blockers, such as ondansetron (Zofran) C.) Anticholinergic drugs, such as scopolamine (Transderm-Scop) D.) Neuroleptic drugs, such as chlorpromazine (Thorazine)

A.) Prokinetic drugs, such as metoclopramide (Reglan)

The preceptor explains full-spectrum nursing to a new graduate nurse. What should the preceptor include when discussing patient situation? Select all that apply. A.) Refers to the context for care B. ) Includes the client's relationships C.) Addresses the resources for client care D. ) Ensures the same nurse provides care to the client E.) Focuses on the patient's environment outside the care setting

A.) Refers to the context for care B. ) Includes the client's relationships C.) Addresses the resources for client care E.) Focuses on the patient's environment outside the care setting

Which is an example of whistleblowing? Select all that apply. A.) Reporting fraudulent billing practices B.) Reporting patient's health status against the patient's wishes C.) Reporting unsafe work practices D ) Reporting a coworker for working under the influence of drugs E.) Reporting a hospital to The Joint Commission for substandard care

A.) Reporting fraudulent billing practices C.) Reporting unsafe work practices D.) Reporting a coworker for working under the influence of drugs

Which characteristic do the various definitions of critical thinking have in common? A.) Requires reasoned thought B.) Asks the questions "why?" or "how?" C.) ) Is a hierarchical process D.) Demands specialized thinking skills

A.) Requires reasoned thought

The nurse considers using a deontological theory for an ethical issue. What should the nurse have in anticipation of using this theory? A.) Rules B.) Beliefs C.) Values D.) Expenses

A.) Rules

A healthcare provider prescribes a one-time ASAP dose of intravenous morphine sulfate of 25 mg for a client with a terminal illness. Which law should the nurse consult prior to responding to this healthcare provider's prescription? A.) Safe Harbor B.) Good Samaritan C.) Americans With Disabilities Act (ADA) D.) Health Insurance Portability and Accountability Act (HIPAA)

A.) Safe Harbor

The nurse is preparing an educational session that focuses on the difference between professional and personal values. What should the nurse use as an example of a professional Select all that apply. A.) Service B.) Loyalty C.) Altruism D.) Beneficence E) Human dignity

A.) Service B.) Loyalty C.) Altruism E) Human dignity

A patient tells the nurse that she wants to start taking the herbal product goldenseal to improve her health. The nurse will assess for which potential cautions or contraindications to goldenseal? (Select all that apply.) A.) Taking a proton-pump inhibitor B.) Nasal congestion C.) Нурothyroidism D.) Hypertension E.) Sinus infections F.) . Pregnancy

A.) Taking a proton- pump inhibitor D.) Hypertension F.) . Pregnancy

The nurse identifies that a client is at risk for developing cardiovascular system problems. Which information did the nurse use to make this clinical determination? A.) Works as a computer operator B.) Stopped smoking decades ago C.) Drinks coffee without cream or sugar D.) Ingests fresh fruit and vegetables daily

A.) Works as a computer operator

When administering mineral oil, the nurse recognizes that it can interfere with the absorption of which substance? A.). Fat-soluble vitamins B.). Water-soluble vitamins C). Minerals D.) Electrolytes

A.). Fat-soluble vitamins

Which statement is accurate about nasotracheal suctioning? Select all that apply. A.)Apply suction for no longer than 10 to 15 seconds during a single pass. B.)Apply suction while inserting and removing the catheter. C.)Reapply oxygen between suctioning passe for ventilator patients. D)Gently rotate the suction catheter as you remove it. E.)Allow intervals of at least 30 seconds between suctioning.

A.)Apply suction for no longer than 10 to 15 seconds during a single pass. D.)Gently rotate the suction catheter as you remove it. E.)Allow intervals of at least 30 seconds between suctioning.

A patient diagnosed with hypertension is taking an angiotensin-converting enzyme (ACE) inhibitor. When planning care, which outcome would be appropriate for the patient? A.)BP will be lower than 135/85 mm Hg on all occasions. B.) BP will be normal after 2 to 3 weeks on medication. C.) Patient will not experience dizziness on rising. D.) Urine output will increase to at least 50 mL/hr.

A.)BP will be lower than 135/85 mm Hg on all occasions.

A patient with COPD has a pulse oximetry reading of 97%. What other finding would indicate adequate tissue and organ oxygenation? Select all that apply. A.)Normal urine output B.)Strong peripheral pulses C.)Clear breath sounds bilaterally D.)Normal muscle strength E.)Orientation

A.)Normal urine output B.)Strong peripheral pulses D.)Normal muscle strength E.)Orientation

A client has a chest drainage system. What should the nurse include when teaching the client about this system? Select all that apply. A.)Perform frequent coughing and deep-breathing exercises. B.) Sit up in a chair but do not walk while the drainage system is in place. C.)Get out of bed without assistance as much as possible. D)Immediately notify the nurse if experiencing increased shortness of breath. E.)Make sure the collection device is above the level of the chest tube insertion site.

A.)Perform frequent coughing and deep-breathing exercises. D)Immediately notify the nurse if experiencing increased shortness of breath.

Which type of medicine do those of Hindu faith typically practice? A.Ayurvedic medicine B.Western medicine C. Chiropractic medicine D.Qigong

A.Ayurvedic medicine

The nurse is giving an intradermal ( ID ) injection and will choose which syringe for this injection ? a . b . c. d .

B

Which assessment question helps assess immediate memory ? A ) " How did you get to the hospital today ? " B ) " Can you repeat the numbers 2 , 7,9 for me ? " C ) " Do you recall the three items I mentioned earlier ? " D ) " What was your birth date including the year ? "

B ) " Can you repeat the numbers 2 , 7,9 for me ? "

While the nurse assesses a newborn of African American descent , the mother points out a blue - black Mongolian spot on the newborn's back and asks , " What's that ? Is something wrong with my baby ? " Which response by the nurse is best ? A ) " I'll ask the physician to look at the spot . " B ) " Those spots are quite common and typically fade with time . " C ) " You may want a plastic surgeon to look at that . " D ) " That spot is benign so it's nothing you need to worry about . "

B ) " Those spots are quite common and typically fade with time . "

As a general rule , how much liquid soap should the nurse use for effective hand washing ? A ) 2 mL B ) 3 mL C ) 6 mL . D ) 7 ML

B ) 3 mL

The nurse applies resistance to the top of the client's foot and asks him to pull his toes toward his knee . The nurse observes active motion against some , but not against full , resistance . How should the nurse document this finding ? A ) 5 : Normal B ) 4 : Slight weakness C ) 3 : Weakness D ) 2 : Poor ROM

B ) 4 : Slight weakness

The nurse asks the patient to spread the fingers and then bring them together again . What is the nurse testing when asking the patient to bring the fingers together ? A ) Abduction B ) Adduction C ) Flexion D ) Extension

B ) Adduction

A patient is admitted to the hospital with tuberculosis . Which precaution must the nurse institute when caring for this patient ? A) Droplet B ) Airborne C ) Direct contact D ) Indirect contact

B ) Airborne

Which nursing behavior indicates the highest potential for spreading infections among clients ? A ) Disinfects dirty hands with antibacterial soap B ) Allows alcohol - based rub to dry for 10 seconds C ) Washes hands only when leaving each room . D ) Uses cold water for medical asepsis

B ) Allows alcohol - based rub to dry for 10 seconds

The primary care provider orders peak and trough levels for a patient who is receiving intravenous vancomycin every 12 hours . When should the nurse obtain a blood specimen to measure the trough ? A ) With the morning routine laboratory studies B ) Approximately 30 minutes before the next dose C ) Two hours after the next dose infuses D ) While the drug infuses

B ) Approximately 30 minutes before the next dose dose

The nurse observes that an older patient's gait and balance appear to be slightly unsteady . What assessment should the nurse perform next ? A ) Perform the Get Up and Go Test . B ) Ask the patient if he has fallen in the past year . C ) Refer the patient for a comprehensive fall evaluation . D ) Administer the Timed Up and Go Test .

B ) Ask the patient if he has fallen in the past year .

When should the nurse instruct a new mother to expect the anterior fontanel of her infant to fuse ? A ) At about 8 weeks B ) At about 14 months C ) By 6 months of age D ) Before 1 year of age

B ) At about 14 months

Which assessment should the nurse perform if a palpable thyroid gland is present ? A ) Illuminate the thyroid gland for the presence of fluid . B ) Auscultate the thyroid gland for bruits . C ) Percuss the thyroid gland for mass size . D ) Measure the thyroid gland to assess change .

B ) Auscultate the thyroid gland for bruits .

A patient has sustained a spinal cord injury and is no longer able to get in and out of the bathtub without assistance . Which nursing diagnosis appropriately addresses this problem ? A ) Feeding Self - care Deficit B ) Bathing Self - care Deficit C ) Dressing Self - care deficit D ) Activity Intolerance

B ) Bathing Self - care Deficit

Which abnormal laboratory value is associated with icteric sclerae ? A ) Bleeding time B ) Bilirubin C ) Hemoglobin D ) Glucose

B ) Bilirubin

Which statement describes a correct step in removing and cleaning a hearing aid ? A ) Clean only the external surfaces , not the canal portion B ) Clean the top part of the canal portion of the device C ) Insert a wax loop or toothpick into the hearing aid D ) Remove the battery before taking the hearing aid from the ear .

B ) Clean the top part of the canal portion of the device

An 85 - year - old patient is brought to the emergency department with lethargy and hypotension . When the nurse assesses the patient's tongue , she notes that it appears dry and furry . What does this finding indicate to the nurse ? A ) Fungal infection B ) Dehydration C ) Allergy D ) Iron deficiency

B ) Dehydration

A patient with a history of falling continually attempts to get out of bed unassisted despite frequent reminders to call for help first . Which action should the nurse take first ? A ) Apply a cloth vest restraint . B ) Encourage a family member to stay with the patient . C ) Administer lorazepam ( an antianxiety medication ) . D ) Keep the patient's bed siderails up .

B ) Encourage a family member to stay with the patient .

The nurse prepares an educational session on safety for a community group . What should the nurse emphasize as the leading causes of accidental death in preschoolers ? Select all that apply . A) Falls . B ) Fires C ) Poisoning D ) Drowning E ) Motor vehicle injuries

B ) Fires C ) Poisoning D ) Drowning E ) Motor vehicle injuries

The nurse is removing personal protective equipment ( PPE ) . Which item should be removed first ? A ) Gown B ) Gloves C ) Face shield D ) Hair covering

B ) Gloves

What should the nurse consider for a patient who has a temperature of 101 ° F ( 38.3 ° C ) ? A ) Acetaminophen ( Tylenol ) B ) Increased fluids C ) Bedrest D ) Tepid bath

B ) Increased fluids

Which is a benefit of bathing ? Select all that apply . A) Constricts blood vessels B ) Increases depth of respirations C ) Gives opportunity for assessments D ) Reduces sensory input E ) Increases cardiac output

B ) Increases depth of respirations C ) Gives opportunity for assessments

Which portion of the ear is responsible for maintaining equilibrium ? 1 A ) External ear B ) Inner ear C ) Middle ear D ) Ossicles

B ) Inner ear

A patient who just returned from the postanesthesia care unit is complaining of severe incision . pain . Which drug contained in the medication administration record will offer the fastest relief ? A ) Liquid acetaminophen with codeine B ) Intravenous morphine sulfate C ) Intramuscular meperidine D ) Oral oxycodone tablets

B ) Intravenous morphine sulfate

Which point should the nurse include when teaching safety precautions to a mother of a toddler ? Select all that apply . A ) Make sure the child sleeps on his back at night . B ) Keep the telephone number of the poison control center accessible . C ) Use a front - facing car seat placed in the back seat of the car . D ) Keep syrup of ipecac on hand in case of accidental poisoning . E ) Cover electrical outlets .

B ) Keep the telephone number of the poison control center accessible . C ) Use a front - facing car seat placed in the back seat of the car . . E ) Cover electrical outlets .

A patient with a radiation implant is weak and needs help even to turn in bed . Which action . should the nurse take when caring for this patient ? A ) Avoid giving the patient a complete bed bath B ) Limit the amount of time spent with the patient . C ) Allow extra time for the patient to express feelings . D ) Do not allow anyone to visit the patient .

B ) Limit the amount of time spent with the patient .

What type of immunity is provided by intravenous ( IV ) administration of immunoglobulin G ? A) Cell - mediated B ) Passive C ) Humoral D ) Active

B ) Passive

Which is the most commonly reported incident in hospitals ? A ) Equipment malfunction B ) Patient falls C ) Laboratory specimen errors D ) Treatment delays .

B ) Patient falls

Assuming that all are accurate , which documentation about a patient's level of consciousness is best ? A ) Patient is lethargic and slept when undisturbed . B ) Patient responds to tactile stimulation ; falls back to sleep immediately after tactile and verbal stimulation are stopped . C ) Patient slept throughout the day , missing his meals and bath . D ) Patient appears to be tired as he slept throughout the day except when bathed .

B ) Patient responds to tactile stimulation ; falls back to sleep immediately after tactile and verbal stimulation are stopped .

How should the nurse dispose of the breakfast tray of a patient who requires airborne isolation ? A ) Place the tray in a specially marked trash can inside the patient's room . B ) Place the tray in a special isolation bag held by a second healthcare worker at the patient's door . C ) Return the tray with a note to dietary services so it can be cleaned and reused for the next meal . D ) Carry the tray to an isolation trash receptacle located in the dirty utility room and dispose of it there .

B ) Place the tray in a special isolation bag held by a second healthcare worker at the patient's door .

A patient is admitted to the hospital for chemotherapy and has a low white blood cell count . Which precaution should the staff take with this patient ? A ) Contact B ) Protective C ) Droplet D ) Airborne

B ) Protective

The nurse obtains vital signs for a 56 - year - old patient who underwent surgery yesterday . Which finding ( s ) require ( s ) further assessment ? Select all that apply . A ) Blood pressure 110/64 mm Hg . B ) Pulse rate 118 beats / minute C ) Respiratory rate 35 breaths / minute D ) Oral temperature 98.6 ° F ( 37 ° C ) E ) Pulse oximetry reading 94 % on room air

B ) Pulse rate 118 beats / minute C ) Respiratory rate 35 breaths / minute

While donning sterile gloves , the nurse notices the edges of the glove package are slightly yellow . The yellow area is over 1 inch away from the gloves and only appears to be on the outside of the glove package . What is the best action for the nurse to take at this point ? A ) Continue using the gloves inside the package because the package is intact . B ) Remove gloves from the sterile field and use a new pair of sterile gloves . C ) Throw all supplies away that were to be used and begin again . D ) Use the gloves and make sure the yellow edges of the package do not touch the client .

B ) Remove gloves from the sterile field and use a new pair of sterile gloves .

Which test should the patient undergo when the Weber test is positive ? A ) Romberg test B ) Rinne test C ) Pure tone audiometry D ) Tympanometry

B ) Rinne test

A client's epidermis has insufficient melanin . Which nursing diagnosis is appropriate ? A ) Risk for Infection B ) Risk for Impaired Skin Integrity C ) Risk for Deficient Fluid Volume D ) Impaired Skin Integrity

B ) Risk for Impaired Skin Integrity

Based on developmental stage , how should the nurse modify the comprehensive physical examination of an older adult ? A ) Work rapidly to finish as quickly as possible . B ) Sequence the exam to limit position changes . C ) Demonstrate equipment before using it . D ) Omit portions of the exam that may be tiring .

B ) Sequence the exam to limit position changes .

Based on developmental stage , how should the nurse modify the comprehensive physical examination of an older adult ? A ) Work rapidly to finish as quickly as possible . B ) Sequence the exam to limit position changes . C ) Demonstrate equipment before using it . D ) Omit portions of the exam that may be tiring .

B ) Sequence the exam to limit position changes .

The nurse notes ptosis in a patient who just arrived in the emergency department . What should the nurse realize this finding might indicate ? A ) Hyperthyroidism B ) Stroke C ) Glaucoma D ) Macular degeneration

B ) Stroke

A patient with diarrhea is incontinent of liquid stool . The nurse documents that the patient has excoriated skin on the buttocks . Which finding by the nurse led to this documentation ? A ) Skin was softened from prolonged exposure to moisture B ) Superficial layers of skin were absent C ) The epidermal layer of skin was rubbed away D) A lesion caused by tissue compression was present .

B ) Superficial layers of skin were absent

The nurse is caring for a patient who underwent abdominal surgery 24 hours ago and has a nasogastric tube to intermittent suction . How should the nurse proceed when performing an abdominal assessment on this patient ? A ) Avoid palpating the patient's abdomen . B ) Turn off the suction before auscultating bowel sounds . C ) Listen for bowel sounds for 2 minutes in each quadrant . D ) Percuss the abdomen before auscultating bowel sounds .

B ) Turn off the suction before auscultating bowel sounds .

A nurse is teaching a group of mothers about first aid . What should the nurse include to prevent accidental burns in the home ? A ) Wear loose - fitting clothing when cooking . B ) Turn pot handles toward the back of the stove . C ) Use the microwave to warm baby formula . D ) Apply butter to the area should a scald burn occur .

B ) Turn pot handles toward the back of the stove .

A patient with dementia becomes belligerent when the nurse attempts to give him a tub bath . How should the nurse proceed ? A ) Call for assistance to help the patient into the bathtub B ) Wait for the patient to calm down , and then give him a towel bath C ) Allow the patient to go without bathing for a day or two D) Ask another staff member to attempt the tub bath

B ) Wait for the patient to calm down , and then give him a towel bath

The nurse visits the home of a client with mouth infection . What should the nurse instruct the client to do to adequate disinfect eating utensil ? A ) Boil in hot water . B ) Wash in the dishwasher . C ) Soak in a bleach solution . D ) Microwave on the high setting .

B ) Wash in the dishwasher .

Abdominal palpation should be avoided in a child who has which disorder ? A ) Appendicitis B ) Wilms ' tumor C ) Crohn's disease D ) Small bowel obstruction

B ) Wilms ' tumor

After receiving a course of chemotherapy , a patient begins losing hair . This adverse effect of chemotherapy should be documented as : A ) pediculosis B ) alopecia C ) dandruff D ) hirsutism

B ) alopecia

15. A client's mother was Chinese and the father was Samoan and Filipino. What should the nurse do when assessing this client's race? A Document the mother's race as the client's race. B Ask with which race the client most closely identifies. C Document the father's primary race as the client's race. D Document both of the father's races as the client's race.

B Ask with which race the client most closely identifies.

Which assessment question helps assess immediate memory? A) "How did you get to the hospital today?" B) "Can you repeat the numbers 2, 7, 9 for me?" C) "Do you recall the three items I mentioned earlier?" D) "What was your birth date including the year?"

B) "Can you repeat the numbers 2, 7, 9 for me?"

A patient who sustained a spinal cord injury will perform intermittent self-catheterization after discharge. After discharge teaching, which statement by the patient would indicate correct understanding of the procedure? A) "I will need to replace the catheter weekly." B) "I can use clean, rather than sterile, technique at home." C) "I will remember to inflate the catheter balloon after insertion." D) "I will dispose of the catheter after use and get a new one each time."

B) "I can use clean, rather than sterile, technique at home."

A patient who has not voided for several hours is prescribed an indwelling urinary catheter. How should the nurse explain the procedure the patient? A) "I need to put a Foley in you because you haven't voided since your surgical procedure." B) "I need to insert a tube into your bladder to drain the urine because you haven't urinated since surgery." C) "I need to catheterize you because you haven't urinated since having your surgery." D) "I need to place a catheter in your bladder because you haven't voided since surgery."

B) "I need to insert a tube into your bladder to drain the urine because you haven't urinated since surgery."

The nurse instructs a woman about providing a clean-catch urine specimen. Which statement indicates that the patient correctly understands the procedure? A) "I will be sure to urinate into the 'hat' you placed on the toilet seat." B) "I will wipe my genital area from front to back before I collect the specimen midstream." C) "I will need to lie still while you put in a urinary catheter to obtain the specimen." D) "I will collect my urine each time I urinate for the next 24 hours."

B) "I will wipe my genital area from front to back before I collect the specimen midstream."

A patient asks the nurse about the difference between diphenoxylate with atropine (Lomotil) and the over-the-counter drug loperamide (Imodium). Which response by the nurse is correct? A.) "Lomotil acts faster than Imodium." B) "Imodium does not cause physical dependence." C.) "Lomotil is available in suppository form." D.) "Imodium is a natural antidiarrheal drug."

B) "Imodium does not cause physical dependence

The nurse receives a telephone verbal order for pain medication for a client: morphine 4 mg intravenously every hour as needed for pain. How should the nurse document this telephone order? A) 09/02/17 0845 morphine 4 mg intravenously q 1 hour PRN pain. Kay Andrews, RN B) 09/02/17 0845 morphine 4 mg intravenously q 1 hour PRN pain T.O.: Dr. D.Kelly/Kay Andrews, RN C) 09/02/17 0845 morphine 4 mg intravenously q 1 hour PRN pain V.O.: Dr. D. Kelly/Kay Andrews, RN D) 09/02/17 0845 morphine 4 mg intravenously q 1 hour V.O. Kay Andrews, RN

B) 09/02/17 0845 morphine 4 mg intravenously q 1 hour PRN pain T.O.: Dr. D.Kelly/Kay Andrews, RN

A client's Vital signs 4 hours ago were temperature (oral) 101'4 F (38.6C), heart rate beats/min, respiratory rate 26 breaths/min, and blood pressure 124/78 mm Hg. The temperature is now 99.4°F (37.4°C). Based only on the expected relationship between temperature and respiratory rate, what should the nurse anticipate the client's respiratory rate to be? A) 16 B) 18 C) 20 D) 22

B) 18

The nurse must administer an enema to an adult patient with constipation. Which is a safe and effective distance for the nurse to insert the tubing into the patient's rectum? Select all that apply. A)'2 in. (5.1 cm) B) 3 in. (7.6 cm) C) 4 in. (10.2 cm) D) 5 in. (12.7 cm) E) 6 in. (15.25 cm)

B) 3 in. (7.6 cm) C) 4 in. (10.2 cm)

Considering normal developmental and physical maturation in children, for which age would a goal of "Achieves bowel control by the end of this month" be most realistic? A) 18 months B) 3 years C) 4 years D) 5 years

B) 3 years

The client's temperature is 101.1°F. Which is the correct conversion to centigrade? A) 38.0°C B) 38.4°C C) 38.8°C D) 39.2°C

B) 38.4°C

The nurse manager documents that a staff nurse uses effective critical-thinking skills when planning and providing client care. What did the manager observe to make this decision? Select all that apply. A) Documents client responses to care B) Addresses the most acute cNlient care issues first C) Utilizes knowledge of past client care situations D) Articulates potential solutions to client problems E) Studies the pros and cons of individual interventions

B) Addresses the most acute cNlient care issues first C) Utilizes knowledge of past client care situations D) Articulates potential solutions to client problems E) Studies the pros and cons of individual interventions

Which is considered a primary care service? A) Providing wound care B) Administering childhood immunizations C) Providing drug rehabilitation D) Outpatient hernia repair

B) Administering childhood immunizations

The nurse prepares to meet with assigned clients after receiving hand-off communication. Which statement or question demonstrates that the nurse is in the working phase with a client? A) I see that you live near the hospital. Have you been living here awhile? B) After breakfast we can review the instructions for insulin self-injection again. C) I'm going to miss talking with you ever day but you are better and ready to go home now. D) As soon as I get your admission papers I'll be in to talk with you about your health problems.

B) After breakfast we can review the instructions for insulin self-injection again.

A mother brings her 6-month-old infant to the clinic for a well-baby checkup. How should the nurse proceed when weighing the patient? A) Have the mother remain outside the room. B) Ask the mother to remove the infant's clothing and diaper. C) Weigh the infant wearing only the diaper. D) Place the infant supine on the scale with his knees extended.

B) Ask the mother to remove the infant's clothing and diaper.

The surgeon enters a computerized order for a patient in the postoperative period after a unilateral thoracotomy for lung cancer. The order states: OOB in AM. Which action indicates that the nurse is the following the surgeon's order ? A) Performs oral care B) Assists the patient out of bed C) Assists the patient with bathing D) Changes the patient's operative dressings

B) Assists the patient out of bed

When should the nurse instruct a new mother to expect the anterior fontanel of her infant to fuse? A) At about 8 weeks B) At about 14 months C) By 6 months of age D) Before I year of age

B) At about 14 months

Which assessment should the nurse perform if a palpable thyroid gland is present? A.) Illuminate the thyroid gland for the presence of fluid. B) Auscultate the thyroid gland for bruits. C ) Percuss the thyroid gland for mass size. D.) Measure the thyroid gland to assess change.

B) Auscultate the thyroid gland for bruits.

A patient's blood group is B. From which donor groups can this patient receive blood? Select all that apply. A) A B) B C) O D) AB

B) B C) O

A middle-aged patient with a history of alcohol abuse is admitted with acute pancreatitis. This patient will most likely be deficient in which nutrient? A) Iron B) B vitamins C) Calcium D) Phosphorus

B) B vitamins

A client who follows a vegan eating plan is experiencing numbness and tingling of the fingers and toes. Which supplement should the nurse recommend to this client? A) C B) B12 C) Zinc D) Magnesium

B) B12

A client asks what causes the urine to have an odor. What should the nurse respond to this client? Select all that apply. A) Beets cause a fragrant odor. B) Bacteria cause an ammonia odor. C) Onions can cause a distinctive odor. D) Spices cause a vanilla-smelling odor. E) A congenital disorder causes a sweet odor.

B) Bacteria cause an ammonia odor. C) Onions can cause a distinctive odor. E) A congenital disorder causes a sweet odor.

24. A client who does not speak English needs extensive care. What resources are available to maximize communication with this client? Select all that apply. A) U.S. embassies B) Bilingual staff C) Telephone interpreters D) Client's family members E) Face-to-face interpreters

B) Bilingual staff C) Telephone interpreters E) Face-to-face interpreters

Which abnormal laboratory value is associated with icteric sclerae? A) Bleeding time B) Bilirubin C) Hemoglobin D) Glucose

B) Bilirubin

A patient newly diagnosed with breast cancer tells the nurse, I'm worried I won't live to see my children grow up. Which response by nurse best conveys concern and active listening? A) There have been many advances in breast cancer treatment; hope for the best B) Breast cancer is a serious disease; I can understand why you're worried C) You're strong and have youth on your side to fight the breast cancer D) I'd be worried, I've seen a lot of patients die of breast cancer

B) Breast cancer is a serious disease; I can understand why you're worried

The healthcare team suspects that a patient has an intestinal infection. Which action should the nurse take to help confirm the diagnosis? A) Prepare the patient for an abdominal flat plate. B) Collect a stool specimen that contains 20 to 30 mL of liquid stool. C) Administer a laxative to prepare the patient for a colonoscopy. D) Test the patient's stool using a fecal occult test.

B) Collect a stool specimen that contains 20 to 30 mL of liquid stool.

The mother of a 4-year-old child with painful bruises and facial contusions explains that the mother's significant other pushed the child down the stairs in anger. What should the nurse do first? A) Notify the nursing supervisor of the suspected physical abuse. B) Complete a physical assessment of the child. C) Obtain an order for pain medication. D) Notify Child Protective Services of the suspected abuse.

B) Complete a physical assessment of the child.

A patient with trigeminal neuralgia is prescribed a mechanical soft diet. This diet places the patient at risk for which complication? A) Dehydration B) Constipation C) Hyperglycemia D) Diarrhea

B) Constipation

The nurse prepares a teaching session on blood pressure for a group of nursing students. What should the nurse explain can falsely elevate the blood pressure measurement? Select all that apply. A) Cuff that is too wide B) Cuff that is too narrow C) Mild to moderate pain present D) Measuring after the client smokes E) Measuring after a client ambulates

B) Cuff that is too narrow C) Mild to moderate pain present D) Measuring after the client smokes E) Measuring after a client ambulates

Patients may be deficient in which vitamin during the winter months? A) A B) D C) E D) K

B) D

When the nurse assesses the patient's tongue, she notes that it appears dry and An 85-year-old patient is brought to the emergency department with lethargy and furry. What does this finding indicate to the nurse? A) Fungal infection B) Dehydration C) Allergy D) Iron deficiency

B) Dehydration

What can the nurse assess using Erik Erikson's theory? A) Moral development B) Developmental tasks C) Social identity D) Self-esteem

B) Developmental tasks

A client becomes verbally abusive to the nurse after having an argument with the spouse. Which coping mechanism is this client exhibiting? A) Reaction formation B) Displacement C) Denial D) Conversion

B) Displacement

When changing a diaper, the nurse observes that a 2-day-old infant has passed a green-black, tarry stool. What should the nurse do? A) Notify the provider immediately. B) Do nothing; this is normal. C) Give the baby sterile water until the mother's milk comes in. D) Apply a skin barrier cream to the buttocks to prevent irritation.

B) Do nothing; this is normal.

The nurse is caring for a patient of Japanese heritage who refuses pain medication despite the nurse's explaining its importance in the healing process. Which intervention by the nurse is appropriate for this patient? Select all that apply. A) Assess the patient's pain levels at less frequent intervals. B) Document in the patient's record that the patient does not want to take opioids. C) Utilize nonpharmacological measures to help control the patient's pain. D) Notify the primary care provider of the patient's noncompliance. E) Provide the medication without telling the patient.

B) Document in the patient's record that the patient does not want to take opioids. C) Utilize nonpharmacological measures to help control the patient's pain.

A patient is prescribed the narcotic hydromorphone ( Dilaudid ) . Where should the nurse expect to retrieve this drug for administration ? A) Cabinet in the patient's room B) Double - locked medication drawer C) Stock supply cabinet D ) Portable medication cart

B) Double - locked medication drawer

Which information in a client's health history might indicate a risk for primary hypertension? A) Consumes a high-protein diet B) Drinks three to four beers every day C) Has a family history of kidney disease D) Does not engage in physical exercise

B) Drinks three to four beers every day

An older client requires an extended hospital stay to treat a postsurgical complication. How can the medical-surgical nurse best promote self-esteem in this patient? A) Assist the patient to ambulate in the hallway once daily. B) Encourage the patient to participate in self-care. C) Introduce herself to the patient if he does not know her. D) Listen attentiveluhen thanatient sneaks

B) Encourage the patient to participate in self-care.

A resident in a long-term care facility receiving Medicare funds requires care for a stage 2 pressure ulcer. How often must the nurse document this patient's care A) Every 2 weeks B) Every shift C) Every week D) Every 3 months

B) Every shift

A female patient refuses to have a male nurse provide care. Which intervention by the charge nurse is best? A) Explain that hospital policy does not allow nursing assignments based on the gender of the nurse. B) Explore with the patient her beliefs and determine which might have caused her to make this statement. C) Assure the patient that each nurse is capable of providing professional nursing care, regardless of gender. D) Comply with the patient's request and assign a female nurse to care for the patient.

B) Explore with the patient her beliefs and determine which might have caused her to make this statement.

A middle-aged Chinese American client, who is self-employed without health insurance, is recovering from heart surgery. Each day family members spend hours at the bedside. Which is the most important factor for the nurse to focus on when planning discharge? A.Ethnic background B) Family support C) Employment status D) Healthcare coverage

B) Family support

The nurse manager suspects that a staff nurse is taking narcotics from the care area. If reported, of what will this staff nurse be accused? A) Libel B) Felony C) Battery D) Misdemeanor

B) Felony

Which nursing intervention specifically helps reduce a patient's anxiety? A) Teaching the importance of adequate nutrition and hydration B) Giving elear facts pertaining to the patient's eircumstances C) Promoting small-group activities to improve self-esteem D) Monitoring the patient for the risk of suicide

B) Giving elear facts pertaining to the patient's eircumstances

Which is an example of the most basic motivation in Maslow's hierarchy of needs? A) Experiencing loving relationships B) Having adequate housing C) Receiving education D) Living in a crime-free neighborhood

B) Having adequate housing

The nurse prepares to calculate a client's body mass index (BMI), What will the nurse need to measure to make this calculation? Select all that apply. A) Pulse B) Height C) Weight D) Blood pressure E) Respiratory rate

B) Height C) Weight

A patient is prescribed furosemide (Lasix), a loop diuretic, for treatment of congestive heart failure. The patient is risk for which electrolyte imbalance associated with use of this drug? A) Hypocalcemia B) Hypokalemia C) Hypomagnesemia D) Hypophosphatemia

B) Hypokalemia

Which statement by the nurse demonstrates that active listening has occurred? Select all that apply. A) I listened to my patient while I was changing his IV site. B) I made eye contact and listened to my patient to find out his information C) I took notes when I listened to my patient describe his symptoms. D) I sat with my patient and his wife to talk about their fears before the surgery. E) I find the best time to talk with my patient is during the morning bath.

B) I made eye contact and listened to my patient to find out his information D) I sat with my patient and his wife to talk about their fears before the surgery.

The nurse must insert a nasogastric (NG) tube into a patient with a bowel obstruction. Before inserting the tube, the nurse must explain the procedure to the patient. Which explanation by the nurse is best, assuming that all provide correct information? A) I'm going to insert an NG tube and connect it to low Gomco to keep your stomach empty B) Im going to a tube through your nose into your stomach to prevent you from vomiting. C) I'm going to insert an NG tube through your nares to suction your secretions and prevent emesis. D) Lies still, please; I need to elevate the head of the bed and insert this tube

B) Im going to a tube through your nose into your stomach to prevent you from vomiting.

Which patient is most likely experiencing positive nitrogen balance? A) Third-degree burns of his legs B) In the sixth month of a healthy pregnancy C) Resident from a nursing home who has been refusing to eat D) Experiencing acute pancreatitis

B) In the sixth month of a healthy pregnancy

What impact does hypertensive medication have on skin integrity and wound healing? A) Causes cellular toxicity B) Increases the risk of ischemia C) Delays wound healing D) Predisposes to hematoma formation

B) Increases the risk of ischemia

Which portion of the ear is responsible for maintaining equilibrium? A.) External ear B.) Inner ear C) Middle ear D) Ossicles

B) Inner Ear

A patient has recently had a change in a family relationship that is greatly affecting his health. Which nursing diagnosis would be most appropriate for this patient? A) Parental Role Conflict B) Interrupted Family Processes C) Compromised Family Coping D) Ineffective Coping

B) Interrupted Family Processes

The nurse manager of the medical intensive care unit formed a group to help the staff cope with stress more effectively. Which of the comments by a group member causes the manager to evaluate the group as successful? A) This was a good idea to form a group; I've been wanting to get to know some of the people working the other shifts. B) It really helps me to share feelings about how hard it is to see pain and suffering every day C) I now have a group to help me when I need to work through situations in my own life causing me stress. D) It feels good to have a chance to get away from the unit and talk on a regular basis.

B) It really helps me to share feelings about how hard it is to see pain and suffering every day

The nurse assesses a client from a non-English-speaking culture. Why should the nurse avoid using the term minority when asking this client about ethnicity and race? A) It represents power. B) It suggests inferiority. C) It is the group with the most authority. D) It explains how everyone else is inferior.

B) It suggests inferiority.

Which is considered a religious denomination within the tradition of Christianity? A) Buddhism B) Jehovah's Witnesses C) Sikhism D) Islam

B) Jehovah's Witnesses

The nurse developed a care plan for a patient to help prevent Impaired Skin Integrity. The nurse ensured the nursing assistive personnel changed the patient's position every 2 hours. In the evaluation phase of the nursing process, which would the nurse do first? A) Determine whether she has gathered enough assessment data. B) Judge whether the interventions achieved the stated outcomes. C) Follow up to verify that care for the nursing diagnosis was given. D) Decide whether the nursing diagnosis was accurate for the patient's condition

B) Judge whether the interventions achieved the stated outcomes.

Which laboratory result should alert the nurse to a potential problem? A) Na" = 137 mEg/L B) K+= 5.2 mEg/L C) Ca = 9.2 mg/dL D) Mg = %3D 1.8 mg/dL

B) K+= 5.2 mEg/L

The nurse must irrigate the colostomy of a patient who is unable to move independently. How should the nurse position the patient for this procedure? A) Semi-Fowler's position B) Left side-lying position C) Supine with the head of the bed lowered flat D) Supine with the head of bed raised to 30 degrees

B) Left side-lying position

The nurse maintains eye contact while a client explains the symptoms of a current health problem. Which critical-thinking skill is the nurse using? A) Separating relevant from irrelevant data B) Listening carefully; reading thoughtfully C) Organizing or grouping information in meaningful ways D) Making inferences about the meaning of the information

B) Listening carefully; reading thoughtfully

Where in the body is glucose stored? Select all that apply. A) Brain B) Liver C) Skeletal muscles D) Smooth muscles E) Intestines

B) Liver C) Skeletal muscles

During the alarm stage of the general adaptation syndrome, which metabolic change occurs? Select all that apply. A) Rate of metabolism decreases. B) Liver converts more glycogen to glucose. C) Use of amino acids decreases. D) Amino acids and fats are more available for energy. E) The brain uses more glucose for energy.

B) Liver converts more glycogen to glucose. D) Amino acids and fats are more available for energy. E) The brain uses more glucose for energy.

Which factor in a patient's medical history is most likely to prolong the half-life of certain drugs? A) Heart disease B) Liver disease C) Rheumatoid arthritis D) Tobacco use

B) Liver disease

Which medication class will the primary care provider most likely prescribe to increase urine output in the patient with congestive heart failure? A) Thiazide diuretic B) Loop diuretic C) MAO inhibitor D) Anticholinergic

B) Loop diuretic

A few nurses on a unit have proposed to the nurse manager that the process for documenting care on the unit be changed. They have described a completely new system. The nurse should have a critical attitude because it helps the manager to do which item? A) Consider all the possible advantages and disadvantages. B) Maintain an open mind about the proposed change, C) Apply the nursing process to the situation. D) Make a decision based on past experience with documentation.

B) Maintain an open mind about the proposed change,

Which is an appropriate goal for a patient with urinary incontinence? Select all that apply. A) Increase the intake of citrus fruits. B) Maintain daily oral fluids to 8 to 10 servings per day. C) Limit daily caffeine intake to less than 100 mg. D) Engage in high-impact, aerobic exercise. E) Lose weight if BMI is greater than 30.

B) Maintain daily oral fluids to 8 to 10 servings per day. C) Limit daily caffeine intake to less than 100 mg. E) Lose weight if BMI is greater than 30.

Which intervention helps a client maintain a sense of personhood during hospitalization? Select all that apply. A) Addressing the patient by his first name B) Making eye contact if it is comfortable for the patient C) Always offering an explanation before beginning a procedure D) Speaking to others about the patient so that the patient can hear you E) Listening actively when the patient speaks

B) Making eye contact if it is comfortable for the patient C) Always offering an explanation before beginning a procedure E) Listening actively when the patient speaks

Upon initial assessment, the nurse notes bruises and scratches on the arms, legs, and trunk an older client. With which state law is the nurse complying when the supervisor is notified regarding this patient as a potential victim of abuse? A) Good Samaritan Law B) Mandatory Reporting Law C) Nurse Practice Act D) Nursing Standards of Practice

B) Mandatory Reporting Law

Which is an appropriate intervention for a patient with hypovolemia? Select all that apply. A) Teach deep-breathing techniques. B) Monitor I&O daily. C) Encourage fluid intake. D) Monitor electrolyte balance. E) Measure daily weights.

B) Monitor I&O daily. C) Encourage fluid intake. D) Monitor electrolyte balance. E) Measure daily weights.

When using the SBAR model to communicate with a physician, what information does the nurse offer first? A) Statement of the problem and its probable cause B) Nurse's name, patient's name, and reason for the communication C) History of information related to and leading up to the situation D) A solution to the problem or what is needed from the physician

B) Nurse's name, patient's name, and reason for the communication

Which is the biggest disadvantage of having nursing assistive personnel (NAP) assist nurses in practice? A) Nurses must know what aspects of care can legally and safely be delegated to the NAP. B) Nurses may rely too heavily on information gathered by the NAP when making patient care decisions. C) Nurses are removed from many components of direct patient care that have been delegated to the NAP. D) Nurses still maintain responsibility for the patient care given by the NAP.

B) Nurses may rely too heavily on information gathered by the NAP when making patient care decisions.

The nurse in a long-term care facility is teaching a group of residents about increasing dietary fiber. Which foods should the nurse explain are high in fiber? A) White bread, pasta, and white rice B) Oranges, raisins, and strawberries C) Whole milk, eggs, and bacon D) Peaches, orange juice, and bananas

B) Oranges, raisins, and strawberries

The nursing assistive personnel (NAP) informs the nurse that a patient has fallen out of bed and is in pain. The nurse assesses the patient and provides care. How should the nurse document the patient's fall ? A) Patient found on floor in pain after falling out of bed. B) Patient found on floor after falling out of bed; found by NAP Smith. C) Patient fell out of bed but is currently in bed. D) Patient reminded to not climb OOB after falling.

B) Patient found on floor after falling out of bed; found by NAP Smith.

Assuming that all are accurate, which documentation about a patient's level of consciousness is best? A) Patient is lethargic and slept when undisturbed. B) Patient responds to tactile stimulation; falls back to sleep immediately after tactile and verbal stimulation are stopped. C) Patient slept throughout the day, missing his meals and bath. D) Patient appears to be tired as he slept throughout the day except when bathed.

B) Patient responds to tactile stimulation; falls back to sleep immediately after tactile and verbal stimulation are stopped.

Which outcome is appropriate for the patient who underwent urinary diversion surgery and creation of an ileal conduit for invasive bladder cancer? A) Patient will resume his normal urination pattern by (target date). B) Patient will perform urostomy self-care by (target date). C) Patient will perform self-catheterization by (target date). D) Patient's urine will remain clear with sufficient volume.

B) Patient will perform urostomy self-care by (target date).

The nurse is caring for a client who emigrated from Puerto Rico. What should the nurse learn to best care for this client? A) Practices of the ethnic group B) Patient's individual cultural beliefs C) Values of her own culture D) Spanish-speaking community

B) Patient's individual cultural beliefs

The staff development trainer creates a program on culture for a group of new nurses. What should be included about vulnerable populations? Select all that apply. A) Vulnerable populations exist primarily in minority groups. B) People in a vulnerable population have limited access to healthcare. C) Individuals in vulnerable groups are more likely to have health problems. D) Examples of vulnerable populations include the poor, very young, and very old. E) A person in a vulnerable population is more prone to engage in high-risk

B) People in a vulnerable population have limited access to healthcare. C) Individuals in vulnerable groups are more likely to have health problems. D) Examples of vulnerable populations include the poor, very young, and very old. E) A person in a vulnerable population is more prone to engage in high-risk

A client with sluggish bowel movements asks what causes evacuation to occur. What muscle activity should the nurse explain this client? Select all that apply. A) Flatus B) Peristalsis C) Mass peristalsis D) Haustral churning E) Valsalva maneuver

B) Peristalsis C) Mass peristalsis D) Haustral churning

A client with sluggish bowel movements asks what causes evacuation to occur. What muscle activity should the nurse explain to this client? Select all that apply. A) Flatus B) Peristalsis C) Mass peristalsis D) Haustral churning R) Valsalva maneuver

B) Peristalsis C) Mass peristalsis D) Haustral churning

A patient with chronic renal failure will be dialyzed for the first time the following morning. Which is an appropriate nursing intervention for the patient? Select all that apply. A) Encourage oral fluid intake as desired. B) Place the patient on strict I&O. C) Weigh the patient before and after dialysis. D) Maintain a total fluid restriction of 1,000 mL as prescribed. E) Monitor laboratory values.

B) Place the patient on strict I&O. C) Weigh the patient before and after dialysis. D) Maintain a total fluid restriction of 1,000 mL as prescribed. E) Monitor laboratory values.

Which laboratory test result most accurately reflects a patient's nutritional status? A) Albumin B) Prealbumin C) Transferrin D) Hemoglobin

B) Prealbumin

Which statement pertaining to Benner's practice model for clinical competence is true? A) Progression through the stages is constant, with most nurses reaching the proficient stage. B) Progression through the stages involves continual development of thinking and technical skills. C) The nurse must have experience in many areas before being considered an expert. D) The nurse's progress through the stages is determined by years of experience and skills.

B) Progression through the stages involves continual development of thinking and technical skills.

Which nutrient deficiency increases the risk for pressure ulcers? A) Carbohydrate B) Protein C) Fat D) Vitamin K

B) Protein

An 80-year-old patient fell and fractured her hip and is in the hospital. Before the fall, the patient lived at home with her spouse and managed their activities of daily living (ADLs) very well. The hospital is ready to discharge the patient because the recommended length of stay in a hospital has been exceeded. However, she cannot walk or perform ADLs independently. She requires lengthy physical therapy and further monitoring of medications and both physical and mental status. To which type of facility should the patient be transferred? A) Nursing home B) Rehabilitation center C) Outpatient therapy center D) None of these; she should receive home healthcare

B) Rehabilitation center

The nurse hears rhonchi when auscultating a client's lungs. Which nursing intervention would be appropriate for the nurse to implement before reassessing lung sounds? A) Have the client take several deep breaths. B) Request the client take a deep breath and cough. C) Take the client's blood pressure and apical pulse. D) Count the client's respiratory rate for 1 minute.

B) Request the client take a deep breath and cough.

Which test should the patient undergo when the Weber test is positive? A) Romberg test B) Rinne test C) Pure tone audiometry D) Tympanometry

B) Rinne test

Based only on Maslow's hierarchy of needs, which nursing diagnosis should have the highest priority? A) Self-care Deficit B) Risk for Aspiration C) Impaired Physical Mobility D) Disturbed Sensory Perception

B) Risk for Aspiration

A client requires sedation, intubation, and mechanical ventilation for 2 weeks. What would an appropriate nursing diagnosis for the client? A) Risk for Infection related to subcutaneous injuries B) Risk for Impaired Skin Integrity related to immobility C) Impaired Tissue Integrity related to ventilator dependency D) Impaired Skin Integrity related to ventilator dependency

B) Risk for Impaired Skin Integrity related to immobility

Which is an example of theoretical knowledge? A) A nurse uses sterile technique to catheterize a patient. B) Room air has an oxygen concentration of 21%. C) Glucose monitoring machines should be calibrated daily. D) An irregular apical heart rate should be compared with the radial pulse.

B) Room air has an oxygen concentration of 21%.

A patient is recovering from surgery for a ruptured appendix. Because the surgeon did not surgically close the wound, what wound healing process will occur? A) Primary intention B) Secondary intention C) Tertiary intention D) Approximation

B) Secondary intention

For which purpose is a graphic flow sheet superior to other methods of recording data? A) Easy documentation of routine vital signs B) Seeing the patterns of a patient's fever C) Describing the symptoms accompanying a rising temperature D) Checking to make sure vital signs were taken

B) Seeing the patterns of a patient's fever

A patient undergoing fertility treatments for the past 9 months learms that despite in-vitro fertilization she still is not pregnant. Which component of self-concept is this patient at risk for experiencing a crisis? A) Body image B) Self-esteem C) Personal identity D) Role performance

B) Self-esteem

The nurse is preparing to admit a patient from the emergency department. The transferring nurse reports that the patient is obese. The nurse has been overweight at one time and works very hard now to maintain a healthy weight. She immediately thinks, "I know I tend to feel negatively about obese people; I figure if I can stop eating, they should be able to. I must remember how very difficult that is and be very careful not to be judgmental of this patient." Which is the nurse illustrating? A) Theoretical knowledge B) Self-knowledge C) Using reliable resources D) Use of the nursing process

B) Self-knowledge

The nurse is preparing to admit a patient from the emergency department. The transferring nurse reports that the patient is obese. The nurse has been overweight at one time and works very hard now to maintain a healthy weight.She immediately thinks, "I know I tend to feel negatively about obese people; I figure if I can stop eating, they should be able to. I must remember how very difficult that is and be very careful not to be judgmental of this patient.Which is the nurse illustrating? A) Theoretical knowledge B) Self-knowledge C) Using reliable resources D) Use of the nursing process

B) Self-knowledge

While assessing a new wound, the nurse notes red, watery drainage. What type of drainage will the nurse document this as? A) Sanguineous B) Serosanguineous C) Serous D) Purosanguineous

B) Serosanguineous

17. A patient who lost his job last month has now been told that his wife wants a divorce. He says, "I know I don't have much to offer a woman. She wants more than what I am, and now I'm not even bringing home any money." Which nursing diagnosis is most appropriate? A) Chronic Low Self-Esteem B) Situational Low Self-Esteem C) Disturbed Personal Identity D) Disturbed Body Image

B) Situational Low Self-Esteem

The nurse designs goals to address a client's risk for health issues. What should the nurse include when creating these goals? A) Methods to prevent complications B) Strategies to prevent the health issues C) Approaches to resolve the health issue D) Actions to rule out potential health issues

B) Strategies to prevent the health issues

When assessing the quality of a client's pedal pulses, what is the nurse assessing ? Select all that apply. A) Rhythm of the pulses B) Strength of the pulses C) Bilateral equality of pulses D) Rate compared with apical pulse E) Location of the pulse

B) Strength of the pulses C) Bilateral equality of pulses

A patient tells the nurse "I'm having alot of pain in my hip "Which response by the nurse is open-ended and would stimulate the patient to provide the most complete data? Select all that apply. A) Is your pain severe? B) Tell me about your pain. C) When did you first notice this pain ? D) How would you describe your pain? E) What makes the pain worse ?

B) Tell me about your pain. D) How would you describe your pain? E) What makes the pain worse ?

A patient being admitted in hypertensive crisis reports stopping blood pressure medication 3 weeks ago. Which response by the nurse is best? A) You're lucky you didn't have a stroke; you really need to take your medication B) Tell me more about your experience with your high blood pressure medication C) Why did you stop taking your high blood pressure medication D) It's very important to take your blood pressure medication

B) Tell me more about your experience with your high blood pressure medication

Which example includes both objective and subjective data? A) The client's blood pressure is 132/68 and her heart rate is 88. B) The client's cholesterol is elevated, and he states he likes fried food. C) The client states she has trouble sleeping and that she drinks coffee in the evening. D) The client states he gets frequent headaches and that he takes aspirin for the pain.

B) The client's cholesterol is elevated, and he states he likes fried food.

In which circumstance might the nurse defer obtaining informed consent for care and treatment of a patient? A) The patient is confused and cannot understand or sign the consent form. B) The patient is brought to the emergency department in cardiac arrest; no family is present. C) The surgeon requests that the patient be sent to the surgical suite before you get the consent form signed. D) An unconscious patient is admitted to your unit; he is alone.

B) The patient is brought to the emergency department in cardiac arrest; no family is present.

What do critical thinking and the nursing process have in common? A) They are both linear processes used to guide one's thinking. B) They are both thinking methods used to solve a problem. C) They both use specific steps to solve a problem. D) They both use similar steps to solve a problem.

B) They are both thinking methods used to solve a problem.

Two days after a patient undergoes abdominal surgery, the surgical incision is red and slightly edematous; it is oozing a small amount of serosanguineous (pink-tinged serous) fluid. On the basis of these data, what can the nurse conclude? Select all that apply. A) The wound is most likely infected. B) This is a vascular response to inflammation. C) Damaged cells are being regenerated. D) Exudate formation is occurring. E) The wound is beginning the repair process.

B) This is a vascular response to inflammation. D) Exudate formation is occurring.

The nurse is updating a care plan for a patient who has a nursing diagnosis of Anxiety. Which patient behavior might suggest that the problem is resolving? A) Pacing in the hallway at intervals B) Using relaxation techniques C) Speaking rapidly when spoken D) Avoiding eye contact

B) Using relaxation techniques

The nurse records a patient's hourly urine output from an indwelling catheter as follows: 0700: 36 mL 0800: 45 ml 0900: 85 ml. 1000: 62 mL 1100: 50 mL 1200: 48 mL 1300: 94 ml 1400: 78 ml. 1500: 60 ml. How should the nurse describe the patient's urine output? A) Low B) Within normal limits C) High D) Inconclusive

B) Within normal limits

Which statement about the nursing process is correct? A.) It was developed from the ANA Standards of Care. B. ) It is a problem-solving method to guide nursing activities. C.) It is a linear process with separate, distinct steps. D. ) It involves care that only the nurse will give.

B. ) It is a problem-solving method to guide nursing activities.

The nurse maintains eye contact while a client explains the symptoms of a current health problem. Which critical-thinking skill is the nurse using? A.) Separating relevant from irrelevant data B. ) Listening carefully; reading thoughtfully C.) Organizing or grouping information in meaningful ways D.) Making inferences about the meaning of the information

B. ) Listening carefully; reading thoughtfully

A patient was diagnosed with pancreatic cancer last month, and has complained of a dull ache in the abdomen for the past 4 months. This pain has been gradually increasing, and the pain relievers taken at home are no longer effective. What type of pain is the patient experiencing? a. Acute pain b. Chronic pain c. Somatic pain d. Neuropathic pain

B. Chronic pain

Which individual established the American Red Cross? A) Louisa May Alcott B) Clara Barton C) Dorothea Dix D) Harriet Tubman

B. Clara Barton

A patient has left-sided weakness because of a recent stroke. Which type of special needs assessment would it be most important to perform? A) Family B) Functional C) Community D) Psychosocial

B. Functional

The nurse notes in a patient's medication history that the patient is taking allopurinol (Zyloprim). Based on this finding, the nurse interprets that the patient has which disorder? a. Rheumatoid arthritis b. Gout c. Osteoarthritis d. Systemic lupus erythematosus

B. Gout

Which form of communication is the nurse using when interviewing the patient during the admission health history and physical assessment? A) Small group B) Interpersonal C) Group D) Intrapersonal

B. Interpersonal

The nurse is reviewing herbal therapies. Which is a common use of the herb feverfew? a. Muscle aches b. Migraine headaches c. Leg cramps d. Incision pain after surgery

B. Migraine headaches

Which is an example of a problem that nurses can treat independently? A) Hemorrhage B) Nausea C) Fracture D) Infection

B. Nausea

Which client would probably have a higher than normal respiratory rate? A) Recovering from surgery and receiving a narcotic analgesic B) Recovering from surgery and lost a unit of blood intraoperatively C) Lived at a high altitude and then moved to sea level D) Exposed to the cold and is now hypothermic

B. Recovering from surgery and lost a unit of blood intraoperatively

When measuring a blood pressure, which step is correct? Select all that apply. A) Use a bladder that encircles 40% of the arm. B) Wrap the cuff snugly around the clients arm. C) Ask the client to hold the arm at heart level. D) Have the client sit with feet flat on the floor. E) Roll up a sleeve before applying the cuff.

B. Wrap the cuff snugly around the client's arm D. Have the client sit with feet flat on the floor

The Joint Commission requires which type of assessment to be performed on all patients? A) Functional ability B) Pain C) Cultural D) Wellness

B. pain

The nurse is teaching a pregnant woman about the increased oxygen demand that develops during pregnancy. Which client statement indicates that teaching has been effective? A.) "I may need to drink more fluids in order to make more oхуen." B.) "I may need to take an iron supplement so that I am not anemic." C) "I will necd a multivitamin supplement for several months." D.) "I will need to cat more fruits and vegetables."

B.) "I may need to take an iron supplement so that I am not anemic."

The nurse is providing instructions to a patient who has a new prescription for a corticosteroid metered-dose inhaler. Which statement by the patient indicates that further instruction is needed? (Select all that apply.) A.) "I will rinse my mouth with water after using the inhaler and then spit out the water." B.) "I will gargle after using the inhaler and then swallow." C.) "I will clean the plastic inhaler casing weekly by removing the canister and then washing the casing in warm soapy water. I will then let it dry before reassembling." D.)"I will use this inhaler for asthma attacks. E.) "I will continue to use this inhaler, even if I am feeling better." F.) "I will use a peak flow meter to measure my response to therapy."

B.) "I will gargle after using the inhaler and then swallow." D.)"I will use this inhaler for asthma attacks.

A patient has been advised to add a nasal spray (an adrenergic decongestant) to treat a cold. The nurse will include which instruction? A.) "You won't see effects for at least 1 week." B.) "Limit use of this spray to 3 to 5 days." C.) "Continue the spray until nasal stuffiness has resolved." D. ) "Avoid use of this spray if a fever develops."

B.) "Limit use of this spray to 3 to 5 days."

A mother calls the pediatrician's office to report that her 18-month-old child has eaten half of a bottle of baby aspirin. She says, "I have a bottle of syrup of ipecac. Should I give it to him? He seems fine right now. What do I do?" What is the nurse's best response? A.) "Go ahead and give him the ipecac, and then call 911. B.) . "Don't give him the ipecac. Call the Poison Control number immediately for instructions." C. ) "Please come to the office right away so that we can check him." D. ) "Go ahead and take him to the emergency room right now."

B.) . "Don't give him the ipecac. Call the Poison Control number immediately for instructions."

A patient is in an urgent care center with an acute asthma attack. The nurse expects that which medication will be used for initial treatment? A.) An anticholinergic such as ipratropium(Atrovent) B.) A short-acting betaz agonist such as albuterol (Proventil) C.) A long-acting beta, agonist such as salmeterol (Serevent) D.) A corticosteroid such as fluticasone (Flovent)

B.) A short-acting betaz agonist such as albuterol (Proventil)

The nurse is preparing to administer methylnaltrexone (Relistor), a peripherally acting opioid antagonist. This drug is appropriate for which patient? A.) A patient with diarrhea B.) A terminally ill patient who has opioid-induced constipation C.) A patient who is scheduled for a colonoscopy D.) A patient who will be having colon surgery in the morning

B.) A terminally ill patient who has opioid-induced constipation

The nurse manager documents that a staff nurse uses effective critical-thinking skills when planning and providing client care. What did the manager observe to make this decision? Select all that apply. A. ) Documents client responses to care B.) Addresses the most acute client care issues first C. ) Utilizes knowledge of past client care situations D. ) Articulates potential solutions to client problems E. ) Studies the pros and cons of individual interventions

B.) Addresses the most acute client care issues first C. ) Utilizes knowledge of past client care situations D. ) Articulates potential solutions to client problems E. ) Studies the pros and cons of individual interventions

A patient is on a chemotherapy regimen an outpatient clinic and is receiving a chemotherapy drug that is known to be highly emetogenic. The nurse will implement which interventions regarding the pharmacologic management of nausea and vomiting? (Select all that apply.) A.) Giving antinausea drugs at the beginning of the chemotherapy infusion B.) Administering antinausea drugs 30 to 60 minutes before chemotherapy is started C.) For best therapeutic effects, medicating for nausea once the symptoms begin D.) Observing carefully for the adverse effects of restlessness and anxiety E.) Instructing the patient that the antinausea drugs may cause extreme drowsiness F.) Instructing the patient to rise slowly from a sitting or lying position because of possible orthostatic hypotension

B.) Administering antinausea drugs 30 to 60 minutes before chemotherapy is started E.) Instructing the patient that the antinausea drugs may cause extreme drowsiness F.) Instructing the patient to rise slowly from a sitting or lying position because of possible orthostatic hypotension

The nurse is reviewing medications for the treatment of asthma. Which drugs are used for acute asthma attacks? (Select all that apply.) A.) Salmeterol (Serevent) inhaler B.) Albuterol (Proventil) nebulizer solution C.) Epinephrine D.) Montelukast (Singulair) E.). Fluticasone (Flovent) Rotadisk inhaler

B.) Albuterol (Proventil) nebulizer solution C.) Epinephrine

A client recovering from a respiratory infection is concerned about a new onset of diarrhea. What should the nurse assess in this client? A.) Last use of steroids B.) Amount of vitamin C ingested C.) Frequency of decongestant use D.) Use of over-the- counter antitussives

B.) Amount of vitamin C ingested

The nurse is reviewing the mechanism of action of antidiarrheal drugs. Which type of antidiarrheal medication works by decreasing the intestinal muscle tone and peristalsis of the intestines? A.). Adsorbents such as Pepto-Bismol B.) Anticholinergics such as belladonna alkaloids C.) Probiotics such as Lactinex D.) Lubricants such as mineral oil

B.) Anticholinergics such as belladonna alkaloids

Which nursing intervention reduces the risk of clot formation in the legs? Select all that apply. A.)Keep the patient's hips and knees flexed while the patient is in bed. B.) Apply compression devices (e.g., sequential compression devices [SCDS]). C.)Turn the patient frequently or encourage frequent position changes. D.) Promote adequate hydration by encouraging oral intake. E.) Elevate the patient's legs above the level of the heart.

B.) Apply compression devices (e.g., sequential compression devices [SCDS]). C.)Turn the patient frequently or encourage frequent position changes. D.) Promote adequate hydration by encouraging oral intake. E.) Elevate the patient's legs above the level of the heart.

After the ventilator is discontinued from a client with a cervical injury, the nurse stays with the client to provide support. Which moral principle is the nurse demonstrating? A.) Nonmaleficence B.) Autonomy C.) Beneficence D.) Fidelity

B.) Autonomy

Which medication would the nurse expect to be prescribed for a patient with heart failure? Select all that apply. A.) Nitrates B.) Beta-adrenergic agents C.) Diuretics D.) Anticoagulants E. ) Anticholesterol agents

B.) Beta-adrenergic agents C.) Diuretics

A patient asks the nurse about the uses of echinacea. Which use will the nurse include in the response? A.) Memory enhancement B.) Boosting the immune system C). Improving mood D.) Promoting relaxation

B.) Boosting the immune system

A patient wants to prevent problems with constipation and asks the nurse for advice about which type of laxative is safe to use for this purpose. Which class of laxative is considered safe to use on a long-term basis? A.) Emollient laxatives B.) Bulk-forming laxatives C.) . Hyperosmotic laxatives D.) Stimulant laxatives

B.) Bulk-forming laxatives

Chronic stress may lead directly to cardiovascular disease because of the repeated release of which chemical? A.) Histamine B.) Catecholamines C.) Cortisol D.) Protease

B.) Catecholamines

The nurse prepares material to teach a group of family members on cardiopulmonary resuscitation. Which information should the nurse emphasize in this teaching? A.) Give rescue breaths over 3 seconds. B.) Deliver 100 compressions a minute. C.) Support the head before calling for help. D.) Ensure rescue breathing occurs every 10 compressions.

B.) Deliver 100 compressions a minute.

A patient on chemotherapy is using ondansetron (Zofran) for treatment of nausea. The nurse will instruct the patient to watch for which adverse effect of this drug? A.) Dizziness B.) Diarrhea C.) Dry mouth D.) Blurred vision

B.) Diarrhea

A patient who has AIDS has lost weight and is easily fatigued because of his malnourished state. The nurse anticipates an order for which antinausea drug to stimulate his appetite? A.) Metoclopramide (Reglan), a prokinetic drug B.) Dronabinol (Marinol), a tetrahydrocannabinoid C.) Ondansetron (Zofran), a serotonin blocker. D.) Aprepitant (Emend), a substance P/NK1 receptor antagonist

B.) Dronabinol (Marinol), a tetrahydrocannabinoid

The nurse notes that a client's troponin level is elevated. What should this finding indicate to the nurse about the client? A.) Has a high risk for cardiovascular disease B.) Experienced a recent myocardial infarction C.) Had a myocardial infarction several months ago D.) Reduced amount of oxygen carrying capacity in the blood

B.) Experienced a recent myocardial infarction

The nurse is caring for a patient who is experiencing dyspnea. Which position would be the most effective if incorporated into the patient's care? A.)Supine B.) Head of bed elevated 80 degrees C.) Head of bed elevated 30 degrees D.) Lying on left side

B.) Head of bed elevated 80 degrees

A client is coughing and has bilateral rhonchi throughout the lung fields. Which nursing diagnosis is most appropriate for these assessment findings? A.)Impaired Gas Exchange B.) Ineffective Airway Clearance C.) Ineffective Breathing Pattern D.) Impaired Spontaneous Ventilation

B.) Ineffctive Airway Clearance

for new graduate nurses. Which statement should the instructor emphasize being the key The staff development instructor prepares an in-service presentation on full-spectrum nursing point about this delivery approach? A.) It encourages collaboration when planning care. B.) It is a unique blend of thinking, doing, and caring. C.) It focuses on assessment as the cornerstone of care. D.) It relies on client responses to guide interventions.

B.) It is a unique blend of thinking, doing, and caring.

The nurse developed a care plan for a patient to help prevent Impaired Skin Integrity. The the evaluation phase of the nursing process, which would the nurse do first? nurse ensured the nursing assistive personnel changed the patient's position every 2 hours. A. ) Determine whether she has gathered enough assessment data. B.) Judge whether the interventions achieved the stated outcomes. C. ) Follow up to verify that care for the nursing diagnosis was given. D. ) Decide whether the nursing diagnosis was accurate for the patient's condition.

B.) Judge whether the interventions achieved the stated outcomes.

Which is true about a nursing code of ethics? A.) Legally binding B.) Not legally binding C.) Legally binding in some circumstances D.) Not admissible in court

B.) Not legally binding

The family of an alert, oriented, and competent client does not agree with the client's decision to have a do not resuscitate order and asks the healthcare team to ignore the decision. The nurse asks the hospital chaplain to come and help the family and the team understand each other's opposing views. Which step of the MORAL model does this illustrate? A.) M-Massage the dilemma B.) O- Outline the options C.) R - Resolve the dilemma D) L-Look back and evaluate

B.) O- Outline the options

A patient is taking a xanthine derivative disease. The nurse will monitor for which adverse effects associated with the use of xanthine as part of treatment for chronic obstructive pulmonary derivatives? A.) Diarrhea B.) Palpitations C.) Bradycardia D.) Drowsiness

B.) Palpitations

A client with pulmonary hypertension and right sided heart failure has conversational dyspnea and shortness of breath. What is the first action the nurse should take? A.) Review and implement the primary care provider's prescriptions for treatments. B.) Perform a quick physical examination of breathing, circulation, and oxygenation. C.) Gather a thorough medical history, including current symptoms, from the family. D.) Administer oxygen to the patient through a nasal cannula.

B.) Perform a quick physical examination of breathing, circulation, and oxygenation.

When using sterile technique to perform tracheostomy care of a new tracheostomy, which action is correct? A.) Apply sterile gloves B.) Place the patient in semi-Fowler's position, if possible. C.) Clean the stoma under the faceplate with hydrogen peroxide. D.) Cut a slit in sterile 4 x 4 gauze halfway through to make a dressing.

B.) Place the patient in semi-Fowler's position, if possible.

A client with a tracheostomy being mechanically ventilated has a pulse oximetry reading of 85% heart rate of 113 beats/min, and respiratory rate of 30 breaths/min. The client is restless, and crackles and rhonchi are auScultated over both lungs. Which action should the nurse take? A.) Call the respiratory therapist to check the ventilator settings. B.) Provide endotracheal suctioning. C.) Provide tracheostomy care D.) Notify the physician of the patient's signs of fluid overload.

B.) Provide endotracheal suctioning

Which part of the electrocardiogram (ECG) tracing represents ventricular repolarization? A.) P wave B.) QRS complex C.) ) T wave D.) U wave

B.) QRS complex

Which is the third step in the MORAL decision- making model? A.) Reassess the dilemma. B.) Resolve the dilemma. C.) Review the problem. D.)) Recall the history of the problem.

B.) Resolve the dilemma.

The nurse is experiencing moral distress over an ethical issue with a client. What should the nurse do to help reduce this distress? Select all that apply. A) Consider terminating employment. B.) Seek spiritual guidance and support. C.) Make a commitment to care for self. threatened. D.) Analyze personal values that are E.) View the situation from the other perspective.

B.) Seek spiritual guidance and support. C.) Make a commitment to care for self. threatened. D.) Analyze personal values that are E.) View the situation from the other perspective.

A patient is taking chemotherapy with a drug that has a high potential for causing nausea and vomiting. The nurse is preparing to administer an antiemetic drug. Which class of antiemetic drugs is most commonly used to prevent nausea and vomiting for patients receiving chemotherapy? A.) Prokinetic drugs, such as metoclopramide (Reglan) B.) Serotonin blockers, such as ondansetron (Zofran) C.) Anticholinergic drugs, such as scopolamine D.) Neuroleptic drugs, such as promethazine (Phenergan)

B.) Serotonin blockers, such as ondansetron (Zofran)

A client has a heart rate of 112 beats per minute. Which type of dysrhythmia should the nurse suspect this client is experiencing? A.) Brady B.) Tachy C.) Junctional D.) Supraventricular

B.) Tachy

A patient who has been newly diagnosed with vertigo will be taking an antihistamine antiemetic drug. The nurse will include which information when teaching the patient about this drug? A.) The patient may skip doses if the patient is feeling well. B.) The patient will need to avoid driving because of possible drowsiness. C.) The patient may experience occasional problems with taste. D.) It is safe to take the medication with a glass of wine the evening to help settle the stomach.

B.) The patient will need to avoid driving because of possible drowsiness.

What do critical thinking and the nursing process have in common? A.) They are both linear processes used to guide one's thinking. B.) They are both thinking methods used to solve a problem. C. ) They both use specific steps to solve a problem. D. ) They both use similar steps to solve a problem.

B.) They are both thinking methods used to solve a problem.

The nurse is reviewing the uses of oral laxatives. Which conditions are general contraindications to or cautions about the use of oral laxatives? (Select all that apply.) A.) Irritable bowel syndrome B.) Undiagnosed abdominal pain C. Nausea and vomiting D.) Fecal impaction E.) Ingestion of toxic substances F.) Acute surgical abdomen

B.) Undiagnosed abdominal pain C. Nausea and vomiting D.) Fecal impaction F.) Acute surgical abdomen

Which consequentialist theory takes the position that the value of an action is determined by its usefulness? A) Ethics of care B.) Utilitarianism C.) Deontology D.) Categorical imperative

B.) Utilitarianism

A patient has prescriptions for two inhalers. One inhaler is a bronchodilator, and the other is a corticosteroid. Which instruction regarding these inhalers will the nurse give to the patient? A.) "Take the corticosteroid inhaler first." B.)"Take the bronchodilator inhaler first." C."Take these two drugs at least 2 hours apart." D) . "It does not matter which inhaler you use first."

B.)"Take the bronchodilator inhaler first."

The nurse instructs the mother of a toddler on safety. What information about the toddler's developmental stage and factors that influence oxygenation should the nurse include? Select all that apply. A.)Frequent, serious respiratory infections B.)Airway obstruction from aspiration of small objects C.)Drowning in small amounts of water around the home D.)Development of asthma E.)Develop shortness of breath with extreme activity

B.)Airway obstruction from aspiration of small objects C.)Drowning in small amounts of water around the home

For which condition is obesity associated with a higher risk of conditions that affect the pulmonary and cardiovascular systems? Select all that apply. A.)Reduced alveolar- capillary gas exchange B.)Lower respiratory tract infections C.)Sleep apnea D.)Hypertension E.)Dyspnea on exertion

B.)Lower respiratory tract infections C.)Sleep apnea D.)Hypertension E.)Dyspnea on exertion

North American healthcare culture typically reflects which culture? A.Asian B.European American C.Latino D.African American

B.European American

The parent of an 18 - month - old child is concerned because the child's legs are bowed . Which response by the nurse is appropriate ? A ) " Your child will most likely require physical therapy . " B ) " You should consider having your child seen by an orthopedic surgeon . " C ) " This is a normal finding in children for 1 year after they begin walking . " D ) " Your child is walking fine , so you don't need to worry . "

C ) " This is a normal finding in children for 1 year after they begin walking . "

For which range of time must a nurse wash her hands before working in the operating room ? A ) 1 to 2 minutes B ) 2 to 4 minutes C ) 2 to 6 minutes D ) 6 to 10 minutes

C ) 2 to 6 minutes

For which patient can the nurse safely delegate morning care to the nursing assistive personnel ( NAP ) ? A ) 32 - year - old admitted with a closed head injury B ) 76 - year - old admitted with septic shock C ) 62 - year - old who underwent surgical repair of a bowel obstruction 2 days ago D ) 23 - year - old admitted with an exacerbation of asthma with dyspnea on exertion

C ) 62 - year - old who underwent surgical repair of a bowel obstruction 2 days ago

An older adult comes to the clinic complaining of pain in the left foot . While assessing the patient , the nurse notes smooth , shiny skin that contains no hair on the client's lower legs . Which condition does this finding suggest ? A ) Venous insufficiency B ) Hyperthyroidism C ) Arterial insufficiency D ) Dehydration

C ) Arterial insufficiency

The nurse is teaching a group of community members about food safety . What should be included in this teaching ? A) Store deli meat for 1 week . B ) Cook eggs until the white is firm . C ) Avoid changing a diaper while cooking . D ) Roast meat at an oven temperature of 250 ° F or above .

C ) Avoid changing a diaper while cooking .

A patient is prescribed fluoxetine 20 mg by mouth daily for treatment of depression . The nurse caring for the patient is unfamiliar with this drug . Which action should the nurse take before administering the medication ? A ) Inform the prescriber that she is not comfortable administering the drug . B ) Ask a nursing colleague for relevant information about the drug . C ) Consult the drug formulary accessible to staff at the patient care unit . D ) Trust the prescriber who writes the dose and administer the drug as intended .

C ) Consult the drug formulary accessible to staff at the patient care unit .

A patient admitted to the hospital with pneumonia has been receiving antibiotics for 2 days . Currently the patient has a normal temperature . Which stage of infection is the patient most likely experiencing ? A ) Incubation B ) Prodromal C ) Decline D ) Convalescence

C ) Decline

The nurse reviews the results of a client's recent wound culture to determine the microorganism causing the infection . Why is the nurse's action important ? A ) Ensures supplies needed for care are available B ) Helps determine the amount of nursing care required C ) Determines whether the microorganism is resistant to antibiotics D ) Estimates the amount of time the client will be hospitalized

C ) Determines whether the microorganism is resistant to antibiotics

Bronchovesicular breath sounds are best heard over which area ? A ) Midline over the trachea just below the larynx B ) Fourth intercostal space , in the midclavicular line C ) First and second intercostal spaces next to the sternum D ) At the base of the lungs near the diaphragm

C ) First and second intercostal spaces next to the sternum

What is the most frequent cause of the spread of infection among institutionalized patients ? A ) Airborne microbes from other patients B ) Contact with contaminated equipment C ) Hands of healthcare workers D ) Exposure from family members

C ) Hands of healthcare workers

A patient's jugular venous pressure measures 5 cm What should this finding indicate t the nurse ? A ) A normal finding B ) Hypovolemia C ) Heart failure D ) Dehydration

C ) Heart failure

A female patient has excessive facial hair . How should the nurse document this finding ? A ) Alopecia B ) Albinism C ) Hirsutism D ) Lanugo

C ) Hirsutism

A patient develops localized heat and erythema over an area on the lower leg . These findings are indicative of which secondary defense against infection ? A ) Phagocytosis B ) Complement cascade . C ) Inflammation S ) Immunity

C ) Inflammation

Why is a lotion without petroleum preferred over a petroleum - based product as a skin protectant ? A ) It prevents microorganisms from adhering to the skin . B ) It facilitates the absorption of latex proteins through the skin . C ) It decreases the risk of latex allergies . D ) It prevents the skin from drying and chaffing .

C ) It decreases the risk of latex allergies .

While bathing a patient with liver dysfunction , the nurse notes yellow skin tone . The nurse should document this finding as : A ) Pallor B ) Erythema C ) Jaundice D ) Cyanosis

C ) Jaundice

The nurse prepares to remove the artificial eye of a patient sitting at the bedside . In which position should the patient be placed to facilitate the removal of this device ? Ask the patient to : A ) Lean forward and rest the arms on the overbed table B ) Sit back in the chair and tilt the head back C ) Move to the bed and lie down D ) Stand up and lean over the bed

C ) Move to the bed and lie down

The nurse assesses a 4 - year - old child's vision as 20/40 . What should the nurse realize this finding indicates ? A ) Myopia B ) Hyperopia C ) Normal D ) Presbyopia

C ) Normal

The nurse notes a small pulsation at the fifth intercostal space midclavicular line . How should the nurse document this finding ? A ) Thrill B ) Murmur C ) Normal finding D ) Heave

C ) Normal finding

What term is used to describe the time it takes for drug concentration to reach a therapeutic level in the blood ? A ) Peak action . B ) Duration of action C ) Onset of action D ) Half - life

C ) Onset of action

A patient with tuberculosis is scheduled for computed tomography ( CT ) . How should the nurse proceed ? Select all that apply . A ) Question the order because the patient must remain in isolation . B ) Place an N - 95 respirator mask on the patient and transport him to the test . C ) Place a surgical mask on the patient and transport him to CT lab . D ) Notify the CT department about precautions prior to transport . E ) Notify housekeeping to clean the CT room before the patient arrives .

C ) Place a surgical mask on the patient and transport him to CT lab . D ) Notify the CT department about precautions prior to transport .

A 48 - year - old patient comes to the physician's office complaining of diminished near vision , which the nurse confirms with testing . How should the nurse document this finding ? A ) Myopia B ) Diplopia C ) Presbyopia D ) Mydriasis

C ) Presbyopia

The nurse is caring for a patient admitted with a closed head injury . Which action by the nurse is appropriate when providing hygiene for this patient ? A ) Avoid bathing the patient . B ) Use cool water for bathing . C ) Provide care in small intervals . D ) Rub briskly when towel drying .

C ) Provide care in small intervals .

A patient in the emergency department is angry , yelling , cursing , and waving his arms when the nurse comes to the treatment cubicle . Which action should the nurse take ? A ) Reassure the patient by entering the room alone . B ) Ask the patient if he is carrying any weapons . C ) Stay between the patient and the door ; keep the door open . D ) Make eye contact while stating firmly , " I will not tolerate cursing and threats . "

C ) Stay between the patient and the door ; keep the door open .

While receiving an intravenous dose of an antibiotic , levofloxacin , a patient develops severe shortness of breath , wheezing , and severe hypotension . Which action should the nurse take first ? A ) Administer epinephrine IM . B ) Give bolus dose of intravenous fluids . C ) Stop the infusion of medication . D ) Prepare for endotracheal intubation .

C ) Stop the infusion of medication .

A patient admitted with an acute exacerbation of chronic obstructive pulmonary disease has a nursing diagnosis of Activity Intolerance . Which type of bath is preferred for this patient ? A ) Tub bath B ) Complete bed bath C ) Towel bath D) Bed bath

C ) Towel bath

The nurse notes that a community member is unkempt and has body odor . What should the nurse consider as impacting this client's hygienic practices ? Select all that apply . A ) Illness B ) Lack of desire C ) Water in the home D ) Money to buy hygiene supplies E ) Lack of knowledge about bathing

C ) Water in the home D ) Money to buy hygiene supplies

The nurse gathers the following data: BP =150/94 mm Hg; neck veins distended; P = 104 beats/min; pulse bounding; respiratory rate = 20 breaths/min; T = 37°C (98.6°F). What disorder should the nurse suspect? A) Hypokalemia B) Hypercalcemia C Hyperkalemia D) Hypervolemia

C Hypervolemia

The nurse is teaching a clinic patient about hypertension. Which statement indicates the patient is present oriented? A) "I know I need to lose weight; I'll have to begin an exercise program right away." B) "If I change my diet and begin exercising, maybe I can control my blood pressure without medications." C) "I know I need to give up foods that contain a lot of salt, but with teenagers in the house it is very difficult. D) "I will reduce the amount of calories, salt, and fat that I eat; I certainly do not want to have a stroke."

C) "I know I need to give up foods that contain a lot of salt, but with teenagers in the house it is very difficult.

Which statement by the nurse is best when communicating with a patient with clinical depression? A) "It's a beautiful day today; you'll feel better if you look out the window." B) "You're having a bad day: I'm sure you'll feel better soon." C) "Life seems overwhelming at times; would you like to discuss how you're feeling?" D) "You are very lucky to have such a supportive family."

C) "Life seems overwhelming at times; would you like to discuss how you're feeling?"

When counseling a patient about behaviors to reduce stress, which goal should the nurse put on the care plan? A) "The patient will limit his intake of fat to no more than 15% of the daily calories consumed." B) "The patient will eat three meals per day at approximately the same time each day." C) "The patient will limit his intake of sugar and salt, as well as sweet and salty foods." D) "The patient will consume no more than three alcoholic beverages a day."

C) "The patient will limit his intake of sugar and salt, as well as sweet and salty foods."

A patient sues the nurse for a fall that occurred because the bed's siderails were not in the upright position. The judgment is for the patient to receive $2 million; however, the nurse has an occurrence policy with double limit coverage of $3 million/$10 million that covered the time period when the incident occurred. What will the nurse's insurance pay to the patient? A) Nothing B) $4 million C) $2 million D) 75% of the $2 million

C) $2 million

How many diapers should a healthy newborn use for urine output each day? 1 A) 4 B) 6 C) 10 D) 15

C) 10

A client' s vital signs at the beginning of the shift are oral temperature 99.3°F (37°C), heart rate 82 beats/min, respiratory rate 14 breaths/min, and blood pressure 118/76 mm Hg. Four hours later the client's oral temperature is 102.2°F (39°C). Based on the temperature change, the nurse should anticipate the client's heart rate would be how many beats/min? A) 62 B) 82 C) 102 D) 122

C) 102

At 1000 on 11/14/17, the nurse receives a telephone order for metoprolol 5 mg intravenously now. What is latest date and time the nurse will expect prescriber to countersign the order? A) 11/14/17 at 1200 B) 11/14/17 at 2200 C) 11/15/17 at 1000 D) 11/16/17 at 1000

C) 11/15/17 at 1000

Which blood pressure has a pulse pressure within normal limits? Select all that apply. A) 104/50 mm Hg B) 120/62 mm Hg C) 120/80 mm Hg D) 130/86 mm Hg E) 140/98 mm Hg

C) 120/80 mm Hg D) 130/86 mm Hg

A healthcare provider prescribes 250 ml of 0.9% sodium chloride to be infused over 2 hours. A microdrip infusion set is being used. What is the drip rate (drops/min) that the nurse should monitor? A) 60 B) 75 C) 125 D) 250

C) 125

A nurse is teaching wellness to a women's group. The nurse should explain the importance of consuming at least how many 8-ounce servings of fluid to promote healthy bowel function? A) 2 to 3 B) 4 to 5 C) 6 to 8 D) 9 to 10

C) 6 to 8

A patient is taking linaclotide (Linzess) to treat irritable bowel syndrome (IBS). The nurse will monitor this patient for which adverse effect? A.) Chest pain B.) Chronic constipation C) Abdominal pain D.) Elevated blood glucose levels

C) Abdominal pain

A patient from Italy has lived in the United States for years and practices elements from both cultures. What is this patient demonstrating? A. Assimilation B.Socialization C) Acculturation D) Immigration

C) Acculturation

A patient's digoxin level is 1.2 ng/mL; the therapeutic range for this drug is 0.5 to 2.0 ng/mL. Which action should the nurse take? A) Notify the prescriber to reduce the dose. B) Withhold the next dose of digoxin. C) Administer the next dose as prescribed. D) Notify the prescribing healthcare provider to increase the dose.

C) Administer the next dose as prescribed.

Which is an example of an illness-prevention activity? A) Encouraging the use of a food diary B) Joining a cancer support group C) Administering immunization for HPV D) Teaching a diabetic patient about his diet

C) Administering immunization for HPV

Which individual is most likely to have a positive body image? A) Child who has been deaf since birth B) Child who was born with cystic fibrosis C) Adolescent of average appearance who had an appendectomy D) Adult born with a spinal defect and associated paralysis of the lower body

C) Adolescent of average appearance who had an appendectomy

The nurse manager receives a note from a staff member's healthcare provider outlining physical limitations at work. Which information should the manager review to ensure the employee's and organization's rights are not violated? A) Nursing scope of practice B) Policy and procedure manual C) Americans With Disabilities Act (ADA) D) Health Insurance Portability and Accountability Act (HIPAA)

C) Americans With Disabilities Act (ADA)

The nurse prepares teaching material for a client scheduled for an ileostomy. What information is essential to include when teaching this client? A) It is usually temporary. B) Irigation can control bowel movements. C) An ostomy device must be wom at all times. D) Changing the diet can control bowel movements.

C) An ostomy device must be wom at all times.

Which is a key treatment intervention for the patient admitted with diverticulitis? A) Antacid B) Antidiarrheal agent C) Antibiotic therapy D) NSAIDS

C) Antibiotic therapy

A patient complains of a vague, uneasy feeling of dread, and has an elevated heart rate. Which nursing diagnosis is most appropriate for this patient? A) Anger B) Fear C) Anxiety D) Hopelessness

C) Anxiety

A patient had surgery 6 hours ago. When the nurse enters the room to turn him, she notes that he is restless and grimacing. Considering the patient's nonverbal communication, what action should the nurse take first? A) Administer pain medication to the patient. B) Turn and reposition the patient. C) Assess to determine the cause of the grimacing. D) Leave the patient's room so he can rest quietly.

C) Assess to determine the cause of the grimacing.

The nurse updates a client's database prior to attending an interdisciplinary care meeting. Which best explains why the nurse performed this action? A) Minimizes repetition between the disciplines B) Reduces the amount of work that another person needs to do C) Assists when deciding the best course of treatment for the client D) Ensures that the nurse remains as the leader of the care meeting

C) Assists when deciding the best course of treatment for the client

A client asks for suggestions to improve a current state of health. What should the nurse include when creating goals for this client's health status? A) Actions to manage chronic health problems B) Strategies to improve health over the life span C) Behaviors that demonstrate health maintenance D) Methods to meet an identified timeline for goal achievement

C) Behaviors that demonstrate health maintenance

Which is considered a strength of the nursing profession? A) Biomedical focus B) Psychosocial focus C) Biopsychosocial focus D) Physical focus

C) Biopsychosocial focus

The nurse is teaching an older female patient how to manage urge incontinence at home. What is the first-line approach to reducing involuntary leakage of urine? A) Insertion of a pessary B) Intermittent self-catheterization C) Bladder training D) Anticholinergic medication

C) Bladder training

Comparing the changes in vital signs as a person ages, which statement is correct? Select all that apply. A) Blood pressure decreases less than heart rate and respiratory rate. B) Respiratory rate remains fairly stable throughout a person's life. C) Blood pressure increases; heart rate and respiratory rate decline. D) Men have higher blood pressure than women until after menopause. E) Body temperature increases with aging.

C) Blood pressure increases; heart rate and respiratory rate decline. D) Men have higher blood pressure than women until after menopause.

A patient has a continuous IV infusion at 60 mL/hr. The right hand IV has infiltrated and the nurse has started a new IV on the left forearm. Which intervention should the nurse also perform? A) Elevate the patient's left forearm, B) Schedule daily dressing changes to the new IV site. C) Change the administration set. D) Place the patient in Fowler's position.

C) Change the administration set.

A patient who has been immobile since sustaining injuries in a motor vehicle crash complains of constipation. The nurse encourages him to consume eight to ten 8-ounce servings of fluid daily. Which fluid should the patient avoid because of the diuretic effect? Select all that apply. A) Cranberry juice B) Water C) Coffee D) Ginger ale E) Tea

C) Coffee E) Tea

A patient with a skin infection is prescribed cephalexin (an antibiotic) 500 mg orally q 12 hours. The patient complains that the last time he took this medication he had frequent episodes of loose stools. Which recommendation should the nurse make to the patient? A) Stop taking the drug immediately if diarrhea develops. B) Take an antidiarrheal agent, such as diphenoxylate. C) Consume yogurt daily while taking the antibiotic. D) Increase your intake of fiber until the diarrhea stops.

C) Consume yogurt daily while taking the antibiotic.

A patient has been vomiting for 2 days, has not been able to cat or drink anything during this time, and has not urinated for 12 hours. Physical examination reveals the following: T= 99.6°F (37.6°C) orally; P-110 beats/min weak and thready; BP = 80/52 mm Hg. Skin and mucous membranes are dry, there is decreased skin turgor, and the patient is experiencing weakness. The following are the most recent laboratory results: Sodium 138 mEq'L Potassium 3.7 mEq/L Calcium 9.2 mg/dL Magnesium 1.8 mg/dL Chloride 99 mEq/L BUN 29 mg/dL Which is an appropriate nursing diagnosis for this patient? A) Impaired Gas Exchange related to ineffective breathing B) Excess Fluid Volume related to limited fluid output C) Deficient Fluid Volume related to abnormal fluid loss D) Electrolyte Imbalance related to decreased oral intake

C) Deficient Fluid Volume related to abnormal fluid loss

A patient is admitted to the emergency department with a stroke. After the patient has been stabilized, which information should the nurse include in the care plan that best meets the patient's needs? A) Acute interventions B) Patient teaching C) Discharge needs D) Family health data

C) Discharge needs

What position should the patient assume before the nurse inserts an indwelling urinary catheter? A) Modified Trendelenburg B) Prone C) Dorsal recumbent D) Semi-Fowler's

C) Dorsal recumbent

The nurse makes a mistake while documenting in the patient's health record. Which action should the nurse take? A) Use an opaque white fluid to cover the documentation error. B) Completely cover the documentation error with black ink. C) Draw a line through the error and initial the change. D) Use correction tape to make the documentation correct.

C) Draw a line through the error and initial the change.

What intervention would be most appropriate for a wound with a beefy red wound bed? A) Mechanical débridement B) Autolytic débridement C) Dressing to keep the wound moist and clean D) Removal of devitalized tissue and a sterile dressing

C) Dressing to keep the wound moist and clean

The nurse identifies the nursing diagnosis Diarrhea related to stress for a patient. Which nursing intervention should be included in the nursing care plan to help the patient relieve the cause of the diarrhea? A) Monitor and record the frequency of stools on the graphic record. B) Administer prescribed antidiarrheal medications as needed. C) Encourage the patient to verbalize about stressors and anxiety. D) Provide oral fluids on a regular schedule.

C) Encourage the patient to verbalize about stressors and anxiety.

When released in response to alarm, which substance promotes a sense of well-being? A) Aldosterone B) Thyroid-stimulating hormone C) Endorphins D) Adrenocorticotropic hormone

C) Endorphins

A young adult male client of Mexican heritage sustains injuries from a motor vehicle crash. When assessing this patient, the nurse must consider that the client may possess which view of pain? A.A belief in taboos against narcotic use to relieve pain B. Expectation of immediate treatment for relief of pain C. Endurance of pain longer and report it less frequently than some patients do D. Use of herbal teas, heat application, and prayers to manage his pain

C) Endurance of pain longer and report it less frequently than some patients do

An adolescent in labor arrives at the hospital emergency department (ED) with her mother who states the family has no medical insurance or money to pay for the delivery. What is the first step that the ED staff should take? A.) Arrange for an ambulance to transport her to the nearest public hospital. B) Explain to the girl and her mother that the hospital only accepts patients who can pay the hospital bill. C) Examine her to determine whether her condition is stable or whether she requires immediate medical attention. D) Inform her mother that she will need to transport her daughter to the nearest public hospital.

C) Examine her to determine whether her condition is stable or whether she requires immediate medical attention.

After sustaining injuries in a motor vehicle accident, a patient experiences a decrease in blood pressure and an increase in heart rate and respiratory rate despite surgical intervention and fluid resuscitation. Which stage of the general adaptation syndrome is the patient most likely experiencing? A) Alarm B) Resistance C) Exhaustion D) Recovery

C) Exhaustion

The nurse develops a plan of care for a mother of three small children who has been admitted with a serious acute illness, which is likely to continue long term. The nurse writes the following intervention: "Facilitate communication between patient and significant other regarding the sharing of responsibilities to accommodate changes brought on by illness." What is the purpose of this intervention? A) Promote self-esteem B) Promote positive body image C) Facilitate role enhancement D) Prevent depersonalization

C) Facilitate role enhancement

A client has a history of urinary retention. Which over-the-counter medication should the nurse instruct the client to avoid taking? Select all that apply. A) Ibuprofen B) Ranitidine C) Fexofenadine D) Diphenhydramine E) Acetaminophen

C) Fexofenadine D) Diphenhydramine

Bronchovesicular breath sounds are best heard over which area? A) Midline over the trachea just below the larynx B) Fourth intercostal space, in the midclavicular line C) First and second intercostal spaces next to the sternum D) At the base of the lungs near the diaphragm

C) First and second intercostal spaces next to the sternum

Which food item should the nurse instruct the patient to consume to prevent or treat constipation? A) Milk and cheese B) Bread and pasta C) Fruits and vegetables D) Lean meats

C) Fruits and vegetables

blood gases (ABGS) results: pH repeat ABGS results are pH 7.30; PCO2:- 40; HCO3 7.38; PCO2: = 32; acidosis; the treatment plan is ineffective. HCO3: = 19 mEg/L. What should = 19 mEq/L; PO2 = 80. After treatment, A patient in respiratory distress has the following arterial the nurse conclude from this information? A) Respiratory B) Metabolic alkalosis; the treatment plan is effective. C) Full compensation; the treatment plan is effective. D) Partial compensation; the treatment plan is ineffective.

C) Full compensation; the treatment plan is effective.

A patient who underwent surgery 24 hours ago is prescribed a clear liquid diet. The patient asks for something drink. Which item may the nurse provide for the patient? A) Tea with cream B) Orange juice C) Gelatin D) Skim milk

C) Gelatin

Which polysaccharide is stored in the liver? A) Insulin B) Ketones C) Glycogen D) Glucose

C) Glycogen

Which is the best approach to validate a clinical inference? A) Have another nurse evaluate it. B) Have the physician evaluate it. C) Have sufficient supportive data. D) Have client's family family confirm it.

C) Have sufficient supportive data.

A patient argues that a statement made in the medical record is incorrect and wants it corrected. Which regulation provides the patient with the right to have the documentation corrected? A) Americans With Disabilities Act (ADA) B) Patient Self-Determination Act (PSDA) C) Health Insurance Portability and Accountability Act (HIPAA) D) Health Care Quality Improvement Act (HCQIA)

C) Health Insurance Portability and Accountability Act (HIPAA)

A patient's jugular venous pressure measures 5 cm. What should this finding indicate to the nurse? A) A normal finding B) Hypovolemia C) Heart failure D) Dehydration

C) Heart failure

Nursing was described as a distinct occupation in the sacred books of which faith? A) Buddhism B) Christianity C) Hinduism D) Judaism

C) Hinduism

A patient has a stage 2 pressure injury that is covered with dry, yellow slough that tightly adheres to the wound. What is the best treatment the nurse could recommend for treating this wound? A) Dry gauze dressing changed twice daily B) Nonadherent dressing with daily wound care C) Hydrocolloid dressing changed as needed D) Wet-to-dry dressings changed three times a day

C) Hydrocolloid dressing changed as needed

The nurse examines the electrocardiogram (ECG) tracing of a client and notes tall T waves, What electrolyte imbalance should the nurse suspect? A) Hypokalemia B) Hypophosphatemia C) Hyperkalemia D) Hypercalcemia

C) Hyperkalemia

A patient's 2:1 parenteral nutrition (PN) container infuses before the pharmacy prepares the next container. This places the patient at risk for which complication? A) Sepsis B) Pneumothorax C) Hypoglycemia D) Thrombophlebitis

C) Hypoglycemia

The nurse provides client education regarding hypertension prevention and management. Which statement indicates that the client understands the instructions? A) I don't have to worry if my blood pressure is high once in a while. B) I guess I will have to make sure I don't drink too much water. C) I can lose some weight to help lower my blood pressure D) I will need to reduce the amount of milk and other dairy products I consume

C) I can lose some weight to help lower my blood pressure

Which is an example of self-knowledge if thought or stated by the nurse? A) I know that I should take the client's apical pulse for 1 minute before giving digoxin. B) I know that I should follow the client's wishes even though it is not what I would want. C) I know that I have religious beliefs that may make it difficult to take care of some clients. D) I know that I need to honor the client's request not to discuss his health concern with the family.

C) I know that I have religious beliefs that may make it difficult to take care of some clients.

Which statement or question by the nurse indicates that the nursepatient relationship is entering the termination phase? A) I'll be admitting you to our nursing unit as soon as I obtain your health history B) You seem upset today. Would you like to talk about whatever is bothering you C) I'm leaving for the day. Is there anything I can do for you before I leave D) Hello My name is Leslie, and I'm going to be your nurse today

C) I'm leaving for the day. Is there anything I can do for you before I leave

The nurse includes functional ability when assessing a client. What is the nurse's purpose in collecting this information? A) Estimates the cost for healthcare B) Evaluates the success of interventions C) Identifies future rehabilitation and palliative care needs D) Reduces the amount of repetition being provided by care providers

C) Identifies future rehabilitation and palliative care needs

During the day shift, a patient's temperature measures 97°F (36.1°C) orally. At 2000, the patient's temperature measures 102°F (38.9°C). What effect does this rise in temperature have on the patient's basal metabolic rate? A) Increases the rate by 7% B) Decreases the rate by 14% C) Increases the rate by 35% D) Decreases the rate by 28%

C) Increases the rate by 35%

A patient has a colostomy in the descending (sigmoid) colon and wants to control bowel evacuation and possibly stop wearing an ostomy pouch. What should the nurse teach so that this patient can achieve this goal? A) Call the primary care provider if the stoma becomes pale, dusky, or black. B) Limit the intake of gas-forming foods such as cabbage, onions, and fish. C) Irrigate the stoma to produce a bowel movement on a schedule. D) Avoid returming to the use of an ostomy appliance if he becomes ill.

C) Irrigate the stoma to produce a bowel movement on a schedule.

A patient with type 1 diabetes mellitus is admitted with hyperglycemia and associated acidosis. Which alternative fuel in the body is responsible for the acidosis? A) Glycogen B) Insulin C) Ketones D) Proteins

C) Ketones

Which phrase is the best example le of an outcome? A) Use the incentive spirometer when awake B) Walk two times during day and evening shifts C) Maintain oxygen saturation above 92% while performing ADLs each morning D) Tolerate 10 sets of range-of-motion exercises with physical therapy

C) Maintain oxygen saturation above 92% while performing ADLs each morning

A client is admitted to the emergency department (ED) in respiratory distress. The results of his arterial blood gases are pH = 7.30; PCO2-40; HCO3 = 19 mEq/L; PO, = 80. How should the nurse interpret these results? A) Respiratory acidosis with normal oxygen levels B) Respiratory alkalosis with hypoxia C) Metabolic acidosis with normal oxygen levels D) Metabolic alkalosis with hypoxia

C) Metabolic acidosis with normal oxygen levels

Which is the most influential factor that has shaped the nursing profession? A) Physicians' need for handmaidens B) Societal need for healthcare outside the home C) Military demand for nurses in the field D) Germ theory influence on sanitation

C) Military demand for nurses in the field

The nurse plans care for a client who is bedridden. Which laxative should be avoided to treat constipation in this client? A) Osmotic B) Stimulant C) Mineral oil D) Stool softener

C) Mineral oil

The mother of a school- age child is distressed because the child does not seem to "value" anything. What should the nurse recommend to this mother? A) Consider moralizing. B) Implement punishment. C) Model the desired behavior. D) Use a laissez- faire approach.

C) Model the desired behavior.

35. A client is being treated for depression. What should the nurse recall before completing an assessment of this client? Select all that apply. A) Depression rarely recurs. B) Everyone likes to talk about feelings. C) More women are depressed than men. D) Depression is associated with spiritual distress. E) Medication should be prescribed for depression.

C) More women are depressed than men. D) Depression is associated with spiritual distress.

The patient's health record contains the following provider's order: furosemide 40 mg intravenously STAT. Where should the nurse look to determine whether the medication was provided and the patient's response? A) Progress notes B) Graphic record C) Narrative notes D) MAR

C) Narrative notes

The charge nurse assigns the care of a patient receiving hemodialysis to a newly hired licensed practical nurse (LPN) who has no experience caring for hemodialysis patients. The LPN fails to inform the charge nurse of not having experience with this type of patient. What should the actions of the charge nurse be considered? A) Malpractice B) Incompetence C) Negligence D) Abandonment

C) Negligence

21. The nurse assesses a 4-vear-old child's vision as 20/40, What should the purse realize this finding indicates? A) Myopia B) Нуperopia C) Normal D) Presbyopia

C) Normal

The nurse notes a small pulsation at the fifth intercostal space midelavicular line. How should nurse document this finding? A) Thrill B) Murmur C) Normal finding D) Heave

C) Normal finding

The nurse receives a document from an attorney that states a former client is suing for injuries obtained when receiving care, What should the nurse do first? A) Call a lawyer. B) Contact the client. C) Notify the organization. D) Contact the insurance company.

C) Notify the organization.

A patient recovering from a bowel resection a few hours has urine output of 50 mL/2 hr. Which action should the nurse take? A) Do nothing; this is normal postoperative urine output. B) Increase the infusion rate of the patient's IV fluids. C) Notify the provider about the patient's oliguria. D) Administer the patient's routine diuretic dose early.

C) Notify the provider about the patient's oliguria.

During an assessment of a newly admitted client the nurse measures blood pressure, abdominal circumference, and pulse rate. Which critical-thinking skill is the nurse using? A) Recognizing gaps in one's own knowledge B) Recognizing the need for more information C) Objectively gathering information on a problem or issue D) Evaluating the credibility and usefulness of sources of information

C) Objectively gathering information on a problem or issue

A client's axillary temperature is 100.8°F. The nurse realizes this is outside normal range for this client and that axillary temperatures do not reflect core temperature. What should the nurse do to obtain a good estimate of the core temperature? A) Add 1°F to 100.8°F to obtain an oral equivalent. B) Add 2°F to 100.8°F to obtain a rectal equivalent. C) Obtain a rectal temperature reading. D) Obtain a tympanic membrane reading.

C) Obtain a rectal temperature reading.

Where should the nurse assess skin color changes in the dark-skinned patient? A) Elbow B) Any exposed area C) Oral mucosa D) Behind the knee

C) Oral mucosa

Which statement best describes self-concept? A) Understanding of other's perception B) Evaluation of self C) Overall view of self D) Perspective of role in society

C) Overall view of self

The nurse is instructing a client how to appropriately dress an infant in cold weather. Which instruction would be most important for the nurse to include? A) Be sure to put mittens on the baby B) Layers the infant's clothing C) Place a cap on the infant's head D) put warm booties on the baby

C) Place a cap on the infant's head

A 48-year-old patient comes to the physician's office complaining of diminished near vision, which the nurse confirms with testing. How should the nurse document this finding? A) Myopia B) Diplopia C) Presbyopia D) Mydriasis

C) Presbyopia

What is the function of the stratum corneum? A) Provides insulation for temperature regulation B) Provides strength and elasticity to the skin C) Protects the body against the entry of pathogens D) Continually produces new skin cells

C) Protects the body against the entry of pathogens

Which task can the nurse safely delegate to the nursing assistive personnel? 1 A) Palpating the bladder of a patient who is unable to void B) Administering a continuous bladder irrigation C) Providing indwelling urinary catheter care D) Obtaining the patient's history and physical assessment

C) Providing indwelling urinary catheter care

The nurse notes that a patient's indwelling urinary catheter tubing contains sediment and crusting is present at the meatus. Which action should the nurse take? A) Notify the provider immediately. B) Flush the catheter tubing with saline solution. C) Replace the indwelling urinary catheter. D) Encourage fluids that increase urine acidity.

C) Replace the indwelling urinary catheter.

A patient who speaks only French was admitted to the hospital after a motor vehicle accident. Assuming that the nurse does not speak French, what is the best way to communicate with this patient? A) Use sign language for communicating. B) Ask a family member to serve as an interpreter. C) Request the services of a hospital interpreter. D) Speak in English, but speak very slowly.

C) Request the services of a hospital interpreter.

Which nutritional goal is appropriate for a patient newly diagnosed with hypertension? A) Limit his intake of protein. B) Avoid foods containing gluten. C) Restrict his use of sodium. D) Limit his intake of potassium-rich foods.

C) Restrict his use of sodium.

The nurse is caring for a patient with a medical diagnosis of hypernatremia, The following prescriptions are written in the client's electronic health record. Which one should the nurse question? A) Administer an IV of DsW at 125 mL/hr. B) Strict I&O monitoring. C) Restrict oral intake to 900 mL every 24 hr. D) Monitor serum electrolytes every 4 hr.

C) Restrict oral intake to 900 mL every 24 hr.

A patient comes to the emergency department complaining of headache, palpitations, nausea, and dizziness. After determining that the patient is anxious, the nurse notes tachycardia and trembling. Which level of anxiety is this patient exhibiting? A) Mild anxiety B) Moderate anxiety C) Severe anxiety D) Panic anxiety

C) Severe anxiety

An older client asks for another nurse because of not wanting a to provide her with care. Which cultural barrier is this client exhibiting? A) Ethnocentrism B) Racism C) Sexism D) Chauvinism

C) Sexism

Which set of topics makes up a hand-off report given in a recommended format? A) Data-action-response B) Subjective-objective-assessment-plan C) Situation-background-assessment-recommendation D) Patient-diagnosis-medications-activity

C) Situation-background-assessment-recommendation

Which electrolyte is the primary regulator of fluid volume? A) Potassium B) Calcium C) Sodium D) Magnesium

C) Sodium

The nurse suspects that a patient's intravenous solution has infiltrated into the tissues. What action ould the nurse take first? A) Aspirate, then inject 0.5 mL normal saline. B) Restart the IV line in a different vein. C) Stop the infusion immediately. D) Notify the primary care provider.

C) Stop the infusion immediately.

While addressing a community group, the nurse explains the importance of replacing saturated fats in the diet with mono- and polyunsaturated fats. Which complication can be reduced if this action is taken? A) Kidney failure B) Liver failure C) Stroke D) Lung cancer

C) Stroke

A client reports difficulty with smoking cessation efforts. Which direction should the nurse provide to ensure for this client's nutritional status? A) Avoid all intake of vitamin E. B) Reduce the intake of vitamin B each day. C) Take a vitamin C supplement of 2,000 mg/day. D) Increase the intake of vitamin D to 2,000 U per day.

C) Take a vitamin C supplement of 2,000 mg/day.

The adult daughter of an older client with an inoperable brain tumor asks the nurse to tell her what is wrong with the client even though the client has specifically said that no one is to know the diagnosis. What should the nurse say to the adult daughter? A) The client has an inoperable brain tumor, but does not wish anyone to know. B) The daughter needs to speak to the physician in charge of the client's care. C) The client has requested that the case not be discussed with anyone, not even family. D.) The client is very sick with a serious case of pneumonia that could lead to death.

C) The client has requested that the case not be discussed with anyone, not even family.

A patient asks the nurse to explain an advance directive. What should the nurse explain about this document? Select all that apply. A) If the patient is unable to communicate, the family may make changes to the advance directive. B) Once an advance directive is signed, no further care will be provided. C) The patient may change the advance directive by telling the physician or by making changes in writing. D) An advance directive will ensure the patient gets as much or as little desired. E) An advance directive is overruled by the healthcare provider's prescriptions

C) The patient may change the advance directive by telling the physician or by making changes in writing. D) An advance directive will ensure the patient gets as much or as little desired.

When teaching a patient about the healing process of an open wound after surgery, which point should the nurse make? A) The patient will need to take antibiotics until the wound is completely healed. B) Because the patient's wound was left open, the wound will likely become infected. C) The patient will have more scar tissue formation than for a wound closed at surgery. D) The patient should expect to remain hospitalized until complete wound healing occurs.

C) The patient will have more scar tissue formation than for a wound closed at surgery.

Which most accurately describes nursing diagnoses? A) They support the nurse's diagnostic reasoning. B) They support the client's medical diagnosis. C) They identify a client's response to a health problem. D) They identify a client's health problem.

C) They identify a client's response to a health problem.

A patient has been diagnosed with hypovolemia. Which hydration prescription should the nurse question? Select all that apply A) 0.9% (normal) saline at 100 mL/hr B) Lactated Ringer's solution at 100 mL/hr C) Total parenteral nutrition solution at 100 mL/hr D) D5W solution at 100 mL/hr E) D5 0.9% NaCl at 100 mLhr

C) Total parenteral nutrition solution at 100 mL/hr D) D5W solution at 100 mL/hr E) D5 0.9% NaCl at 100 mLhr

The nurse prepares information to document a client's care. What should the nurse do to ensure that the documentation is complete? A) Review the medical record for new prescriptions. B) Ask nursing assistive personnel for report on delegated tasks. C) Use the mnemonic F-A-C-T-U-A-L as a reminder when charting. D) Complete all documentation immediately before hand-off communication.

C) Use the mnemonic F-A-C-T-U-A-L as a reminder when charting.

The nurse is instructing a patient about performing home testing for fecal occult blood. The nurse should explain that ingestion of which substance may cause a false-negative fecal occult blood test? A) Vitamin D B) Iron C) Vitamin C D) Thiamine

C) Vitamin C

Which is an applicable goal/outcome for the client with a stage 4 pressure injury? A) Client will maintain intact skin throughout hospitalization. B) Client will limit pressure to wound site throughout treatment course. C) Wound will close with no evidence of infection within 6 weeks. D) Wound will improve prior to discharge as evidenced by a decrease in drainage.

C) Wound will close with no evidence of infection within 6 weeks.

A client who has been hospitalized for an infection states " The nursing assistant told me my vital signs are all within normal limits; that means I'm cured. What would be the nurse's best response? A) Your vital signs confirm that your infection is resolved; how do you feel ? B) I'll let your healthcare provider know so you can be discharged. C) Your vital signs are stable, but there are other things to assess. D) We still need to keep monitoring your temperature for a while.

C) Your vital signs are stable, but there are other things to assess.

When performing a spiritual assessment, who is the preferred source of information? A.Durable power of attorney B. Next of kin C. Patient D. Patient's clergyman

C. Patient

An older client asks for another nurse because of not wanting a "man" to provide her with care. Which cultural barrier is this client exhibiting? A. Ethnocentrism B.Racism C. Sexism D. Chauvinism

C. Sexism

Which is a nonverbal behavior that enhances communication? A) Keeping a neutral expression on the face B) Maintaining a distance of 6 to 12 inches C) Sitting down to speak with the patient D) Asking mostly open-ended questions

C. Sitting down to speak with the patient

A patient who has been hospitalized for 2 weeks has developed a pressure ulcer that contains multidrug-resistant Staphylococcus aureus (MRSA). Which drug would the nurse expect to be chosen for therapy? a. Metronidazole (Flagyl) b. Ciprofloxacin (Cipro) C. Vancomycin (Vancocin) d. Tobramycin (Nebcin)

C. Vancomycin (Vancocin)

The nurse is reviewing the therapeutic effects of nonsteroidal anti-inflammatory drugs (NSAIDs), which include which effect? a. Anxiolytic b. Sedative c. Antipyretic d. Antimicrobial

C. antipyretic

The nurse in the intensive care unit is providing care for only one patient, who was admitted in septic shock. Based on this information, which care delivery model is the nurse implementing? A) Functional B) Primary C) Case method D) Team

C. case method.

Which type of managed care allows patients the greatest choice of providers, medications, and medical devices? A) Health maintenance organization B) Integrated delivery network C) Preferred provider organization D) Employment-based private insurance

C. preferred provider organization

Which is an example of self-knowledge if thought or stated by the nurse? A.) "I know that I should take the client's apical pulse for1 minute before giving digoxin." B.) "I know that I should follow the client's wishes even though it is not what I would want." C.) "I know that I have religious beliefs that may make it difficult to take care of some clients." D.) "I know that I need to honor the client's request not to discuss his health concern with the family."

C.) "I know that I have religious beliefs that may make it difficult to take care of some clients."

a scopolamine transdermal patch (Transderm-Scop). The nurse provides teaching for the use of A patient with motion sickness is planning a cross-country car trip and has a new prescription for this patch medication. The patient shows a correct understanding of the teaching with which statement? A.) "I will change the patch every day." B.) "I will change the patch every other day." C.) "I will change the patch every 3 days." D.) "I will remove the patch only if it stops working."

C.) "I will change the patch every 3 days."

A 62-year-old man with emphysema does not understand the need to stop smoking at this age because lung problems already exist. Which would be the best response to his statement? A.) "You should quit so your family does not get sick from expOsure to secondhand smoke." B.) "You will need to use oxygen, but remember it Is is a a fire hazard to smoke with oxygen in your home." C.) "Once you stop smoking, your body will begin to repair some of the e damage to your lungs." D.) You should ask your primary care provider for a prescription for a nicotine path to help you quit."

C.) "Once you stop smoking, your body will begin to repair some of the damage to your lungs."

A woman who is in the first trimester of pregnancy has been experiencing severe morning sickness. She asks, "I've heard that ginger tablets may be a natural way to ease the nausea and vomiting. Is it okay to try them?" What is the nurse's best response? A.) "They are a safe and natural remedy for nausea when you are pregnant. B.) "Go ahead and try them, but stop taking them once the nausea is relieved." C.) "Some health care providers do not recommend ginger during pregnancy. Let's check with your provider." D.) . "You will need to wait until after the first trimester to try them."

C.) "Some health care providers do not recommend ginger during pregnancy. Let's check with your provider."

When teaching a patient who will be receiving antihistamines, the nurse will include which instructions? (Select all that apply.) A.) "Antihistamines are generally safe to take with over-the-counter medications." B.) "Take the medication on an empty stomach to maximize absorption of the drug." C.) "Take the medication with food to minimize gastrointestinal distress." D.) "Drink extra fluids if possible." E.) "Antihistamines may cause restlessness and disturbed sleep. F.. "Avoid activities that require alertness until you know how adverse effects are tolerated."

C.) "Take the medication with food to minimize gastrointestinal distress." D.) "Drink extra fluids if possible. F.. "Avoid activities that require alertness until you know how adverse effects are tolerated."

A laxative has been ordered for a patient. The nurse checks the patient's medical history and would be concerned if which condition is present? A.) High ammonia levels due to liver failure B.) Diverticulosis C.) . Abdominal pain of unknown origin D.). Chronic constipation

C.) . Abdominal pain of unknown origin

The nurse is preparing to assess a client's cardiovascular system. Which technique should be used when assessing the carotid arteries? A.) Perform capillary refill B.) Palpate for lifts and heaves C.) Auscultate each side for hums D.) Assess skin color and temperature

C.) Auscultate each side for hums

The ethics committee is consulted because an older client asked for a do not resuscitate order; however, the family does not agree. Which model will the ethics committee most likely use in this spatient's case? A.) Social justice B.) Patient benefit C.) Autonomy D.) DNAR determination

C.) Autonomy

A client is prescribed continuous cardiac monitoring. Which action should the nurse take to ensure an accurate electrocardiogram tracing? A.) Select electrode placement sites over bony prominences. B.) Apply the electrodes immediately after cleansing the skin, before the alcohol evaporates. C.) Before applying the electrodes, rub the placement sites with gauze until the skin reddens. D.) Ensure that the gel on the back of the electrodes is dry.

C.) Before applying the electrodes, rub the placement sites with gauze until the skin reddens.

Which information provides the most reliable data about the effectiveness of airway suctioning? A.) The amount, color, consistency, and odor of secretions B.) The patient's tolerance for the procedure. C.) Breath sounds, vital signs, and pulse oximetry before and after suctioning D.) The number of suctioning passes required to clear secretions.

C.) Breath sounds, vital signs, and pulse oximetry before and after suctioning.

During a routine checkup, a patient states that she is unable to take the prescribed antihistamine because of one of its most common adverse effects. The nurse suspects that which adverse effect has been bothering this patient? A.) Constipation B.) Abdominal cramps C.) Drowsiness D.) Decreased libido

C.) Drowsiness

The nurse administers an antitussive/expectorant Cough preparation to a patient with bronchitis. Which response indicates to the nurse that the medication is effective? A.) The amount of sputum decreases with each dose administered. B.) Cough is completely suppressed, and the patient is able to sleep through the night. C.) Dry, unproductive cough is reduced, but voluntary coughing is more productive. D.) Involuntary coughing produces large amounts of thick yellow sputum.

C.) Dry, unproductive cough is reduced, but voluntary coughing is more productive.

The nurse applies bilateral wrist restraints to a client threatening to leave the hospital against medical advice. What is the nurse's action considered? A.) Assault and battery B) Felony C.) False Imprisonment D) Quasi-intentional tort

C.) False Imprisonment

When giving dextromethorphan, the nurse understands that this drug suppresses the cough reflex by which mechanism of action? A.) Causing depression of the central nervous system B.) Anesthetizing the stretch receptors C.) Having direct action on the cough center D.) Decreasing the viscosity of the bronchial secretions

C.) Having direct action on the cough center

A client receives treatment for heart failure. Which information in the client's health history contributed the development of this health problem? A.) Plays golf twice a week B.) Works as a home contractor C.) Ingests a six pack of beer a day D.) Coaches Little League in the summer

C.) Ingests a six pack of beer a day

A patient has been treated with alosetron (Lotronex) for severe for 2 weeks. She calls the clinic and tells the nurse that she has been irritable bowel syndrome (IBS) experiencing constipation for 3 days. The nurse will take which action? A.) Advise the patient to increase intake of fluids and fiber. B.) Advise the patient hold the drug for 2 days. C.) Instruct the patient to stop taking the drug and to come to the clinic right away to be evaluated. D.) Instruct the patient to continue the alosetron and to take milk of magnesia for the constipation.

C.) Instruct the patient to stop taking the drug and to come to the clinic right away to be evaluated.

The prescriber has changed the patient's medication regimen to include the leukotriene receptor antagonist (LTRA) montelukast (Singulair) to treat asthma. The nurse will emphasize which point about this medication? A.) The proper technique for inhalation must be followed. B.) The patient needs to keep it close by at all times to treat acute asthma attacks. C.) It needs to be taken every day on a continuous schedule, even if symptoms improve. D.) When the asthma symptoms improve, the dosage schedule can be tapered and eventually discontinued.

C.) It needs to be taken every day on a continuous schedule, even if symptoms improve.

A patient has just had a chest tube inserted to dry-seal suction drainage. Which is a correct nursing intervention for maintenance? A.) Keep the head of the bed flat for 6 hours. B) Immobilize the patient's arm on the affected site. C.) Keep the drainage system lower than the insertion site. D.) Drain condensation into the humidifier when collects in the tubing.

C.) Keep the drainage system lower than the insertion site.

A gardener needs a decongestant because of seasonal allergy problems and asks the nurse whether he should take an oral form or a nasal spray. Which of these is a benefit of orally administered decongestants? A.) Immediate onset B.) A more potent effect C.) Lack of rebound congestion D.) Shorter duration

C.) Lack of rebound congestion

The nurse is giving oral mineral oil as an ordered laxative dose. The nurse will take measures to prevent which potential problem that may occur with mineral oil? A.) Fecal impaction B.) Electrolyte imbalances C.) Lipid pneumonia D.) Esophageal blockage

C.) Lipid pneumonia

While recovering from surgery, a 74-year-old woman started taking a stimulant laxative, senna (Senokot), to relieve constipation caused by the pain medications. Two weeks later, at her follow-up appointment, she tells the nurse that she likes how "regular" her bowel movements are now that she is taking the laxative. Which teaching principle is appropriate for this patient? A.) She needs to be sure to take this medication with plenty of fluids. B.) It is important to have a daily bowel movement to promote bowel health. C.) Long-term use of laxatives often results in decreased bowel tone and may i I to dependency. D.) She needs to switch to glycerin suppositories to continue having daily bowel movements.

C.) Long-term use of laxatives often results in decreased bowel tone and may i I to dependency.

A patient is severely constipated and needs immediate relief. The nurse knows that which class of laxative will provide the most rapid results? A.) Bulk-forming laxative, such as psyllium (Metamucil) D.ORG B.) Stool softener, such as docusate salts (Colace) C.) Magnesium hydroxide (MOM) D. ) Magnesium oxide tablets

C.) Magnesium hydroxide (MOM)

The nurse prepares material about blood pressure control for a group of community members. What should the nurse include about baroreceptors? A.) Respond to levels of oxygen in the blood B.) Activate the sympathetic nervous system C.) Maintain blood pressure with body position changes D.) Increase respiration based upon carbon dioxide levels

C.) Maintain blood pressure with body position changes

Which is the most important reason for nurses to be critical thinkers? A.) Nurses need to follow policies and procedures. B.) ) Nurses work with other healthcare team members. C.) Nurses care for clients who have multiple health problems. D.) Nurses have to be flexible and work variable schedules.

C.) Nurses care for clients who have multiple health problems.

During an assessment of a newly admitted client the nurse measures blood pressure, abdominal circumference, and pulse rate. Which critical-thinking skill is the nurse using? A.) Recognizing gaps in one's own knowledge B.) Recognizing the need for more information C.) Objectively gathering information on a problem or issue D. ) Evaluating the credibility and usefulness of sources of information

C.) Objectively gathering information on a problem or issue

When evaluating a patient's use of a metered-dose inhaler (MDI), the nurse notes that the patient is unable to coordinate the activation of the inhaler with her breathing. What intervention is most appropriate at this time? A.) Notify the doctor that the patient is unable to use the MDI. B.) Obtain an order for a peak flow meter. C.) Obtain an order for a spacer device. D.) Ask the prescriber if the medication can be given orally.

C.) Obtain an order for a spacer device.

A patient with a treatable form of breast cancer has decided not to pursue radiation or chemotherapy; however, the nurse coerces the patient into accepting the treatment by reminding her about her responsibilities with her children. What type of behavior has the nurse displayed? A.) Nonmaleficence B.) Autonomy C.) Paternalism D) Beneficence

C.) Paternalism

Chest percussion and postural drainage would be an appropriate intervention for which condition? A.) Congestive heart failure. B.) Pulmonary edema C.) Pneumonia D.) Pulmonary Embolus

C.) Pneumonia

The staff development trainer considers using a feminist theory to explain the resolution of an ethical dilemma. What will the trainer use when implementing this theory? A.) Rules B.) Principles C.) Relationships D.)) Risk-benefit analysis

C.) Relationships

When reviewing the health history of a patient, the nurse will note that a potential contraindication to potassium supplements exists if the patient has which problem? A.). Burns B.) Diarrhea C.) Renal disease D.) Cardiac tachydysrhythmias

C.) Renal disease

The nurse is assigned to care for a client with Guillain-Barré syndrome; however, the nurse does not understand the disorder. What is the best way for the nurse to address this gap in knowledge? A. ) Talk with the family. B. ) Complete the client assessment. C.) Research and read about the disorder. D. ) Ask another nurse to explain the disorder.

C.) Research and read about the disorder.

The nurse is assigned to care for a client with Guillain-Barré syndrome; however, the nurse does not understand the disorder. What is the best way for the nurse to address this gap in knowledge? A.) Talk with the family. B.) Complete the client assessment. C.) Research and read about the disorder. D.) Ask another nurse to explain the disorder.

C.) Research and read about the disorder.

The family of an alert, oriented, and competent client does not agree with the client's decision to have a do not resuscitate order and asks the healthcare team to ignore the decision. The healthcare team follows the client's wishes and the family takes the matter to court. The court sides with the family and the order has to be removed, What occurred in this situation? A.) An integrity- producing (good) compromise B.) An ethically sound compromise C.) Settlement of an issue by force D.) An effort to keep peace on the unit

C.) Settlement of an issue by force

When administering a bulk-forming laxative, the nurse instructs the patient to drink the medication mixed in a full 8-ounce glass of water. Which statement best explains the rationale for this instruction? A.) The water acts to stimulate bowel movements. B.) The water will help to reduce the bulk of the intestinal contents.. C.) These laxatives may cause esophageal obstruction if taken with insufficient water. D.) The water acts as a lubricant to produce bowel movements.

C.) These laxatives may cause esophageal obstruction if taken with insufficient water.

A conscious and competent client who is ventilator dependent wants to be removed from the ventilator even though the nurse believes the client is committing suicide. What is the nurse demonstrating when standing at the bedside holding the client's hand? A) Value set B.) Value system C.) Value neutrality D.) Value awareness

C.) Value neutrality

A patient will be taking bismuth subsalicylate (Pepto-Bismol) to control diarrhea. When reviewing the patient's other ordered medications, the nurse recognizes that which medication will interact significantly with the Pepto-Bismol? A.) Acetaminophen (Tylenol), an analgesic B.) Levothyroxine (Synthroid), a thyroid replacement drug C.) Warfarin (Coumadin), an anticoagulant D.) Fluoxetine (Prozac), an antidepressant

C.) Warfarin (Coumadin), an anticoagulant

Which is an example of practical knowledge? A.) The tricuspid valve is between the right atrium and ventricle of the heart. B.) The pancreas does not produce enough insulin in type 1 diabetes. C.) When assessing the abdomen, you should auscultate before palpating. D.) Research shows pain medication given intravenously acts faster than by other routes.

C.) When assessing the abdomen, you should auscultate before palpating.

The nurse is monitoring drug levels for a patient who is receiving theophylline. The most recent theophylline level was 22 mcg/mL, and the nurse evaluates this level to be: A.) below the therapeutic level. B.) at a therapeutic level. C.) above the therapeutic level. D.) at a toxic level.

C.) above the therapeutic level.

A client in labor after 32 weeks' gestation is eager to deliver. Which client statement indicates that teaching provided about fetal development was effective ? A.) "The baby's lung are well developed now, but he will be at increased risk for SIDS if I deliver early." B.) "We should try to stop this labor now because the baby will be born with sleep apnea if I deliver this early." C.) "If I deliver this early my baby is at risk for respiratory distress syndrome, a condition that can be life threatening." D.)"Thanks for reassuring me; I was pretty sure there isn't much risk to the baby this far along in my pregnancy."

C.)"If I deliver this early my baby is at risk for respiratory distress syndrome, a condition that can be life threatening."

Which intervention is likely to reduce the risk of postoperative atelectasis? Select all that aply. A.)Administer bronchodilators. B.)Apply low- flow oxygen. C.)Encourage coughing and deep breathing and Coughing. D.)Administer pain medication. E.)Assist to move and reposition in bed.

C.)Encourage coughing and deep breathing and coughing. D.)Administer pain medication. E.)Assist to move and reposition in bed.

Which provides evidence-based support for the contribution that advanced practice nurses (APNs. make within healthcare? A) Reduced usage of diagnostics using advanced technology B) Decreased number of unnecessary visits to the emergency department C) Improved patient compliance with prescribed treatments D) Increased usage of complementary alternative therapies

C: Improved patient compliance with prescribed treatments

The nurse is teaching nursing assistive personnel ( NAP ) how to give a complete bed bath . Which instruction should the nurse include ? A ) " Cleanse only those areas likely to cause odor . " B) " Provide the patient with warm water for washing his perineum . " C ) " Wash the patient's back , buttocks , and perineum first . " D ) " Bathe the patient from head - to - toe , cleanest areas first . "

D ) " Bathe the patient from head - to - toe , cleanest areas first . "

A patient's ankles appear swollen . When the nurse assesses the edema , the skin depresses 6 mm , and the depression lasts 2 minutes . How should the nurse document this finding ? A) Trace edema . B ) +1 edema C ) +2 edema D ) +3 edema

D ) +3 edema

Bath water should be prepared at which temperature to prevent chilling and excess drying of the skin ? A ) 99 ° F ( 37.2 ° C ) B ) 102 ° F ( 38.9 ° C ) C ) 103 ° F ( 39.4 ° C ) D ) 105 ° F ( 40.6 ° C )

D ) 105 ° F ( 40.6 ° C )

When testing near vision , the nurse should position printed text how many inches away from the patient ? A ) 20 B ) 18 C ) 16 D ) 14

D ) 14

A patient is given furosemide 40 mg orally at 0900. The duration of action for this drug is approximately 6 hours after oral administration . At which time in military hours should the nurse no longer expect to see the effects of this drug ? A ) 0930 B ) 1000 C ) 1100 D ) 1500

D ) 1500

Which is an abnormal capillary refill finding that the nurse should report ? A ) 1 second B ) 2 seconds C ) 3 seconds D ) 4 seconds

D ) 4 seconds

Which aspect of restraint use can the nurse delegate to the nursing assistive personnel ? A) Assessing the patient's status B ) Determining the need for restraint C ) Evaluating the patient's response to restraints D ) Applying and removing the restraints

D ) Applying and removing the restraints

Small hemorrhages are noted under the nailbed of a patient with a history of intravenous drug abuse . With what should the nurse realize this finding is associated ? A ) Low albumin levels B ) Zinc deficiency C ) Renal disease 4 D ) Bacterial endocarditis

D ) Bacterial endocarditis

A child is brought to the emergency department after swallowing liquid cleanser . He is awake , alert and able to swallow . Which action should the nurse take first ? A) Administer a dose of syrup of ipecac . B ) Administer activated charcoal immediately . C ) Give water to the child immediately . D ) Call the nearest poison control center .

D ) Call the nearest poison control center .

What is a patient considered who is infected with a virus but who does not have any outward sign of the disease ? A ) Pathogen B ) Fomite C ) Vector D ) Carrier

D ) Carrier

The nurse notes that the electrical cord on an IV infusion pump is cracked . Which action by the nurse is best ? A ) Continue to monitor the pump to see if the crack worsens . B ) Place the pump back on the utility room shelf . C ) A small crack poses no danger so continue using the pump . D ) Clearly label the pump and send it for repair .

D ) Clearly label the pump and send it for repair .

Despite less - restrictive interventions , a patient's behavior escalates , requiring emergency application of restraints . Which action must the nurse do in this situation ? A ) Obtain a physician's order before applying restraints . B ) Monitor the patient's status every 4 hours while restrained . C ) Release the restraints and check circulation every hour . D ) Continually reevaluate the patient's need for restraint

D ) Continually reevaluate the patient's need for restraint

While palpating the anterior chest , the nurse notes crackling in the skin around the patient's chest tube insertion site . What should the nurse realize this finding indicates ? A ) Tactile fremitus B ) Egophony C ) Bronchophony D ) Crepitus

D ) Crepitus

The admission assessment form indicates that the patient has pedal pulses that are rated 1 in amplitude . What should this finding indicate about the client's pulses ? A ) Bounding B ) Normal C ) Full D ) Diminished

D ) Diminished

The nurse notes that an older client has both upper siderails raised when in the bed . What should the nurse realize about the use of the siderails for this client ? A ) Aids in independence B ) Serves as a mechanical restraint C ) Reminds the client to call for help D ) Eliminates the need for another restraint

D ) Eliminates the need for another restraint

An older client is suspected of drinking a glass of water with crushed pills that were being dissolved in efforts to destroy unused doses . What should the nurse do first ? A ) Schedule for emergency dialysis . B ) Provide a dose of syrup of ipecac . C ) Prepare to administer activated charcoal . D ) Find out the name of the medication being destroyed .

D ) Find out the name of the medication being destroyed .

The primary care provider prescribes furosemide 40 mg IV for a patient with heart failure . Which drug name is used in this prescription ? A ) Chemical B ) Brand C ) Trade D ) Generic

D ) Generic

An adult admitted to the hospital after a stroke does not respond to verbal stimuli . What should the nurse do next to try to provoke a response ? A ) Apply pressure to the mandible at the jaw . B ) Rub the patient's sternum . C ) Squeeze the trapezius muscle . D ) Gently shake the patient's shoulder .

D ) Gently shake the patient's shoulder .

A patient with a history of seizures who takes phenytoin is at risk for which oral problem ? A ) Dryness of the mouth B) Bitter taste C ) Demineralization of the tooth enamel D ) Gingival hyperplasia

D ) Gingival hyperplasia

Which scheduled hygiene care is usually thought of as including a back massage to help the patient relax ? A ) Afternoon care B ) Early morning care C ) Morning care D ) Hour of sleep care

D ) Hour of sleep care

The patient is just beginning to feel symptoms after being exposed to an upper respiratory infection . Which antibody would most likely be found in a test of immunoglobin levels ? A ) IgA B ) IgE C ) IgG D ) IgM

D ) IgM

Wearing poorly fitting shoes may result in which condition ? A ) Tinea pedis B ) Plantar wart C ) Excoriation D ) Ingrown toenail

D ) Ingrown toenail

In which situation would using standard precautions be adequate ? Select all that apply . A ) Interviewing a client with a productive cough B ) Helping a client to perform his own hygiene care C ) Aiding a client to ambulate after surgery D ) Inserting a peripheral intravenous catheter E ) Emptying a urine collection bag

D ) Inserting a peripheral intravenous catheter E ) Emptying a urine collection bag

While assessing an older adult patient , the nurse notes clubbing of the fingers . What does this finding indicate to the nurse ? A ) Fungal infection B ) Malnutrition C ) Iron deficiency D ) Long - term hypoxia

D ) Long - term hypoxia

Which instruction is most important for the nurse to include when teaching a mother of a 3 - year - old about protecting the child against accidental poisoning ? A ) Store medications on countertops out of the child's reach . B ) Purchase medication in child - resistant containers . C ) Take medications in front of the child and explain that they are for adults only . D ) Never leave the child unattended around medications or cleaning solutions .

D ) Never leave the child unattended around medications or cleaning solutions .

To assure effectiveness , when should the nurse stop rubbing antiseptic hand solution over all surfaces of the hands ? A ) When fingers feel sticky B ) After 5 to 10 seconds C ) When leaving the client's room D ) Once fingers and hands feel dry

D ) Once fingers and hands feel dry

The nurse has been teaching a student how to perform mouth care for her unconscious patient . The student will show evidence of learning if the patient is placed in which position ? 1 A ) Supine B ) Prone C ) Semi - Fowler's D ) Side - lying

D ) Side - lying

Which skin assessment finding would cause the nurse to suspect dehydration in a middle - aged patient admitted to the hospital with traveler's diarrhea ? A ) Edema B ) Hyperhidrosis C ) Pallor D ) Tenting

D ) Tenting

How should the nurse document high - pitched breath sounds produced by narrowed airways ? A ) Rales B ) Crackles C ) Rhonchi D ) Wheezing

D ) Wheezing

After instructing a mother about nutrition for a preschool-age child, which statement by the mother would indicate correct understanding of the topic? A) "I usually use dessert only as a reward for eating other foods." B) "I will hide vegetables in casseroles and stews to get my child to eat them." C) "I do not give my child snacks; they simply spoil his appetite for meals." D) "I know that lifelong food habits are developed during this stage of life."

D) "I know that lifelong food habits are developed during this stage of life."

Which response by the patient demonstrates an internal locus of control? A) "My blood sugar wouldn't be out of control if my wife prepared better foods." B) "I knew I shouldn't have come to this hospital; l'd be better if I hadn't." C) "God must be getting even with me for my past behavior." D) "I'm just glad to be alive; the accident could've been a lot worse."

D) "I'm just glad to be alive; the accident could've been a lot worse."

A client asks why aspirin has been prescribed even though pain is not being experienced. What should the nurse respond to this client? A) "It dilates blood vessels." B ) "It dissolves clots in the body." C) "It keeps blood pressure within normal range." D) "It stops the mechanism that makes blood clots."

D) "It stops the mechanism that makes blood clots."

Which urine specific gravity would be expected in a patient with dehydration? A) 1.002 B) 1.010 C) 1.025 D) 1.030

D) 1.030

When testing near vision, the nurse should position printed text how many inches away from the patient? A) 20 B) 18 C) 16 D) 14

D) 14

Which differentiates a nursing diagnosis from a medical diagnosis? A) Terminology for the client's disease or injury B) A part of the client's medical diagnosis C) The client's presenting signs and symptoms D) A client's response to a health problem

D) A client's response to a health problem

Which describes the difference between a collaborative problem and a medical diagnosis? A) A collaborative problem is treated by the nurse; a physician is responsible for the treatment of a medical problem. B) A collaborative problem is a nursing diagnosis that requires specific orders from a physician; a medical diagnosis directs all nursing care. C) A collaborative problem has the potential to become an actual nursing diagnosis; a medical diagnosis rarely changes. D) A collaborative problem requires intervention by the nurse and physician or other professional; a medical diagnosis requires intervention by a physician.

D) A collaborative problem requires intervention by the nurse and physician or other professional; a medical diagnosis requires intervention by a physician.

The nurse prepares an educational program to address a health need for a community. What should the nurse include when identifying the goals for these community members? A) Scale to determine when the goals are achieved B) Outcome labels to plan for the expected health status C) Indicators to evaluate each community member's status D) Actions to promote health behaviors across all life stages

D) Actions to promote health behaviors across all life stages

Which process requires energy to maintain the unique composition of extracellular and intracellular compartments? A) Diffusion B) Osmosis C) Filtration D) Active transport

D) Active transport

A client incorporates alternative healthcare into regular health practices. For which alternative therapy should the patient visit a formally trained practitioner? A) Use of herbs and roots B) Application of oils and poultices C) Burning of dried plants D) Acupuncture

D) Acupuncture

A patient who has been diagnosed with breast cancer decides on a treatment plan and feels positive about her prognosis. Assuming the cancer diagnosis represents a crisis, this patient is most likely experiencing which phase of crisis? A) Precrisis B) Impact C) Crisis D) Adaptive

D) Adaptive

Chloride, bicarbonate, phosphate, and sulfate are examples of what type of charged particles and why? A) Cations, because they carry a positive charge B) Cations, because they carry a negative charge C) Anions, because they carry a positive charge D) Anions, because they carry a negative charge

D) Anions, because they carry a negative charge

A patient of Orthodox Jewish faith is admitted to the hospital with heart failure on Yom Kippur. The physician prescribes digoxin 0.25 mg to be given orally for this patient. Based on the patient's religious affiliation, which action should the nurse take? A) Administer the medication as prescribed. B) Hold the medication until after Yom Kippur. C) Explain the importance of taking the medication despite the holiday. D) Ask the physician to change the route of administration.

D) Ask the physician to change the route of administration.

A patient of Orthodox Jewish faith is admitted to the hospital with heart failure on Yom Kippur. The physician prescribes digoxin 0.25 mg to be given orally for this patient. Based on the patient's religious affiliation, which action should the nurse take? A. Administer the medication as prescribed. B.) Hold the medication until after Yom Kippur. C) Explain the importance of taking the medication despite the holiday. D) Ask the physician to change the route of administration.

D) Ask the physician to change the route of administration.

A high school graduate desires to become a registered nurse in order to work in a community setting. In which type of program should this student enroll? A) RN-BSN B) Associate degree C) Diploma program D) Baccalaureate degree

D) Baccalaureate degree

Which organ relies almost exclusively on glucose for energy? A) Liver B) Heart C) Pancreas D) Brain

D) Brain

A 6-year-old child becomes upset after learning that a blood sample is needed. The child's mother scolds the child and tells him to "act your age." How should the nurse proceed? A) Request that the mother leave the room immediately. B) Request the help of a coworker to hold the child down. C) Inform the child that "this won't hurt a bit." D) Calmly approach the child and tell him what is going to happen.

D) Calmly approach the child and tell him what is going to happen.

The nurse reviews an entry in a medical record before clicking the SAVE button, When performing this review, the nurse analyzes the content for completeness, clarity, accuracy, comprehensiveness, and which other criterion? A) Concise B) Category C) Character D) Chronological

D) Chronological

While palpating the anterior chest, the nurse notes crackling in the skin around the patient's chest tube insertion site. What should the nurse realize this finding indicates? A) Tactile fremitus B) Egophony C) Bronchophony D) Crepitus

D) Crepitus

A patient admitted with depression has a nursing diagnosis of Chronic Low Self-Esteem. Which NOC outcome is essential for this nursing diagnosis? A) Decision Making B) Distorted Thought Content C) Role Performance D) Depression Level

D) Depression Level

Which statement best describes standardized care plans and unit standards of care? A) Usually describe ideal nursing care B) Include specific goals and nursing orders C) Become a part of the patient's comprehensive care plan D) Describe the care needed by patients in defined situations

D) Describe the care needed by patients in defined situations

Which describes the most important use of nursing diagnosis? A) Differentiates the nurse's role from that of the physician B) Identifies a body of knowledge unique to nursing C) Helps nursing develop a more professional image D) Describes the client's needs for nursing care

D) Describes the client's needs for nursing care

Which describes the passive process by which molecules of a solute move through a cell membrane from an area of higher concentration to an area of lower concentration? A) Osmosis B) Filtration C) Hydrostatic pressure D) Diffusion

D) Diffusion

The admission assessment form indicates that the patient has pedal pulses that are rated 1 in amplitude. What should this fin ding indicate about the client's pulses? A) Bounding B) Normal C) Full D) Diminished

D) Diminished

Which law does a hospital use to determine if a patient with no healthcare coverage who secking medical care should receive care or be transferred to another facility? A) Health Care Quality Improvement Act (HCQIA) B) Patient Self-Determination Act (PSDA) C) Newborns" and Mothers" Health Protection Act (NMHPA) D) Emergency Medical Treatment and Active Labor Act (EMTALA)

D) Emergency Medical Treatment and Active Labor Act (EMTALA)

A patient is in crisis. After assessing the situation, what should the nurse do first? A) Determine the imminent cause of the crisis. B) Intervene to relieve the patient's anxiety, C) Decide on the type of help the patient needs. D) Ensure the safety of both the nurse and patient.

D) Ensure the safety of both the nurse and patient.

Which is a characteristic of chronic wounds? A) Are full-thickness wounds B) Result from pressure C) Are usually infected D) Exceed the typical healing time

D) Exceed the typical healing time

The nurse suspects that a client is experiencing stress. What finding did the nurse use to make this clinical determination? A) Works a full-time job B) Goes to the gym three times a week to work out C) Spouse takes care of the children when grocery shopping D) Expresses a lack of time to waste at a healthcare appointment

D) Expresses a lack of time to waste at a healthcare appointment

A client demonstrates a cognitive response to stress. What observation caused the nurse to make this clinical decision? A) Skipping meals B) Avoiding talking with others C) Sleeping late into the morning D) Forgetting where medications are stored

D) Forgetting where medications are stored

An adult admitted to the hospital after a stroke does not respond to verbal stimuli. What should the nurse do next to try to provoke a response? A) Apply pressure to the mandible at the jaw. B) Rub the patient's sternum. C) Squeeze the trapezius muscle. D) Gently shake the patient's shoulder.

D) Gently shake the patient's shoulder.

Which intervention by the nurse first helps to establish a trusting nurse-patient relationship? A) Avoiding topics that may provoke emotional responses from the patient B) Listening to the patient while performing care activities C) Performing care interventions quietly and respectfully D) Greeting the patient by name whenever entering the patient's room

D) Greeting the patient by name whenever entering the patient's room

The nurse is updating the care plan of a patient who must undergo a right mastectomy for breast cancer. Which nursing diagnosis should the nurse anticipate in expectation of the body changes associated with the upcoming surgery? A) Deficient Knowledge B) Impaired Comfort C) Hopelessness D) Grieving

D) Grieving

The nurse notes that a client has had significant diarrhea over the past several days. Which level in the client's arterial blood gas result will be most affected? A) рH B) PO2 C) РСО2 D) HCO3

D) HCO3

When performing a central venous catheter dressing change, which step is correct? A) Wear sterile gloves while removing and discarding the soiled dressing. B) Apply pressure on the catheter-hub junction when removing the soiled dressing. C) Place a sterile transparent dressing over the site and the catheter-hub junction. D) Have the patient wear a mask or turn his head away from the site.

D) Have the patient wear a mask or turn his head away from the site.

Which best describes self-esteem? A) View of oneself as a unique human being B) One's mental image of one's physical self C) One's overall view of oneself D) How well one likes oneself

D) How well one likes oneself

The nurse auscultates low-pitched infrequent bowel sounds in a patient recovering from a bowel resection. How should this finding be documented? A) Hyperactive bowel sounds B) Abdominal bruit sounds C) Normal bowel sounds D) Hypoactive bowel sounds

D) Hypoactive bowel sounds

After a physician discusses cancer treatment options, the patient asks which treatment to choose. Which response by the nurse is best? A) If I were you, I'd go with chemotherapy ? B) What do you think about radiation ? C) Why don't you see what your spouse thinks? D) I'll give you some information about each opinion?

D) I'll give you some information about each opinion?

A nurse has sound, scientific evidence to support changing a procedure that would reduce catheter-related infections on the unit. The nurse manager is unwilling to make the change because it would be too costly. Which response by the nurse represents assertive communication? A) This is a widely used practice. If you read more research, you'd probably wonder why we aren't already doing it B) There is extensive evidence to support the new method, but I don't want to create an issue C) Is the budget more important to the hospital than reducing infections and patient suffering D) I'd like to help gather information regarding the cost of new material versus the savings in treating infections.

D) Id like to help gather information regarding the cost of new material versus the savings in treating infections.

Which intervention would help to prevent or relieve persistent nausea? A) Assess for signs of dehydration. B) Provide dietary supplements. C) Have the patient sit in an upright position for 30 minutes after eating. D) Immediately remove any food that the patient cannot eat.

D) Immediately remove any food that the patient cannot eat.

The nurse notes that a client's respiratory rate is 30 and irregular. Which nursing diagnosis should be identified to help guide this client's care ? A) Anxiety B) Altered oxygenation level C) Risk for poor oxygenation D) Ineffective breathing pattern

D) Ineffective breathing pattern

A 35-year-old patient with testicular cancer is undergoing chemotherapy, which leaves him unable to help care for his young children. As a result, his spouse misses work whenever the children are ill and has become increasingly distressed over the situation. What is the spouse most likely experiencing? A) Role strain B) Interpersonal role conflict C) Role performance D) Inter-role conflict

D) Inter-role conflict

A frail, elderly patient admitted with dehydration to a medical-surgical unit is exhibiting confusion, distractibility, memory loss, and irritability. What is most important for the nurse do? A) Recognize these symptoms as signs of normal, physiological aging. B) Obtain a urine specimen before notifying the primary care provider. C) Be sure she is placed in a room occupied with another patient. D) Interview the patient to screen for clinical depression.

D) Interview the patient to screen for clinical depression.

A patient has anemia. An appropriate goal for that the patient would be for him to increase the intake of which nutrient? A) Calcium B) Magnesium C) Potassium D) Iron

D) Iron

Because of religious beliefs, a patient with which affiliation will most likely refuse a blood transfusion? A) Islam B) Baha'i C) Hinduism D) Jehovah's Witness

D) Jehovah's Witness

Which collaborative interventions will help prevent paralytic ileus in a patient who underwent right hemicolectomy for colon cancer? A) Administer morphine 4 mg IV every 2 hours for pain. B) Administer IV fluids at 125 mL/hr. C) Insert an indwelling urinary catheter to monitor I&O. D) Keep the nasogastric tube to low suction.

D) Keep the nasogastric tube to low suction.

Which is the most appropriate goal for a patient with the nursing diagnosis of Deficient Fluid A) Electrolyte balance restored, as evidenced by improved levels of alertness and cognitive orientation. B) Electrolyte balance restored, as evidenced by sodium returning to normal range. C) Patient demonstrates effective coughing and deep breathing techniques. D) Maintains fluid balance, as evidenced by moist mucous membranes and urinating every 4 hours

D) Maintains fluid balance, as evidenced by moist mucous membranes and urinating every 4 hours

The family member of a client with numerous physiological complaints tells the nurse that the client is pretending to be sick to avoid going to work. Which somatoform disorder is this patient most likely experiencing? A) Hypochondriasis B) Somatization C) Somatoform pain disorder D) Malingering

D) Malingering

In evaluating a client's blood pressure for hypertension, what is the most important action for the nurse to take? A) Use the same type of manometer each time. B) Auscultate all five Korotkoff sounds. C) Measure the blood pressure in both arms. D) Monitor the blood pressure for a pattern.

D) Monitor the blood pressure for a pattern.

A client's pulse oximetry reading is 90%. What action should the nurse take first? A.)Raise the head of the bed. B.)Prepare to administer oxygen. C.)Notify the healthcare provider. D.)Move the sensor to another area.

D) Move the sensor to another area.

The nursing diagnosis Disturbed Personal Identity is identified for a newly admitted patient. Which is an example of an individualized goal for this patient? A) Distorted Thought Control B) Anxiety Level C) Self-Mutilation Restraint D) No Self-Injury, Consistently Demonstrated

D) No Self-Injury, Consistently Demonstrated

A patient with a bleeding duodenal ulcer has difficulty concentrating, complains of shortness of breath, and experiences sleeplessness because of worrying. What is the nurse's priority after documenting this information in the nurses' notes? A) Provide emotional support for the patient using reflective listening technique. B) Do nothing; people with duodenal ulcers typically cannot work. C) Question the patient's family about the information received from the patient. D) Notify the primary care provider and ask for a referral to a mental health professional.

D) Notify the primary care provider and ask for a referral to a mental health professional.

A patient with end - stage cancer is prescribed morphine sulfate to reduce pain. For which effect is this medication prescribed? A) Supportive B) Restorative C) Substitutive D) Palliative

D) Palliative

After being informed of a cancer diagnosis, the patient responds, I'll do whatever you think I should do Which communication style is this patient using? A) Assertive B) Aggressive C) Passive aggressive D) Passive

D) Passive

The nurse prepares information for a newly admitted client. What should the nurse review to ensure that the client's rights are protected? A) HIPAA laws B) Policy manual C) Advance directives D) Patient Care Partnership

D) Patient Care Partnership

What do nursing codes of ethics support? A) Patients can receive emergency treatment regardless of their ability to pay. B) Nurses will educate patients about advance directives. C) Nurses with HIV must disclose their condition to their employer. D) Patients have the right to dignity, privacy, and safety.

D) Patients have the right to dignity, privacy, and safety

A client is scheduled for surgery to create a temporary ostomy. What should the nurse emphasize when teaching about this bowel diversion? A) Produces solid feces B) Creates two separate stomas C) Bypasses the large intestine D) Permits the bowel to rest and heal

D) Permits the bowel to rest and heal

should the nurse suspect is occurring with this patient? to the touch. There is a palpable cord along the vein and the fluid infusion is sluggish. What The nurse notes that a patient's intravenous catheter site is painful, edematous, red, and warm A) Infiltration B) Extravasation C) Hematoma D) Phlebitis

D) Phlebitis

The nurse receives a copy of a client's advance directive. What should the nurse do with this document? A) Return it to the client. B) Give it to the nursing supervisor. C) Hand it to the healthcare provider. D) Place it on the client's medical record.

D) Place it on the client's medical record.

Which intervention should the nurse take first to promote micturition in a patient who is having difficulty voiding? A) Insert an indwelling urinary catheter. B) Notify the provider immediately. C) Insert an intermittent, straight catheter. D) Pour warm water over the patient's perineum.

D) Pour warm water over the patient's perineum.

After an assessment, the nurse realizes that a client demonstrates cognitive responses to stress. What behaviors did the client demonstrate for the nurse to make this clinical determination? Select all that apply. A) Angry B) Lethargic C) Iritable D) Preoccupied E) Decreased attention to detail

D) Preoccupied E) Decreased attention to detail

Which phrase describes a characteristic of both standardized and individualized nursing care plans? A) Reflect critical thinking for a specific patient B) Are preprinted to apply to needs common to a group of patients C) Address a patient's individual needs D) Provide detailed nursing interventions

D) Provide detailed nursing interventions

The client's weight is appropriate for weight. Laboratory values and other assessments reflects normal nutritional status. However, the client tells the nurse, "I probably eat a little too much red meat. And what is this I hear about needing omega 3 oils in my diet ? I don't like to take supplements, and I think I could really improve my nutrition. Which nursing diagnoses should the nurse use when planning this client's care ? A) Balanced Nutrition B) Possible Imbalanced Nutrition: Less Than Body Requirements C) Risk for Imbalanced Nutrition: Less Than Body Requirements D) Readiness for Enhanced Nutrition

D) Readiness for Enhanced Nutrition

Which member of the healthcare team typically serves as the case manager? A) Occupational therapist B) Physician C)Physician's assistant D) Registered nurse

D) Registered nurse

Which outcome indicates that the plan of care for a client with diabetes and severe peripheral neuropathy was effective? A) Begins an aggressive exercise program B) Follows a diet plan of 1,200 calories per day C) Is fitted for deep-depth diabetic footwear D) Remains free of foot wounds

D) Remains free of foot wounds

The client has had a fever, ranging from 99.8°F orally to 103°F orally, over the past 24 hours. How should the nurse classify this fever? A) Constant B) Intermittent C) Relapsing D) Remittent

D) Remittent

Which is the principal system for regulation of fluid and electrolyte balance? A) Cardiac B) Vascular C) Pulmonary D) Renal

D) Renal

A patient is admitted with high BUN and creatinine levels, low blood pH, and elevated serum potassium level. Based on these laboratory findings the nurse suspects which diagnosis? A) Cystitis B) Renal Calculi C) Enuresis D) Renal Failure

D) Renal Failure

When caring for a client with a fever, what should the nurse expect to be increased? A) Urine output B) Sensitivity to pain C) Blood pressure D) Respiratory rate

D) Respiratory rate

Which factor is held in common by many of the world religions? A) Strict health code, including dietary laws B) Belief that one must submit to a god or gods C) Rules prohibiting alcohol consumption D) Sacred writings that reveal the nature of the Supreme Being

D) Sacred writings that reveal the nature of the Supreme Being

A patient's vital signs prior to a blood transfusion were T= 97.6°F (36.4°C); P= 72 beats/min; R = 22 breaths/min; and BP = 132/76 mm Hg. Twenty minutes after the transfusion started, the patient began complaining of feeling "itchy and hot" and a new rash is present on the patient's trunk. Vital signs - 24 breaths/min; BP = 146/88 mm Hg. Based on these are now T= 100.8°F (38.2°C); P= 82 beats/min; R findings, what is the priority intervention? A) Administer an antihistamine (anti-allergenic) medication. B) Flush the blood tubing with D,W immediately. C) Prepare for emergency resuscitation. D) Stop the blood transfusion immediately.

D) Stop the blood transfusion immediately.

A patient complains that urine is passed when coughing or sneezing. How should the nurse document this complaint in the patient's healthcare record? A) Transient incontinence B) Overflow incontinence C) Urge incontinence D) Stress incontinence

D) Stress incontinence

The nurse manager learns of a staff nurse's behavior with a client. Which information suggests that nurse has committed a boundary violation? A) Mentioned that a client's birthday was coming up B) Observed talking with a client's spouse in the hallway C) Asked the healthcare provider to change a client's pain medication D) Switched an assignment so that the nurse can take care of the client

D) Switched an assignment so that the nurse can take care of the client

The nurse educates a patient about the primary risk factors for iritable bowel syndrome. Which patient behavior indicates teaching has been effective? A) Reduces intake of gluten-containing products. B) Does not consume foods that contain lactose. C) Consumes only two servings of caffeinated beverages per day. D) Takes measures to reduce stress level.

D) Takes measures to reduce stress level.

The nurse is assessing a patient for depression. Which set of behavioral symptoms may indicate depression? A) Preoccupation with loss, self-blame, and ambivalence B) Anger, helplessness, guilt, and sadness C) Anorexia, insomnia, headache, and constipation D) Tearfulness, withdrawal, and present substance abuse

D) Tearfulness, withdrawal, and present substance abuse

Which is an example of data that should be validated? A) The client's weight measures 185 lb at the clinic. B) The client's liver function test results are elevated. C) The client's blood pressure is 160/94 mm Hg; he states that that is typical for him. D) The client states she eats a low-sodium diet and reports eating processed food.

D) The client states she eats a low-sodium diet and reports eating processed food.

A patient sustains a laceration of the thigh in an industrial accident. Which step in the inflammatory process will the patient experience first? A) Cellular inflammation B) Exudate formation C) Tissue regeneration D) Vascular response

D) Vascular response

How should the nurse document high-pitehed breath sounds produced by narrowed airways? A) Rales B) Crackles C) Rhonchi D) Wheezing

D) Wheezing

A nursing instructor is guiding nursing students on best practices for interviewing patients. Which student comment would indicate the need for further instruction? A) My patient is a young adult, so I plan to talk to her without her parents in the room. B)Because my patient is old enough to be my grandfather, I will call him Mr C) When reading my patient's health record, I thought of a few questions to ask D) When I give my patient his pain medication, I will have time to ask questions.

D) When I give my patient his pain medication, I will have time to ask questions.

A patient with a colostomy complains to the nurse, "I am noticing really bad odors coming from my pouch." To help control odor, which foods should the nurse advise the patient to consume? A) White rice and toast B) Tomatoes and dried fruit C) Asparagus and melons D) Yogurt and parsley

D) Yogurt and parsley

A nursing instructor asked a group of nursing students to discuss experiences with charting assessment data. Which comment by the student indicates the need for further teaching? A) I find it difficult to avoid using phrases like, the patient tolerated the procedure well. B) It's confusing to have to remember which abbreviations this hospital allow C) I need to work on documenting assessments and interventions right after they are done D)My patient was really quiet and didn't say much, so I documented that he acted depressed

D)My patient was really quiet and didn't say much, so I documented that he acted depressed

A client with respiratory failure is experiencing cyanosis and labored breathing. What action should the nurse take first? A) Study the discharge plan. B) Check the graphic data for vital signs. C) Examine the history and physical. D) Look for an advance directive.

D, Look for an advance directive.

A client asks why a pulse oximeter reading is being done when experiencing a problem with the heart. What should the nurse respond to this client? A. ) "You might not have a heart problem." B.) "I'm just checking it as a routine part of vital signs." C. ) "Heart problems are caused by breathing problems" D. ) "The heart and lungs work together to provide oxygenation." circulation

D. ) "The heart and lungs work together to provide oxygenation." circulation

What is the most effective action by the nurse when delivering spiritual care to a patient of the same religion as the nurse? A. Understanding that the patient shares the same beliefs B. Striving to meet the patient's spiritual needs independently C. Explaining her own religious beliefs to the patient D. Developing a greater awareness of her own spirituality

D. Developing a greater awareness of her own spirituality

For which patient would it be most important to obtain an apical-radial pulse and calculate the pulse deficit? A) Recovering from abdominal surgery 2 hours ago B) Experienced a fractured hip yesterday C) Dehydrated from vomiting D) History of heart and lung disease

D. History of heart and lung disease

Of the following recommended interviewing techniques, which one is the most basic? A) Beginning with neutral topics B) Individualizing your approach C) Minimizing note taking D) Using active listening

D. using active listening

Which is an abnormal capillary refill finding that the nurse should report ? A.) 1 second B.) 2 seconds C.) 3 seconds D.) 4 seconds

D.) 4 seconds

Which differentiates a nursing diagnosis from a medical diagnosis? A.) Terminology for the client's disease or injury B. ) A part of the client's medical diagnosis C. ) The client's presenting signs and symptoms D.) A client's response to a health problem

D.) A client's response to a health problem

A new graduate asks the manager to explain the purpose of full-spectrum nursing. How should the manager respond? A.) Provide care in the most cost-effective manner possible. B.) Minimize the use of overtime when providing client care. C.) Adhere to federal and state regulations when providing care. D.) Achieve safe, effective care and promote good patient outcomes.

D.) Achieve safe, effective care and promote good patient outcomes.

The nurse is reviewing new postoperative orders and notes that the order reads, "Give hydroxyzine (Vistaril) 50 mg IV PRN nausea or vomiting." The patient is complaining of slight nausea, Which action by the nurse is correct at this time? A.) . Hold the dose until the patient complains of severe nausea. B.) . Give the dose orally instead of intravenously. C.) Give the patient the IV dose of hydroxyzine as ordered. D.) Call the prescriber to question the route that is ordered.

D.) Call the prescriber to question the route that is ordered.

Which blood level normally provided the primary stimulus for breathing? A.)PH B.) Oxygen C.) Bicarbonate D.) Carbon dioxide

D.) Carbon dioxide

A client with a closed head injury has a respiratory pattern that progressively increases and then decreases in depth, followed by a period of apnea. What this client's breathing pattern? A.) Biot's breathing B.) Kussmaul's respirations C.) Sleep apnea D.) Cheyne-Strokes respirations

D.) Cheyne-Stokes repirations

A client has lower extremity edema. Which intervention should the nurse implement to help with this client's venous return? A.) Increase the amount of daily exercise. B.) Wear well-fitting shoes with dry socks. C.) Adhere to prescribed medication regimen. D.) Elevate the legs above the level of the heart,

D.) Elevate the legs above the level of the heart,

A client asks the nurse to explain laboratory results; however, the nurse needs to assist another client first. A short while later, the nurse returns to discuss the laboratory results. Which more principle did the nurse demonstrate? A.) Nonmaleficence B.) Autonomy C.) Beneficence D.) Fidelity

D.) Fidelity

A patient is taking intravenous aminophylline for a severe exacerbation of chronic obstructive pulmonary disease. The nurse will assess for which therapeutic response? A.) Increased sputum production B.) Increased heart rate C.) Increased respiratory D.) Increased ease of breathing

D.) Increased ease of breathing

The nurse is providing instructions about the Advair inhaler (fluticasone propionate salmeterol), Which statement about this inhaler is accurate? A.) It is indicated for the treatment of acute bronchospasms. B.) It needs to be used with a spacer for best results. C.) Patients need to avoid drinking water for I hour after taking this drug. D.) It is used for the prevention of bronchospasms.

D.) It is used for the prevention of bronchospasms.

While assessing an older adult patient, the nurse notes clubbing of the fingers. What does this finding indicate to the nurse? A) Fungal infection B) Malnutrition C) Iron deficiency D) Long-term hypoxia

D.) Long-term hypoxia

The nurse administering pain medication every 4 hours is an example of which aspect of patient care? A.) Assessment data B.) Nursing diagnosis C.) Patient outcome D.) Nursing intervention

D.) Nursing intervention

When educating a patient recently placed on inhaled corticosteroids, the nurse will discuss which potential adverse effects? A.) Fatigue and depression B.) Anxiety and palpitations C.) Headache and rapid heart rate D.) Oral candidiasis and dry mouth

D.) Oral candidiasis and dry mouth

The nurse will instruct patients about a passible systemic effect that may occur if excessive amounts of topically applied adrenergic nasal decongestants are used. Which systemic effect may occur? A.) Heartburn B.) Bradycardia C.) Drowsiness D.) Palpitations

D.) Palpitations

While a patient is receiving hygiene care, the chest tube becomes disconnected from the water-seal chest drainage system (CDU). Which action should the nurse take immediately? A.)Clamp the chest tube close to the insertion site. B.) Set up a new drainage system and connect it to the chest tube. C.) Have the patient take and hold a deep breath while the nurse reconnects the tube to the CDU. D.) Place the disconnected end nearest the patient into a bottle of sterile water.

D.) Place the disconnected end nearest the patient into a bottle of sterile water.

A patient is about to undergo a diagnostic bowel procedure. The nurse expects which drug to be used to induce total cleansing of the bowel? A.) Docusate sodium (Colace) B.) Lactulose (Enulose) C.) Mineral oil D.) Polyethylene glycol 3350 (GOLYTELY)

D.) Polyethylene glycol 3350 (GOLYTELY)

A patient who has been on antibiotic therapy for 2 weeks has developed persistent diarrhea. The nurse expects which medication class to be ordered to treat this diarrhea? A.) Lubricants B.) Adsorbents C.) Anticholinergics D.) Probiotics

D.) Probiotics

The nurse administers intravenous morphine sulfate to a patient for pain control. For which adverse effect should the nurse monitor this patient ? A.) Decreased heart rate B.) Muscle weakness C.) Decreased urine output D.) Respiratory depression

D.) Respiratory depression

A client has a nursing diagnosis of Ineffective Breathing Pattern identified on the care plan, What should the nurse expect when assessing this client? A.) O Coughing B.) Cold extremities C.) Adventitious breath sounds D.) Respiratory rate of 8 breaths/min

D.) Respiratory rate of 8 breaths/min

The nurse plans to use utilitarianism to address an ethical problem. What analysis will the nurse need to complete when using this framework? A.) Cost B.) SWOT C.) Root cause D.) Risk-benefit

D.) Risk-benefit

The nurse attends an ethics committee to discuss an issue with a client. What should the nurse keep in mind when advocating for the client? A.) Expect confrontation. B.) Prepare for a power struggle. C.) State personal feelings clearly. D.) Support the client's autonomy

D.) Support the client's autonomy

Which skin assessment finding would cause the nurse to suspect dehydration in a middle- aged patient admitted to the hospital with traveler's diarrhea ? A.) Edema B.) Hyperhidrosis C.) Pallor D.) Tenting

D.) Tenting

Which term is defined as a belief about the worth of something that serves as a principle or a standard that influences decision making? A.) Morals B.) Attitudes C.) Beliefs D.) Values

D.) Values

A patient with chronic obstructive pulmonary disease (COPD) is prescribed 02 at 24% FIO2. What is the most appropriate oxygen delivery method for this patient? A.) Nonrebreather mask B.) Nasal cannula C.) Partial rebreather mask D.) Venturi Mask

D.) Venturi Mask

The nurse is teaching a group of patients about self-administration of insulin. What content is important to include? A. Patients need to use the injection site that is the most accessible. B. If two different insulins are ordered, they need to be given in separate injections. C. When mixing insulins, the cloudy (such as NPH) insulin is drawn up into the syringe first. D.When mixing insulins, the clear (such as regular) is drawn up in the syringe first.

D.When mixing insulins, the clear (such as regular) is drawn up in the syringe first.

When administering nasal spray , which instruction by the nurse is appropriate ? a . " You will need to blow your nose before I give this medication . " b . " You will need to blow your nose after I give this medication . " c . " When I give this medication , you will need to hold your breath . " d . " You need to sit up for 5 minutes after you receive the nasal spray . "

a . " You will need to blow your nose before I give this medication . "

While the nurse is assisting a patient in taking his medications , the medication cup falls to the floor , spilling the tablets . What is the nurse's best action at this time ? a . Discarding the medications and repeating preparation b . Asking the patient if he will take the medications c . Waiting until the next dose time , and then giving the medications d . Retrieving the medications and administering them to avoid waste

a . Discarding the medications and repeating preparation

The nurse is administering an IV push medication through an IV lock . After injecting the medication , which action will be taken next ? a . Flushing the lock b . Regulating the IV flow c . Clamping the tubing for 10 minutes d . Holding the patient's arm up to improve blood flow

a . Flushing the lock

The nurse will plan to use the Z - track method of intramuscular ( IM ) injections for which situation ? a . The medication is known to be irritating to tissues . b . The patient is emaciated and has very little muscle mass . c . The medication must be absorbed quickly into the tissues . d . The patient is obese and has a deep fat layer below the muscle mass .

a . The medication is known to be irritating to tissues .

The nurse is performing an admission assessment . Which findings reflect components of a cultural assessment ?( Select all that apply . ) a . The patient uses aspirin as needed for pain . b . The patient has a history of hypertension . c . The patient uses herbal tea to relax in the evenings . d . The patient does not speak English . e . The patient is allergic to shellfish . f . The patient does not eat pork products for religious reasons .

a . The patient uses aspirin as needed for pain c . The patient uses herbal tea to relax in the evenings d . The patient does not speak English f . The patient does not eat pork products for religious reasons .

An adult patient is prescribed a unit of packed red blood cells. Which gauge needle should be inserted to administer this blood product? a) 18 gauge b) 22 gauge c) 24 gauge d) 26 gauge

a) 18 gauge

Which documentation entry related to PRN medication administration is complete? a) 6/5/14 0900 morphine 4 mg IV given in right antecubetal fossa for pain rated 8 on a 1-10 scale, J. Williams RN b) 0600 famotidine 20 mg IV given in right hand, S. Abraham RN c) 9/2/14 0900 levothyroxine 50 mcg PO given d) 1/16/14 furosemide 40 mg PO given, J. Smith RN

a) 6/5/14 0900 morphine 4 mg IV given in right antecubetal fossa for pain rated 8 on a 1-10 scale, J. Williams RN

Laboratory test results indicate that warfarin anticoagulant therapy is suddenly ineffective in a patient who has been taking the drug for an extended time. The nurse suspects an interaction with herbal medications. What type of interaction does the nurse suspect? a) Antagonistic drug interaction b) Synergistic drug interaction c) Idiosyncratic reaction d) Drug incompatibility

a) Antagonistic drug interaction

The nurse notes that the spouse of a terminally ill client stops visiting every day and talks about meeting with friends after work. Which type of grief is the spouse demonstrating? a) Anticipatory b) Complicated c) Uncomplicated d) Disenfranchised

a) Anticipatory

The nursing instructor provides a program on the state board of nursing for students preparing to graduate from a school of nursing. What should the instructor emphasize about the role of the state boards of nursing? Select all that apply. a) Approve nursing education programs b) Define the scope of practice for nursing c) Develop rules and regulations for nursing practice d) Provide guidance on knowledge, skills, and attitudes e) Publish statements of duties for competent performance

a) Approve nursing education programs b) Define the scope of practice for nursing c) Develop rules and regulations for nursing practice

A patient of Scandinavian heritage is admitted for observation after sustaining injuries in a motor vehicle accident. The nurse expects that he may endure pain stoically, without grimacing or vocalizing. Of what is the example? a) Archetype b) Bias c) Prejudice d) Stereotype

a) Archetype

The spouse of a recently deceased client is having difficulty with grieving. What action should the nurse implement to help the spouse through the process? Select all that apply. a) Ask questions about the deceased person's hobbies and activities. b) Encourage the use of poems and stories about the deceased person. c) Encourage questions and respond to them within a reasonable period of time. d) Recommend removing the deceased person's clothing and personal items. e) Go through photo albums and ask questions about the people in the pictures.

a) Ask questions about the deceased person's hobbies and activities. b) Encourage the use of poems and stories about the deceased person. c) Encourage questions and respond to them within a reasonable period of time. e) Go through photo albums and ask questions about the people in the pictures.

Which dysrhythmia confirms death? a) Asystole b) Pulseless electrical activity c) Ventricular fibrillation d) Ventricular tachycardia

a) Asystole

Which are cues rather than inferences? Select all that apply. A) Ate 50% of his meal B) Patient feels better today C) States, "I slept well" D) White blood cell count 15,000/mm3 E) Blood pressure 120/80 mm Hg

a) Ate 50% of his meal c) States, "I slept well" d) White blood cell count 15,000/mm3 e) Blood pressure 120/80 mm Hg

A nurse observes a nursing assistive personnel (NAP) fail to wash her hands before and after placing a patient on a bedpan. What should the nurse do when providing negative feedback to the NAP? a) Be certain to offer constructive criticism about the task and do so in private. b) Ask the unit manager to be present to document responses of both parties. c) Call a meeting of all NAPs and stress hand washing to the entire group. d) Keep a record of the NAP's actions and save them for her annual formal review.

a) Be certain to offer constructive criticism about the task and do so in private.

A new graduate nurse is having difficulty understanding the importance of caring when implementing full-spectrum nursing. What should the nurse manager explain to this nurse? Select all that apply. A) Caring motivates and facilitates doing. B) Caring ensures positive client outcomes. C) Caring ensures adherence to the care plan. D) Caring motivates and facilitates thinking. E) Caring replaces steps in the nursing process.

a) Caring motivates and facilitates doing. d) Caring motivates and facilitates thinking.

Which is a characteristic of an effective nurse manager? Select all that apply. a) Clinical expertise b) Business sense c) Master's degree d) Leadership skills e) Worked as a staff nurse on the care area

a) Clinical expertise b) Business sense d) Leadership skills

An experienced nurse serves as a mentor to a new graduate. Which is a responsibility of the person being mentored? Select all that apply. a) Demonstrates an ability to move toward independence b) Has the ability to encourage excellence in others c) Seeks feedback and uses it to modify behaviors d) Demonstrates flexibility and an ability to change e) Demonstrates trustworthiness

a) Demonstrates an ability to move toward independence c) Seeks feedback and uses it to modify behaviors d) Demonstrates flexibility and an ability to change

The nurse manager identifies several staff as " good followers". Which statement best describes these nurses' behavior? Select all that apply. a) Demonstrates teamwork b) Acts in accordance with another c) Strives to prevent group cohesion d) Takes another person as a role model e) Works with others to accomplish a group mission

a) Demonstrates teamwork b) Acts in accordance with another d) Takes another person as a role model e) Works with others to accomplish a group mission

The staff development instructor plans a training session on full-spectrum nursing. Which concept should the instructor include in this information? Select all that apply. A) Doing B) Caring C) Thinking D) Collaboration E) Patient situation

a) Doing b) Caring c) Thinking e) Patient situation

An older client recovering from a stroke in an acute care facility is transferred to a nursing home for ongoing care and the inability to provide self-care at home. Which type of loss is this patient most likely experiencing? a) Environmental loss b) Internal loss c) Perceived loss d) Psychological loss

a) Environmental loss

The nurse prepares a teaching session on spirituality for the families of clients receiving hospice care. What core issues should the nurse include in this teaching? Select all that apply. a) Faith b) Hope c) Prayer d) Forgiveness e) Compassionate love

a) Faith b) Hope e) Compassionate love

The surgical unit is experiencing difficulty recruiting new RNs, although the hospital has an excellent reputation in the community and has no difficulty recruiting nurses for other units. A task force has been formed, consisting of one nurse from each shift on the unit, the unit manager, and the hospital nurse recruiter. The group has gathered data and identified the problem. What is the next step in this process? a) Generate possible solutions. b) Evaluate whether the problem has been resolved. c) Implement the solution changes. d) Evaluate suggested solutions.

a) Generate possible solutions.

A nurse with 2 years' experience frequently appears stressed, has difficulty completing work. and clocks out late even when the assignment is light. The charge nurse describes the nurse as" running from one duty to the next and having no organization or daily routine. Which situation most likely describes this nurse? a) Has time management problems b) Has a heavy patient load c) Works at a hospital that is understaffed d) Is in a management position

a) Has time management problems

Which are examples of a health-promotion activity? Select all that apply. A) Helping a client develop a plan for a low-fat, low-cholesterol diet B) Disinfecting an abraded knee after a child falls off a bicycle C) Administering a tetanus vaccination after an injury from a car accident D) Distributing educational brochures about the benefits of exercise E) Administering a measles, mumps, rubella (MMR) immunization to a toddler at 15 months of age

a) Helping a client develop a plan for a low-fat, low-cholesterol diet d) Distributing educational brochures about the benefits of exercise e) Administering a measles, mumps, rubella (MMR) immunization to a toddler at 15 months of age

A patient is in the bathroom and asks the nurse to leave medications on the bedside table. What should the nurse do? a) Inform the patient that she will return when he is finished in the bathroom. b) Wait outside the bathroom door until the patient is ready for the dose. c) Withhold the dose until the next administration time later in the day. d) Document that the dose was omitted in the medication administration record.

a) Inform the patient that she will return when he is finished in the bathroom.

A client asks the nurse to explain what is meant by " being spiritual" What should the nurse respond? Select all that apply. a) It provides insight into the meaning of life. b) It provides an opportunity to transcend the physical world. c) It involves personal subjective experiences that take place over time. d) It provides codes of conduct that integrate values into a way of living. e) It results from life events that cause internal upheaval, struggle, and challenge.

a) It provides insight into the meaning of life. b) It provides an opportunity to transcend the physical world. c) It involves personal subjective experiences that take place over time. e) It results from life events that cause internal upheaval, struggle, and challenge.

The nurse manager prepares information on the regulations of nursing practice for a group of new nursing employees. Which information should the manager include in this presentation? Select all that apply. A) Laws B) Standards of practice C) Facility policies and procedures D) The Joint Commission regulations E) Professional organization guidelines

a) Laws b) Standards of practice e) Professional organization guidelines

For a patient to be eligible for insurance benefits that cover hospice care, a physician must certify which of the criteria as applying to the patient? Select all that apply. a) Life expectancy is not more than 6 months. b) Life expectancy is not more than 12 months. c) Condition is expected to improve slightly. d) Condition is not expected to improve. e) An advance directive has been written.

a) Life expectancy is not more than 6 month d) Condition is not expected to improve.

The nurse is beginning a new position as a manager. Which is an informational responsibility of this nurse's new role? Select all that apply. a) Monitor b) Advocacy c) Spokesperson d) Public relations e) Employee development

a) Monitor c) Spokesperson d) Public relations

The nurse thanks a nursing assistive personnel (NAP) for helping with client care and praises the NAP's patience with a challenging client. What is the possible outcome from the feedback provided to the NAP? a) Motivation b) Time management c) Improved skill level d) Improved resource use

a) Motivation

How should the nurse dispose of a contaminated needle after administering an injection? a) Place the needle in a specially marked, puncture-proof container. b) Recap the needle, and carefully place it in the trash can. c) Recap the needle, and place it in a puncture-proof container. d) Place the needle in a biohazard bag with other contaminated supplies.

a) Place the needle in a specially marked, puncture-proof container

The nurse notes that a client whose spouse recently died is managing grief and planning for the future. Which factor is affecting the client's grief? Select all that apply. a) Previous losses b) Financial status c) Support systems d) Significance of the loss e) Circumstances of the loss

a) Previous losses c) Support systems d) Significance of the loss e) Circumstances of the loss

The nurse must administer eardrops to an infant. How should this medication be given? a) Pull the pinna down and back before instilling the drops. b) Pull the pinna upward and outward before instilling the drops. c) Instill the drops directly; no special positioning is necessary. d) Position the patient supine with the head of the bed elevated 30°.

a) Pull the pinna down and back before instilling the drops.

A preceptor researches information to best explain the concept of assessment to a new graduate nurse. What should the preceptor include when defining assessment? Select all that apply. A) Recording data B) Collecting data C) Categorizing data D) Identifying outcomes E) Using a systematic process

a) Recording data b) Collecting data c) Categorizing data e) Using a systematic process

A client with a gynecological problem believes she is being punished for having had an abortion as a teenager. What should the nurse do to assist with this spiritual needs? a) Refer her to the hospital chaplain. b) Provide teaching material about the health problem. c) Explain that it is unlikely that the abortion caused the health problem. d) Suggest that the client forgive herself for an action that occurred when she was a teenager.

a) Refer her to the hospital chaplain

The healthcare provider telephones the care area to provide a verbal medication prescription for a client. What action should the nurse take? Select all that apply. a) Repeat the prescription to the provider. b) Tell the provider that verbal orders are not permitted. c) Ask the unit secretary to write the prescription in the medical record. d) Remind the provider that verbal orders need to be signed within 1 week. e) Sign the prescription with the provider's name followed by the nurse's name.

a) Repeat the prescription to the provider. e) Sign the prescription with the provider's name followed by the nurse's name.

The nurse manager is in the midst of creating a budget for the next year. Which economic information should the nurse keep in mind when preparing this budget? a) Resources are scarce. b) The need for nurses will increase. c) The number of vacant positions for nurses will grow. d) There is an increased need for advanced practice nurses.

a) Resources are scarce.

The nurse registers to attend an in-service education program. What should the nurse expect this program to provide? Select all that apply a. review of new policies b. introduction to new equipment c. credit toward a professional nursing degree d. continuing education hours for license renewal e. enhancement of competency in knowledge, skills, and attitudes

a) Review of new policies b) Introduction to new equipment e) Enhancement of competency in knowledge, skills, and attitudes

The nurse manager is reviewing information about power to include in a staff presentation on empowerment. Which type of power does the manager possess? a) Reward b) Referent c) Expertise d) Legitimate

a) Reward

The nurse decides to create a computerized care plan for a client. What should the nurse keep in mind when creating this plan of care? Select all that apply. A) Select outcomes for the client. B) Determine indicators for the client. C) Identify the target time for each outcome. D) Permit the computer to make client care decisions. E) Ensure protocols are included for basic care needs.

a) Select outcomes for the client. b) Determine indicators for the client. c) Identify the target time for each outcome.

The nurse reviews advance directives with a client. Which statement about an advance directive document indicates that teaching has been effective? a) Specifies healthcare intentions if unable to make self-directed decisions b) Identifies the activities considered to be evidence of quality care c) Verifies understanding of the risks and benefits associated with a procedure d) Allows the autonomy to leave the hospital even if it is against medical advice

a) Specifies healthcare intentions if unable to make self-directed decisions

A patient tells the nurse, " I feel that God has abandoned me. I am so angry that I can't even pray. The patient refuses to see his clergyman when he calls. Which is the most appropriate nursing diagnosis for this patient? a) Spiritual Distress b) Risk for Spiritual Distress c) Impaired Religiosity d) Moral Distress

a) Spiritual Distress

Which activity is involved in delegating tasks to other members of the nursing team? Select all that apply. a) Supervising patient care that is given b) Determining the skill mix of unit personnel c) Assessing the needs of the clients involved d) Deciding which tasks to assign to a team member e) Completing evaluations of staff

a) Supervising patient care that is given b) Determining the skill mix of unit personnel c) Assessing the needs of the clients involved d) Deciding which tasks to assign to a team member

The nurse cares for a newly admitted client. What will the nurse include when creating a comprehensive plan of care for this client? Select all that apply. A) Teaching needs B) Basic care needs C) Medical treatment D) Nursing diagnoses E) Insurance coverage

a) Teaching needs b) Basic care needs c) Medical treatment d) Nursing diagnoses

When performing a hand-off report, the nurse should communicate information on which aspect of care? Select all that apply. A) Teaching performed B) Any change in client status C) Treatments administered D) Hygiene measures performed E) Routine care provided

a) Teaching performed b) Any change in client status c) Treatments administered

The nurse prepares a comprehensive plan of care for a new admitted client. For which reason should the nurse include protocols within the plan of care? Select all that apply. A) The client needs a specific treatment. B) The client is prescribed a particular diet. C) The client has a particular medical diagnosis. D) The client needs assistance with routine activities. E) The client is scheduled for a specific diagnostic test.

a) The client needs a specific treatment. c) The client has a particular medical diagnosis. e) The client is scheduled for a specific diagnostic test.

The nurse identifies a collaborative problem for a client. What should the nurse recall when creating the nursing diagnosis statement for this client problem? Select all that apply. A) The problem should be a potential problem. B) The disease, test, or treatment is the etiology. C) The etiology can be treated with independent nursing interventions. D) The focus for the interventions is to monitor and prevent complications. E) The second part of the statement is the problem that is being monitored.

a) The problem should be a potential problem. b) The disease, test, or treatment is the etiology. d) The focus for the interventions is to monitor and prevent complications. e) The second part of the statement is the problem that is being monitored.

The nurse notes that a client selected Christian Scientist as the primary religion. What should the nurse consider as possible beliefs for this client? Select all that apply. a) Tobacco and alcohol are avoided. b) Illness is caused by faulty thinking. c) Medical care is approved for children. d) Blood transfusions are not accepted by adults. e) The Sacrament of Reconciliation is an important facet of the religion.

a) Tobacco and alcohol are avoided. b) Illness is caused by faulty thinking. c) Medical care is approved for children. d) Blood transfusions are not accepted by adults.

The nurse manager notes that a staff nurse rarely addresses a client's spiritual needs. When talking with the nurse, what should the manager identify as barriers to spiritual care? Select all that apply. a) Trying to be all things to all people b) Fear of an insufficient knowledge base c) Lack of general awareness of spirituality d) Difference in religion between the client and the nurse e) Differences in spirituality between the client and the nurse

a) Trying to be all things to all people b) Fear of an insufficient knowledge base c) Lack of general awareness of spirituality e) Differences in spirituality between the client and the nurse

The nurse reviews a medical record and notes that a terminally ill client has higher-brain death. What should the nurse expect when assessing this client? a) Unconsciousness b) Irregular heartbeat c) Altered respiratory rate d) Absent deep tendon reflexes

a) Unconsciousness

According to William Worden, which task in the grieving process takes longest to achieve? a) Working through the pain and grief b) Accepting the reality of the loss c) Adjusting to the environment without the deceased d) Investing emotional energy

a) Working through the pain and grief

The nurse notes that a healthcare provider speaks with the same tone to every client about every situation. What characteristics of verbal communication should the nurse keep in mind when talking with a client? Select all that apply. A) Pitch B) Timing C) Brevity D) Volume E) Cadence

a) pitch b) timing e) cadence

When creating nursing diagnoses for a client, the nurse spends considerable time on identifying the etiology. Which category of etiology should the nurse consider when writing nursing diagnoses? Select all that apply. A) Social B) Economic C) Situational D) Treatment related E) Pathophysiological

a) social c) situational d) treatment related e) pathophysiological

The nurse is providing teaching to a patient taking an oral tetracycline antibiotic. Which statement by the nurse is correct? a. "Avoid direçt sunlight and tanning beds while on this medication.". b. "Milk and cheese products result in increased levels of tetracycline." c. "Antacids taken with the medication help to reduce gastrointestinal distress." d. "Take the medication until you are feeling better."

a. "Avoid direçt sunlight and tanning beds while on this medication.".

A patient who is diagnosed with genital herpes is taking topical acyclovir. The nurse will provide which teaching for this patient? (Select all that apply.) a. "Be sure to wash your hands thoroughly before and after applying this medicine." b. "Apply this ointment until the lesion stops hurting." c. "Use a clean glove when applying this ointment," d. "If your partner develops these lesions, then he can also use the medication." e. "You will need to avoid touching the area around your eyes." f. "You will have to practice abstinence when these lesions are active."

a. "Be sure to wash your hands thoroughly before and after applying this medicine." c. "Use a clean glove when applying this ointment," e. "You will need to avoid touching the area around your eyes." f. "You will have to practice abstinence when these lesions are active."

A 30-year-old woman is in the clinic for her yearly gynecologic exam and asks the nurse about the "new vaccine that prevents HPV." She wants to receive the papillomavirus vaccine (Gardasil). Which response by the nurse is most appropriate? a. "For women, the recommended age for this vaccine is 13 to 26 years of age." b. "We will need to make sure you are not pregnant first." c. "There will be a total of three injections." d. "I will check with your health care provider and then get the first dose of the vaccine ready."

a. "For women, the recommended age for this vaccine is 13 to 26 years of age."

The patient is to receive oral guaifenesin (Mucinex) twice a day. Today, the nurse was busy and gave the medication 2 hours after the scheduled dose was due. What type of problem does this represent? a. "Right time" b. "Right dose" c. "Right route" d. "Right medication"

a. "Right time"

A 75-year-old woman with type 2 diabetes has recently been placed on glipizide (Glucotrol), 10 mg daily. She asks the nurse when the best time would be to take this medication. What is the nurse's best response? a. "Take this medication in the morning, 30 minutes before breakfast." b. "Take this medication in the evening with a snack." c. "This medication needs to be taken after the midday meal." d. "It does not matter what time of day you take this medication."

a. "Take this medication in the morning, 30 minutes before breakfast."

A patient is receiving aminoglycoside therapy and will be receiving a beta-lactam antibiotic as well. The patient asks why two antibiotics have been ordered. What is the nurse's best response? a. "The combined effect of both antibiotics is greater than each of them alone." b. "One antibiotic is not strong enough to fight the infection." c. "We have not yet isolated the bacteria, so the two antibiotics are given to a wide range of microorganisms." d. "We can give a reduced amount of each one if we give them together."

a. "The combined effect of both antibiotics is greater than each of them alone."

The nurse is administering insulin lispro (Humalog) and will keep in mind that this insulin will to have an effect within which time frame? a. 15 minutes b. 1 to 2 hours c. 80 minutes d. 3 to 5 hours

a. 15 minutes

During the development of a new drug, which would be included in the study by the researcher to prevent any bias or unrealistic expectations of the new drug's usefulness? a. A placebo b. FDA approval c. Informed consent d. Safety information

a. A placebo

A patient is undergoing major surgery and asks the nurse about a living will. He states, "I don't want anybody else making decisions for me. And I don't want to prolong my life." The patient is demonstrating which ethical term? a. Autonomy b. Beneficence c. Justice d. Veracity

a. Autonomy

1. The patient is receiving two different drugs. At current dosages and dosage forms, both drugs are absorbed into the circulation in identical amounts. Which term is used to identify this principle? a. Bioequivalent b. Synergistic c. Prodrugs d. Steady state

a. Bioequivalent

During an intravenous (IV) infusion of amphotericin B, a patient develops tingling and numbness in his toes and fingers. What will the nurse do first? a. Discontinue the infusion immediately. b. Reduce the infusion rate gradually until the adverse effects subside. c. Administer the medication by rapid IV infusion to reduce these effects. d. Nothing; these are expected side effects of this medication.

a. Discontinue the infusion immediately.

When reviewing the various schedules of controlled drugs, the nurse knows that which description correctly describes Schedule II drugs? a. Drugs with high potential for abuse that have accepted medical use b. Drugs with high potential for abuse that do not have accepted medical use c. Medically accepted drugs that may cause moderate physical or psychologic dependence d. Medically accepted drugs with limited potential for causing physical or psychologic dependence

a. Drugs with high potential for abuse that have accepted medical use

The nurse is reviewing the sputum culture results of a patient with pneumonia and notes that the patient has a gram-positive infection. Which generation of cephalosporin is most appropriate for this type of infection? a. First generation b. Second generation c. Third generation d. Fourth generation

a. First generation

Which action is most appropriate regarding the nurse's administration of a rapid-acting insulin to a hospitalized patient? a. Give it within 15 minutes of mealtime. b. Give it after the meal has been completed. c. Administer it once daily at the time of the midday meal. d. Administer it with a snack before bedtime.

a. Give it within 15 minutes of mealtime.

The nurse is administering intravenous acyclovir (Zovirax) to a patient with a viral infection. Which administration technique is correct? a. Infuse intravenous acyclovir slowly, over at least 1 hour. b. Infuse intravenous acyclovir by rapid bolus. c. Refrigerate intravenous acyclovir. d. Restrict oral fluids during intravenous acyclovir therapy.

a. Infuse intravenous acyclovir slowly, over at least 1 hour.

The nurse is reviewing the information about the herpes zoster vaccine (Zostavax) before administering the dose. Which statements about the vaccine are true? (Select all that apply.) a. It is a one-time vaccine. b. The vaccine is recommended for patients 50 years of age and older. c. The vaccine is given to children to prevent chickenpox. d. It is used to prevent postherpetic neuralgia. e. It is contraindicated in patients who have already had shingles. f. The vaccine is used to prevent reactivation of the zoster virus that causes shingles.

a. It is a one-time vaccine. b. The vaccine is recommended for patients 50 years of age and older. f. The vaccine is used to prevent reactivation of the zoster virus that causes shingles.

A patient is in the HIV clinic for a follow-up appointment. He has been on antiretroviral therapy for HIV for more than 3 years. The nurse will assess for which potential adverse effects of long- term antiretroviral therapy? (Select all that apply.) a. Lipodystrophy b. Liver damage c. Kaposi's sarcoma d. Osteoporosis e. Type 2 diabetes

a. Lipodystrophy b. Liver damage d. Osteoporosis e. Type 2 diabetes

A patient will be receiving nitrofurantoin (Macrodantin) treatment for a urinary tract infection. The nurse is reviewing the patient's history and will question the nitrofurantoin order if which disorder is present in the history? (Select all that apply.) a. Liver disease b. Coronary artery disease с. Нурerthyroidism d. Type 1 diabetes mellitus e. Chronic renal disease

a. Liver disease e. Chronic renal disease

The nurse is administering intravenous vancomycin (Vancocin) to a patient who has had gastrointestinal surgery. Which nursing measures are appropriate? (Select all that apply.) a. Monitoring serum creatinine levels b. Restricting fluids while the patient is on this medication c. Warning the patient that a flushed feeling or facial itching may occur d. Instructing the patient to report dizziness or a feeling of fullness in the ears e. Reporting a trough drug level of11 meg/mL and holding the drug f. Reporting a trough drug level of 24 mcg/mL and holding the drug

a. Monitoring serum creatinine levels c. Warning the patient that a flushed feeling or facial itching may occur d. Instructing the patient to report dizziness or a feeling of fullness in the ears e. Reporting a trough drug level of11 meg/mL and holding the drug

A patient who has advanced cancer is receiving opioid medications around the clock to keep him comfortable as he nears the end of his life. Which term best describes this type of therapy? a. Palliative therapy b. Maintenance therapy c. Empiric therapy d. Supplemental therapy

a. Palliative therapy

The nurse is administering parenteral drugs. Which statement is true regarding parenteral drugs? a. Parenteral drugs bypass the first-pass effect. b. Absorption of parenteral drugs is affected by reduced blood flow to the stomach. c. Absorption of parenteral drugs is faster when the stomach is empty. d. Parenteral drugs exert their effects while circulating in the bloodstream.

a. Parenteral drugs bypass the first-pass effect.

The nurse is reviewing the health history of a new patient who may need immunizations. Active immunizations are usually contraindicated in which patients? (Select all that apply.) a. Patients with a febrile illness b. Children younger than 1 year of age c. Elderly patients d. Patients who are immunosuppressed e. Those receiving cancer chemotherapy

a. Patients with a febrile illness d. Patients who are immunosuppressed e. Those receiving cancer chemotherapy

A member of an investigational drug study team is working with healthy volunteers whose participation will help to determine the optimal dosage range and pharmacokinetics of the drug. The team member is participating in what type TESTBANK of study? a. Phase I b. Phase II c. Phase III d. Phase IV

a. Phase I

Place the phases of the nursing process in the correct order, with 1 as the first phase and 5 as the last phase. (Select all that apply.) a. Planning b. Evaluation c. Assessment d. Implementation e. Nursing Diagnoses

a. Planning b. Evaluation c. Assessment d. Implementation e. Nursing Diagnoses

Which activity best reflects the implementation phase of the nursing process for the patient who is newly diagnosed with hypertension? a. Providing education on keeping a journal of blood pressure readings b. Setting goals and outcome criteria with the patient's input c. Recording a drug history regarding over-the-counter medications used at home d. Formulating nursing diagnoses regarding deficient knowledge related to the new treatment regimen

a. Providing education on keeping a journal of blood pressure readings

When giving medications, the nurse will follow the rights of medication administration. The rights include the right documentation, the right reason, the right response, and the patient's right to refuse. Which of these are additional rights? (Select all that apply.) a. Right drug b. Right route c. Right dose d. Right diagnosis e. Right time f. Right patient

a. Right drug b. Right route c. Right dose e. Right time f. Right patient

During therapy with amphotericin B, the nurse will monitor the patient for known adverse effects that would be reflected by which laboratory result? a. Serum potassium level of 2.7 mEq/L b. Serum potassium level of 5.8 mEq/L c. White blood cell count of 7000 cells/mm d. Platelet count of 300,000/ microliter

a. Serum potassium level of 2.7 mEq/L

A 45-year-old man has received a series of immunizing drugs in preparation for a trip to a developing country. Within hours, his wife brings him to the emergency department because he has developed edema of the face, tongue, and throat and is having trouble breathing. The nurse suspects that, based on the patient's history and symptoms, he is experiencing which condition? a. Serum sickness b. Cross-sensitivity c. Thrombocytopenic purpura d. Adenopathy

a. Serum sickness

When reviewing the medication orders for a patient who is taking penicillin, the nurse notes that the patient is also taking the oral anticoagulant warfarin (Coumadin). What possible effect may occur as the result of an interaction between these drugs? a. The penicillin will cause an enhanced anticoagulant effect of the warfarin. b. The penicillin will cause the anticoagulant effect of the warfarin to decrease. c. The warfarin will reduce the anti-infective action of the penicillin. d. The warfarin will increase the effectiveness of the penicillin.

a. The penicillin will cause an enhanced anticoagulant effect of the warfarin.

The patient is experiencing chest pain and needs to take a sublingual form of nitroglycerin. Where does the nurse instruct the patient to place the tablet? a. Under the tongue b. On top of the tongue c. At the back of the throat d. In the space between the cheek and the gum

a. Under the tongue

The nurse is reviewing the history of a patient who has a new order for a nonsteroidal anti- inflammatory drug (NSAID) to treat tendonitis. Which conditions are contraindications to the use of NSAIDs? (Select all that apply.) a. Vitamin K deficiency b. Arthralgia c. Peptic ulcer disease d. Neuropathy e. Pericarditis

a. Vitamin K deficiency c. Peptic ulcer disease

During antibiotic therapy, the nurse will monitor closely for signs and symptoms of a hypersensitivity reaction. Which of these assessment findings may be an indication of a hypersensitivity reaction? (Select all that apply.) a. Wheezing b. Diarrhea c. Shortness of breath d. Swelling of the tongue e. Itching f. Black, hairy tongue

a. Wheezing c. Shortness of breath d. Swelling of the tongue e. Itching

The nurse is caring for an older client of Chinese heritage. When planning outcomes for this patient, which actions by the nurse would meet the American Nurses Association standards for outcomes identification? Select all that apply. A) Developing culturally appropriate outcomes B) Using the outcomes preprinted on the clinical pathway C) Choosing the best outcome for the patient, regardless of the costs involved in bringing it about D) Involving the patient and family in formulating the outcomes E) Identifying outcomes with the least amount of nursing interventions

a. developing culturally appropriate outcomes d. involving the patient and family in formulating the outcomes

During drug therapy for pneumonia, a female patient develops a vaginal superinfection. The nurse explains that this infection is caused by: a. large doses of antibiotics that kill normal flora. b. the infection spreading from her lungs to the new site of infection. c. resistance of the pneumonia-causing bacteria to the drugs. d. an allergic reaction to the antibiotics.

a. large doses of antibiotics that kill normal flora.

The nurse is reviewing medication errors . Which situation is an example of a medication error ? a . A patient refuses her morning medications . b . A patient receives a double dose of a medication because the nurse did not cut the pill in half . c. A patient develops hives after having started an IV antibiotic 24 hours earlier . d . A patient complains of severe pain still present 60 minutes after a pain medication was given .

b . A patient receives a double dose of a medication because the nurse did not cut the pill in half .

The nurse is preparing to give an aqueous intramuscular ( IM ) injection to an average - sized adult . Which actions are appropriate ? ( Select all that apply . ) a . Choose a 26- or 27 - gauge , 1/4 - to 1/2 - inch needle . b . Choose a 22- to 27 - gauge 1- to 1½ - inch needle. c . Choose the dorsogluteal site , the preferred site for IM injections for adults . d . Insert the needle at a 45 - degree angle . e . Insert the needle at a 90 - degree angle . f . Before injecting the medication , withdraw the plunger to check for blood return .

b . Choose a 22- to 27 - gauge 1- to 1½ - inch needle. e . Insert the needle at a 90 - degree angle . f . Before injecting the medication , withdraw the plunger to check for blood return .

During a period of time when the computerized medication order system was down , the prescriber wrote admission orders , and the nurse is transcribing them . The nurse is having difficulty transcribing one order because of the prescriber's handwriting . Which is the best action for the nurse to take at this time ? a . Ask a colleague what the order says . b . Contact the prescriber to clarify the order . c . Wait until the prescriber makes rounds again to clarify the order . d . Ask the patient what medications he takes at home .

b . Contact the prescriber to clarify the order .

A patient with asthma is to begin medication therapy using a metered - dose inhaler . What is an important reminder to include during teaching sessions with the patient ? a . Repeat subsequent puffs , if ordered , after 5 minutes . b . Inhale slowly while pressing down to release the medication . c . Inhale quickly while pressing down to release the medication . d . Administer the inhaler while holding it 3 to 4 inches away from the mouth .

b . Inhale slowly while pressing down to release the medication .

The nurse is reviewing the concept of drug polymorphism . Which factors contribute to drug polymorphism ? ( Select all that apply . ) a . The number of drugs ordered by the physician b . Inherited factors c . The patient's diet and nutritional status d . Different dosage forms of the same drug e . The patient's health beliefs and practices f . The patient's drug history g . The various available forms of a drug

b . Inherited factors c . The patient's diet and nutritional status e . The patient's health beliefs and practices

During a busy night shift , a new nurse administered an unfamiliar medication without checking it in a drug handbook . Later that day , the patient had a severe reaction because he has renal problems , which was a contraindication to that drug . The nurse may be liable for which of these ? a . Medical negligence b . Nursing negligence c . Nonmaleficence d . Autonomy

b . Nursing negligence

When administering medication by IV bolus ( push ) , the nurse will occlude the IV line by which method ? a . Not pinching the IV tubing at all b . Pinching the tubing just above the injection port c . Pinching the tubing just below the injection port d . Pinching the tubing just above the drip chamber of the infusion set

b . Pinching the tubing just above the injection port

When adding medications to a bag of intravenous ( IV ) fluid , the nurse will use which method to mix the solution ? a . Shaking the bag or bottle vigorously b . Turning the bag or bottle gently from side to side c . Inverting the bag or bottle one time after injecting the medication d . Allowing the IV solution to stand for 10 minutes to enhance even distribution of medication

b . Turning the bag or bottle gently from side to side

1. The nurse can prevent medication errors by following which principles ? ( Select all that apply . ) a . Assess for allergies after giving medications . b . Use two patient identifiers before giving medications . c. Do not give a medication that another nurse has drawn up in a syringe . d . Minimize the use of verbal and telephone orders . e . Use trade names instead of generic names to avoid confusion .

b . Use two patient identifiers before giving medications . c. Do not give a medication that another nurse has drawn up in a syringe . d . Minimize the use of verbal and telephone orders .

A patient has a nursing diagnosis of "Noncompliance with medication regimen related to a belief that God will heal her and that it would show a lack of faith to take the medication. The nurse and a clergyman have spent some time discussing spiritual and treatment issues with the patient. Which patient statement indicates that progress is being made toward achieving compliance with healthcare therapy? Select all that apply. a)" I will try to pray more often for stronger faith that God will heal me. ́ b) "Let me think about it until tomorrow; I may see my way to taking those pills then." c) "You know, I've known some very holy people who were not cured by God" 4) "There is no confusion in my mind about the right thing for me to do." 5) "I'm not sure if this treatment is God's will for me ́

b) "Let me think about it until tomorrow; I may see my way to taking those pills then." c) "You know, I've known some very holy people who were not cured by God"

A patient with a history of chronic obstructive pulmonary disease has a living will that states he does not want endotracheal intubation and mechanical ventilation as a means of respiratory resuscitation. As the patient's condition deteriorates, the patients ask whether he can change his decision. Which response by the nurse is best? a) "I'll call your physician right away so he can discuss this with you." b) "You have the right to change your decision about treatment at any time " c) "Are you sure you want to change your decision about treatment at any time" ́ d) "We must follow whatever is written in your living will."

b) "You have the right to change your decision about treatment at any time "

Throughout the course of an illness, a patient has denied its seriousness, even though health professionals have explained prognosis of death very clearly. Physiological signs now indicate that death will probably occur within a short period of time, but the patient is still firmly in a state of emotional denial. The patient say to the nurse, "Tell my wife to stop hovering and go home." How should the nurse respond? a) "Your physical signs indicate that you will likely not live more than a few more days." b) "You seem very sure that you are not going to die. Please tell me more about what you are feeling." c) "It seems to me you would be feeling some anger and wondering why all this is happening to you" d) "It would be best for your family if you were able to work through this and come to accept the reality of your situation."

b) "You seem very sure that you are not going to die. Please tell me more about what you are feeling."

The spouse of a dying client who wants to be present at the time of death desires to go home to shower but is afraid the client will die during that time. Which response by the nurse is best? a) "Certainly, go ahead; your husband will most likely hold on until you return? b) "Your husband could live for days or a few hours; you should do whatever you are comfortable with ́ c) You need to take care of yourself; go home and shower, and ,I'll stay at his bedside while you are gone ́ d) Don't worry. Your husband is in good hands; ,I'll look out for him ́

b) "Your husband could live for days or a few hours; you should do whatever you are comfortable with ́

Which client has the greatest need for comprehensive discharge planning? A) A 12-year-old boy who had outpatient surgery on his knee and lives with his mother B) A woman who was just diagnosed with renal failure and has started peritoneal dialysis C) A man who has been readmitted for exacerbation of his chronic obstructive pulmonary disease D) A woman who has just given birth to her second child and lives with her husband and 18-month-old daughter

b) A woman who was just diagnosed with renal failure and has started peritoneal dialysis

Which set of vital signs is within normal limits for a client at rest? A) Infant: T 98.8°F (rectal), HR 160, RR 16, BP 120/54 B) Adolescent: T 98.2°F (oral), HR 80, RR 18, BP 108/68 C) Adult: T 99.6°F (oral), HR 48, RR 22, BP 130/84 D) Older adult: T 98.6°F (oral), HR 110, RR 28, BP 170/95

b) Adolescent: T 98.2°F (oral), HR 80, RR 18, BP 108/68

Which theories focus on what the leader does? a) Trait theories b) Behavioral theories c) Situational theories d) Transformational theories

b) Behavioral theories

According to the Uniform Determination of Death Act, which bodily function must be lost to declare death? a) Consciousness b) Brain stem function c) Cephalic reflexes d) Spontaneous respirations

b) Brain stem function

A client who is experiencing ongoing grief after the unexpected death of a spouse several years ago questions the existence of God. Which nursing diagnosis would be the most appropriate for this client? a) Death Anxiety b) Chronic Sorrow c) Spiritual Distress d) Risk for Spiritual Distress

b) Chronic Sorrow

The nurse is identifying short-term goals for the day. Which statement exemplifies this type of goal? a) Register for a certification review course in a year. b) Complete documentation prior to hand-off communication. c) Discuss the implementation of a new checklist for staff to use. d) Plan a cruise with family after graduating from a Master's degree program.

b) Complete documentation prior to hand-off communication.

The manager is conducting an informal negotiation between two staff members and is focusing on managing the emotions and setting the ground rules. Which stage does this demonstrate? a) Setting the stage b) Conducting the negotiation c) Making offers and counteroffers d) Agreeing on resolution of the conflict

b) Conducting the negotiation

The family notices that a terminally ill client is more focused and coherent and they ask whether the client is really going to die. When should the nurse recognize that a sudden surge of activity is most likely to occur? a) Moments before death b) Days to hours before death c) 1 to 2 weeks before death d) 1 to 3 months before death

b) Days to hours before death

A home health patient previously lived with her sister for more than 20 years. Although it has been over a year since her sister died, the patient tells the nurse, "Its no worse now, but I never feel any relief from this overwhelming sadness. I still can't sleep a full night. The house is a mess; I feel too tired even to take a bath. But, sometimes at night, she comes to me and I can see her plain as can be. The patient's clothing is not clean and her hair is not combed. She is apparently not eating adequately. It is the patient's birthday. What should the nurse conclude about the patient? a) Grieving longer than usual because of the closeness of the relationship with her sister b) Experiencing a depressive disorder rather than simply grieving the loss of her sister c) Feeling guilt and worthlessness because her sister died and she is still alive d) Interpreting the birthday as a trigger event, which is causing her to hallucinate

b) Experiencing a depressive disorder rather than simply grieving the loss of her sister

Which behavior might be a warning sign that a child needs professional help after the death of a loved one? Select all that apply. a) Interest in usual activities b) Extended regression c) Withdrawal from friends d) Inability to sleep e) Intermittent sadness

b) Extended regression c) Withdrawal from friends d) Inability to sleep

The nurse plans to assess a client's spiritual needs with the JAREL tool. What dimensions will the nurse assess through this tool? Select all that apply. a) Hope/love b) Faith/belief c) Transcendence/peace d) Life/self-responsibility e) Life-satisfaction/self-actualization

b) Faith/belief d) Life/self-responsibility e) Life-satisfaction/self-actualization

Which expected outcome is best for a patient with a nursing diagnosis of Deficient Knowledge related to new drug treatment regimen? a) After an explanation and written materials, describes the expected actions and adverse reactions of his medication b) In 1 week after instructional session, describes the expected actions and adverse reactions of his medications c) Follows the treatment plan as prescribed d) Experiences no adverse effect from his prescribed treatment plan

b) In 1 week after instructional session, describes the expected actions and adverse

The staff development instructor prepares an in-service presentation on full-spectrum nursing for new graduate nurses. Which statement should the instructor emphasize as being the key point about this delivery approach? A) It encourages collaboration when planning care. B) It is a unique blend of thinking, doing, and caring. C) It focuses on assessment as the cornerstone of care. D) It relies on client responses to guide interventions.

b) It is a unique blend of thinking, doing, and caring.

The nurse identifies a collaborative problem for a client. Which type of interventions should be selected to address this problem? Select all that apply. A) Client requested B) Nursing prescribed C) Physician prescribed D) Pharmacist prescribed E) Risk management suggested

b) Nursing prescribed c) Physician prescribed

When providing postmortem care, the nurse places dentures in the mouth and closes the eyes and mouth of the patient within 2 to 4 hours after death. Why is the timing of the action so important? a) Prevent blood from settling in the head, neck, and shoulders. b) Perform these actions more easily before rigor mortis develops. c) Set the mouth in a natural position for viewing by the family. d) Prevent discoloration caused by blood settling in the facial area.

b) Perform these actions more easily before rigor mortis develops

The primary care provider prescribes nitroglycerin 1/150 g SL for a patient experiencing chest pain. How should the nurse administer the drug? a) Place the drug in the cheek and allow it to dissolve. b) Place the drug under the tongue and allow it to dissolve. c) Inject the drug superficially into the subcutaneous tissue. d) Give the pill and water to the patient for him to swallow the tablet.

b) Place the drug under the tongue and allow it to dissolve.

The nurse spends time listening to a client explain the course of an illness. Which spirituality intervention is the nurse implementing? a) Touch b) Presence c) Reminiscence d) Exploring meaning

b) Presence

Which patient goal is most appropriate when managing the patient dying of cancer? a) Request pain medication when needed. b) Report or demonstrate satisfactory pain control. c) Use only nonpharmacological measures to control pain. d) Verbalize understanding that it may not be possible to control his pain.

b) Report or demonstrate satisfactory pain control.

Which assessment data best supports a report of severe pain in an adult client whose baseline vital signs are within an average normal range? A) Oral temperature 100°F (37.8°C) B) Respiratory rate 26 breaths/min and shallow C) Apical heart rate 56 beats/min D) Blood pressure 124/82 mm Hg

b) Respiratory rate 26 breaths/min and shallow

The nurse uses a standardized care plan for Impaired Walking related to left-sided weakness when planning care for a patient. Which activity will the nurse perform when individualizing care for this patient? A) Transcribe medical orders. B) State the frequency for ambulation. C) Perform a comprehensive assessment. D) Validate conflicting data with the patient.

b) State the frequency for ambulation.

Which patient is at most risk for experiencing difficult grieving? a) The middle-aged woman whose grandmother died of advanced Parkinson's disease b) The young adult with three small children whose wife died suddenly in an accident c) The middle-aged person whose spouse suffered a slow, painful death d) The older adult whose spouse died of complications of chronic renal disease

b) The young adult with three small children whose wife died suddenly in an accident

Which best explains why it is difficult for the profession to develop a definition of nursing? A) There are too many different and conflicting images of nurses. B) There are constant changes in healthcare and the activities of nurses. C) There is disagreement among the different nursing organizations. D) There are different education pathways and levels of practice.

b) There are constant changes in healthcare and the activities of nurses.

Which intervention is most appropriate when a client develops a "death rattle"? a) Perform nasotracheal suctioning of secretions. b) Turn the patient on his side and raise the head of the bed. c) Insert a nasopharyngeal airway as needed. d) Administer morphine sulfate intravenously.

b) Turn the patient on his side and raise the head of the bed.

What is the first stage of the complex process of change? a) Recognizing resistance b) Unfreezing c) Forming a comfort zone d) Actively resisting

b) Unfreezing

Which are examples of cues? Select all that apply. A) Taking a brisk walk five times a week B) Using laxatives to have a bowel movement C) Needing more sleep than usual D) Decreasing the amount of fat in the diet E) Weighing less than indicated by developmental norms

b) Using laxatives to have a bowel movement c) Needing more sleep than usual e) Weighing less than indicated by developmental norms

The nurse must administer hepatitis B immunoglobulin 0.5 mL intramuscularly to a 3-day-old infant. Which injection site should the nurse choose to administer this injection? a) Ventrogluteal b) Vastus lateralis c) Deltoid d) Dorsogluteal

b) Vastus lateralis

The insulin order reads, "Give 10 units of NPH insulin and 5 units of regular insulin, subcut, every morning before breakfast." Choose the proper syringe for this injection.

b.

A patient is taking nystatin (Mycostatin) oral lozenges to treat an oral candidiasis infection resulting from inhaled corticosteroid therapy for asthma. Which instruction by the nurse is appropriate? a. "Chew the lozenges until they are completely dissolved." b. "Let the lozenge dissolve slowly and completely in your mouth without chewing it." c. "Rinse your mouth with water before taking the inhaler." d. "Rinse your mouth with mouthwash after taking the inhaler."

b. "Let the lozenge dissolve slowly and completely in your mouth without chewing it."

The nurse is preparing a patient for a computed tomography (CT) scan of the abdomen. Which statement by the nurse is best (all contain correct information)? A) You will need to remain NPO for the 4 hours prior to your CT scan B) You cannot have anything to eat or drink for 4 hours before your test C) You will need to be NPO and drink this contrast medium before your test D) You may need to void before you go down to the department for your CT scan

b. "You cannot have anything to eat or drink for 4 hours before your test."

The nurse is preparing to transfuse a patient with a unit of packed red blood cells (PRBCS). Which intravenous solution is correct for use with the PRBC transfusion? a. 5% dextrose in water (D5W) b. 0.9% sodium chloride (NS) c. 5% dextrose in 0.45% sodium chloride (D5NS) d. 5% dextrose in lactated Ringer's solution (D5LR)

b. 0.9% sodium chloride (NS)

A nurse is working in an immunization clinic. A new colleague asks, "When is the first dose of the diphtheria, tetanus, and acellular pertussis (DtaP, Daptacel) given?" The nurse knows that this series is started at what age? a. At birth b. 6 weeks c. 3 months d. 1 year

b. 6 weeks

The nurse is developing a diagnostic statement when planning care for a client. The statement reads: Possible Risk for Constipation related to irregular defecation habits Patient states that when I'm busy, I can't always take the time to go to the bathroom. What is wrong with the format of this diagnostic statement? A) Possible nursing diagnoses do not have signs and symptoms. B) A nursing diagnosis is either a possible risk or a risk, not both. C) Constipation is a medical diagnosis. D) The etiology is actually a defining characteristic.

b. A nursing diagnosis is either a possible risk or a risk, not both.

The nurse is preparing to transfuse a patient with a unit of packed red blood cells (PRBCS). Which patient would be best treated with this transfusion? a. A patient with a coagulation disorder b. A patient with severe anemia с. A patient who has lost a massive amount of blood after an accident d. A patient who has a clotting-factor deficiency

b. A patient with severe anemia

How does a risk nursing diagnosis differ from a possible nursing diagnosis? A) A risk diagnosis is based on data about the patient. B) A possible diagnosis is based on partial (or incomplete) data. C) Nurses collect the data to support risk diagnoses. D) A possible diagnosis becomes an actual diagnosis when symptoms develop.

b. A possible diagnosis is based on partial (or incomplete) data.

A young adult with a severe episode of asthma bronchoconstriction comes to the emergency department with signs of respiratory distress. When the nurse performs the admission assessment, the patient is not oriented to place or time. Which nursing diagnosis is most suitable for this patient? A) Chronic Confusion B) Acute Confusion C) Impaired Verbal Communication D) Readiness for Enhanced Communication

b. Acute Confusion

A 57-year-old woman being treated for end-stage breast cancer has been using a transdermal opioid analgesic as part of the management of pain. Lately, she has been experiencing breakthrough pain. The nurse expects this type of pain to be managed by which of these interventions? a. Administering NSAIDs b. Administering an immediate-release opioid c. Changing the opioid route to the rectal route d. Making no changes to the current therapy

b. Administering an immediate-release opioid

For which patient would it be most important to perform a comprehensive discharge plan? A) A teen who is a first-time mother and single and lives with her parents B) An older adult who has had a stroke affecting the left side of his body and lives alone C) A middle-aged man who has had outpatient surgery on his knee and requires crutches D) A young woman who was admitted to the hospital for observation following an accident

b. An older adult who has had a stroke affecting the left side of his body and lives alone

A patient is in the urgent care center after experiencing a black widow spider bite. The nurse prepares to give which product to treat this injury? a. Live vaccine b. Antivenins or antisera с. Tetanus immune globulin d. Active immunizing drug

b. Antivenins or antisera

A 12-month-old infant has received an MMR II (measles, mumps, and rubella virus vaccine), and her mother calls the clinic that afternoon to ask about helping her fussy infant to "feel better." What will the nurse suggest? a. Apply an ice pack to the injection site. b. Apply warm compresses to the injection site. c. Observe the site for further swelling and redness. d. Bring the infant in to the emergency department for an immediate examination.

b. Apply warm compresses to the injection site.

Vicodin (acetaminophen/hydrocodone) is prescribed for a patient who has had surgery. The nurse informs the patient that which common adverse effects can occur with this medication? (Select all that apply.) a. Diarrhea b. Constipation c. Lightheadedness d. Nervousness e. Urinary retention f. Itching

b. Constipation c. Lightheadedness e. Urinary retention f. Itching

The nurse selects the nursing diagnosis of Impaired Memory related to fluid and electrolyte imbalances as evidenced by inability to recall recent events. Which goal/outcome must be included on the care plan? A) Checks current medications for mind-altering side effects B) Demonstrates use of techniques to help with memory loss C) Drinks at least 1500 cc of fluid per day D) Takes electrolyte supplements with meals

b. Demonstrates use of techniques to help with memory loss

The nurse is administering an amphotericin B infusion. Which actions by the nurse are appropriate? (Select all that apply.) a. Administering the medication by rapid IV infusion b. Discontinuing the drug immediately if the patient develops tingling and numbness in the extremities c. If adverse effects occur, reducing the IV rate gradually until they subside d. Using an infusion pump for IV therapy e. Monitoring the IV site for signs of phlebitis and infiltration f. Administering premedication for fever and nausea g. Knowing that the intravenous solution for amphotericin B will be cloudy h. Knowing that muscle twitching may indicate hypokalemia

b. Discontinuing the drug immediately if the patient develops tingling and numbness in the extremities d. Using an infusion pump for IV therapy e. Monitoring the IV site for signs of phlebitis and infiltration f. Administering premedication for fever and nausea

A patient who has undergone a lung transplant has contracted cytomegalovirus (CMV) retinitis. The nurse expects which drug to be ordered for this patient? a. Acyclovir (Zovirax) b. Ganciclovir (Cytovene) c. Ribavirin (Virazole) d. Amantadine (Symmetrel)

b. Ganciclovir (Cytovene)

After starting treatment for type 2 diabetes mellitus 6 months earlier, a patient is in the office for a follow-up examination. The nurse will monitor which laboratory test to evaluate the patient's adherence to the antidiabetic therapy over the past few months? a. Hemoglobin levels b. Hemoglobin AlC level c. Fingerstick fasting blood glucose level d. Serum insulin levels

b. Hemoglobin AlC level

A patient is recovering from abdominal surgery, which he had this morning. He is groggy but complaining of severe pain around his incision. What is the most important assessment data to consider before the nurse administers a dose of morphine sulfate to the patient? a. His pulse rate b. His respiratory rate c. The appearance of the incision d. The date of his last bowel movement

b. His respiratory rate

A patient with a severe fungal infection has orders for voriconazole (Vfend). The nurse is reviewing the patient's medical record and would be concerned if which assessment finding is noted? a. Decreased breath sounds in the lower lobes b. History of cardiac dysrhythmias c. History of type 2 diabetes d. Potassium level of 4.0 mEq/L

b. History of cardiac dysrhythmias

When administering morning medications for a newly admitted patient, the nurse notes that the patient has an allergy to sulfa drugs. There is an order for the sulfonylurea glipizide (Glucotrol). Which action by the nurse is correct? a. Give the drug as ordered 30 minutes before breakfast. b. Hold the drug, and check the order with the prescriber. c. Give a reduced dose of the drug with breakfast. d. Give the drug, and monitor for adverse effects.

b. Hold the drug, and check the order with the prescriber.

If a patient is taking fluconazole (Diflucan) with an oral anticoagulant, the nurse will monitor for which possible interaction? a. Reduced action of oral anticoagulants b. Increased effects of oral anticoagulants с. Нурokalemia d. Decreased effectiveness of the antifungal drug

b. Increased effects of oral anticoagulants

A client who arrives in the emergency department, pale and breathing rapidly becomes unconscious and collapses to the floor. The nurse rapidly assesses the client and decides the first series of actions that are needed. What does this scenario demonstrate? A) Formal planning B) Informal planning C) Ongoing planning D) Initial planning

b. Informal planning

The nurse is reviewing a patient's medication list and notes that sitagliptin (Januvia) is ordered. The nurse will question an additional order for which drug or drug class? a. Glitazone b. Insulin c. Metformin (Glucophage) d. Sulfonylurea

b. Insulin

Which statement about the nursing process is correct? A) It was developed from the ANA Standards of Care. B) It is a problem-solving method to guide nursing activities. C) It is a linear process with separate, distinct steps. D) It involves care that only the nurse will give.

b. It is a problem-solving method to guide nursing activities

During diuretic therapy, the nurse monitors the fluid and electrolyte status of the patient. Which assessment findings are symptoms of hyponatremia? (Select all that apply.) a. Red, flushed skin b. Lethargy c. Decreased urination d. Hypotension e. Stomach cramps f. Elevated temperature

b. Lethargy d. Hypotension e. Stomach cramps

A few nurses on a unit have proposed to the nurse manager that the process for documenting care on the unit be changed. They have described a completely new system. The nurse should have a critical attitude because it helps the manager to do which item? A) Consider all the possible advantages and disadvantages. B) Maintain an open mind about the proposed change. C) Apply the nursing process to the situation. D) Make a decision based on past experience with documentation.

b. Maintain an open mind about the proposed change

The nurse is preparing to administer insulin intravenously. Which statement about the administration of intravenous insulin is true? a. Insulin is never given intravenously. b. Only regular insulin can be administered intravenously. c. Insulin aspart or insulin lispro can be administered intravenously, but there must be a 50% dose reduction. d. Any form of insulin can be administered intravenously at the same dose as that is ordered for subcutaneous administration.

b. Only regular insulin can be administered intravenously.

A patient has been treated for lung cancer for 3 years. Over the past few months, the patient has noticed that the opioid analgesic is not helping as much as it had previously and more medication is needed for the same pain relief. The nurse is aware that this patient is experiencing which of these? a. Opioid addiction b. Opioid tolerance c. Opioid toxicity d. Opioid abstinence syndrome

b. Opioid tolerance

The nurse is reviewing the medication history of a patient who will be taking a sulfonamide antibiotic. During sulfonamide therapy, a significant drug interaction may occur with which of these drugs or drug classes? (Select all that apply.) a. Opioids b. Oral contraceptives c. Sulfonylureas d. Antihistamines e. Phenytoin (Dilantin) f. Warfarin (Coumadin)

b. Oral contraceptives c. Sulfonylureas e. Phenytoin (Dilantin) f. Warfarin (Coumadin)

Which is the most obvious reason for using a framework when assessing a patient? A) Prioritize assessment data B) Organize and cluster data C) Separate subjective and objective data D) Identify primary from secondary data

b. Organize and cluster data

A patient is receiving gabapentin (Neurontin), an anticonvulsant, but has no history of seizures. The nurse expects that the patient is receiving this drug for which condition? a. Inflammation pain b. Pain associated with peripheral neuropathy c. Depression associated with chronic pain d. Prevention of seizures

b. Pain associated with peripheral neuropathy

A patient refuses a dose of medication. How should the nurse document the event? A) Patient is uncooperative and refuses the prescribed dose of digoxin. B) Patient refuses the 0900 dose of digoxin. C) Patient is belligerent and argumentative and refuses the 0900 dose of digoxin. D) 0900 dose of digoxin not given.

b. Patient refuses the 0900 dose of digoxin

The nurse is reviewing facts about pharmacology for a review course. The term legend drug refers to which item? a. Over-the-counter drugs b. Prescription drugs c. Orphan drugs d. Older drugs

b. Prescription drugs

A patient will be having oral surgery and has received an antibiotic to take for 1 week before the surgery. The nurse knows that this is an example of which type of therapy? a. Empiric b. Prophylactic c. Definitive d. Resistance

b. Prophylactic

Which nursing diagnosis is appropriate for a patient who has started aminoglycoside therapy? a. Constipation b. Risk for injury (renal damage) c. Disturbed body image related to gynecomastia d. Imbalanced nutrition, less than body requirements, related to nausea

b. Risk for injury (renal damage)

Which is an example of theoretical knowledge? A) A nurse uses sterile technique to catheterize a patient. B) Room air has an oxygen concentration of 21%. C) Glucose monitoring machines should be calibrated daily. D) An irregular apical heart rate should be compared with the radial pulse.

b. Room air has an oxygen concentration of 21%.

The nurse is preparing to give a potassium supplement. Which laboratory test should be checked before the patient receives a dose of potassium? a. Complete blood count b. Serum potassium level C. Serum sodium level d. Liver function studies

b. Serum potassium level

The nurse will be injecting a drug into the fatty tissue of the patient's abdomen. Which route does this describe? a. Intradermal b. Subcutaneous c. Intramuscular d. Transdermal

b. Subcutaneous

A patient with a history of chronic obstructive pulmonary disease (COPD) and type 2 diabetes has been treated for pneumonia for the past week. The patient has been receiving intravenous corticosteroids as well as antibiotics as part of his therapy. At this time, the pneumonia has resolved, but when monitoring the blood glucose levels, the nurse notices that the level is still elevated. What is the best explanation for this elevation? a. The antibiotics may cause an increase in glucose levels. b. The corticosteroids may cause an increase in glucose levels. c. His type 2 diabetes has converted to type 1. d. The hypoxia caused by the COPD causes an increased need for insulin.

b. The corticosteroids may cause an increase in glucose levels.

A patient has been diagnosed with metabolic syndrome and is started on the biguanide metformin (Glucophage). The nurse knows that the purpose of the metformin, in this situation, is which of these? a. To increase the pancreatic secretion of insulin b. To decrease insulin resistance c. To increase blood glucose levels d. To decrease the pancreatic secretion of insulin

b. To decrease insulin resistance

What is the purpose of completing an occurrence report? A) Provide a legal defense should the patient seek legal action after an unusual occurrence B) Track problems and identify areas for quality improvement C) Report errors to the Food and Drug Administration D) Report medical errors to The Joint Commission

b. Track problems and identify areas for quality improvement

Nurses have the ethical responsibility to tell the truth to their patients. What is this principle known as? a. Justice b. Veracity c. Beneficence d. Autonomy

b. Veracity

After a severe auto blood loss of more than 30% of his blood volume. The nurse prepares to administer which accident, a patient has been taken to the trauma unit and has an estimated product? a. Albumin b. Whole blood c. Packed red blood cells d. Fresh frozen plasma

b. Whole blood

The nurse knows to administer acarbose (Precose), an alpha-glucosidase inhibitor, at which time? a. 30 minutes before break fast b. With the first bite of each main meal c. 30 minutes after breakfast d. Once daily at bedtime

b. With the first bite of each main meal

The nurse is providing counseling to a woman who is HIV positive and has just discovered that she is pregnant. Which anti-HIV drug is given to HIV-infected pregnant women to prevent transmission of the virus to the infant? a. Acyclovir (Zovirax) b. Zidovudine (Retrovir) c. Ribavirin (Virazole) d. Foscarnet (Foscavir)

b. Zidovudine (Retrovir)

The nurse is reviewing a list of verbal medication orders . Which is the proper notation of the dose of the drug ordered ? a . Digoxin.125 mg b . Digoxin .1250 mg c . Digoxin 0.125 mg d . Digoxin 0.1250 mg

c . Digoxin 0.125 mg

A patient says he prefers to chew rather than swallow his pills . One of the pills has the abbreviation SR behind the name of the medication . The nurse needs to remember which correct instruction regarding how to give this medication ? a . Break the tablet into halves or quarters . b . Dissolve the tablet in a small amount of water before giving it . c . Do not crush or break the tablet before administration . d . Crush the tablet as needed to ease administration .

c . Do not crush or break the tablet before administration .

The nurse needs to administer insulin subcutaneously to an obese patient . Which is the proper technique for this injection ? a . Using the Z - track method b . Inserting the needle at a 5- to 15 - degree angle until resistance is felt c . Pinching the skin at the injection site , and then inserting the needle to below the tissue fold at a 90 - degree angle d . Spreading the skin tightly over the injection site , inserting the needle , and then releasing the skin

c . Pinching the skin at the injection site , and then inserting the needle to below the tissue fold at a 90 - degree angle

When giving a buccal medication to a patient , which action by the nurse is appropriate ? a . Encouraging the patient to swallow , if necessary b . Administering water after the medication has been given c . Placing the medication between the upper or lower molar teeth and the cheek d . Placing the tablet under the patient's tongue and allowing it to dissolve completely

c . Placing the medication between the upper or lower molar teeth and the cheek

When taking a telephone order for a medication , which action by the nurse is most appropriate ? a . Verify the order with the charge nurse . b . Call back the prescriber to review the order c . Repeat the order to the prescriber before hanging up the telephone . d . Ask the pharmacist to double - check the order .

c . Repeat the order to the prescriber before hanging up the telephone .

A 2 - year - old child is to receive eardrops . The nurse is teaching the parent about giving the eardrops . Which statement reflects the proper technique for administering eardrops to this child ? a . Administer the drops without pulling on the ear lobe . b . Straighten the ear canal by pulling the lobe upward and back . c . Straighten the ear canal by pulling the pinna down and back . d . Straighten the ear canal by pulling the pinna upward and outward .

c . Straighten the ear canal by pulling the pinna down and back .

After administering an intradermal ( ID ) injection for a skin test , the nurse notices a small bleb at the injection site . The best action for the nurse to take at this time is to : a . apply heat . b . massage the area . c . do nothing . d . report the bleb to the physician .

c . do nothing .

Which intervention takes priority for the patient receiving hospice care? a) Turning and repositioning the patient every 2 hours b) Assisting the patient out of bed and into a chair twice a day c) Administering pain medication to keep the patient comfortable d) Providing the patient with small frequent, nutritious meals

c) Administering pain medication to keep the patient comfortable

The nurse is teaching parents ways to give oral medication to their child. Which action would they implement to improve compliance? a) Crush time-release capsules to put in his favorite food. b) Give medication quickly before he knows what is happening. c) Allow the child to eat a frozen pop before receiving the medication. d) Mask the flavor of medication in a toddler cup with orange juice.

c) Allow the child to eat a frozen pop before receiving the medication.

The nurse, a Roman Catholic, notes that a client with a terminal illness identifies as being an atheist. What should the nurse do to support this client's spiritual needs? a) Suggest the chaplain visit with the client. b) Offer to read Bible passages with the client. c) Assess the client's value system and beliefs. d) Explain the importance of religion at end of life.

c) Assess the client's value system and beliefs.

The nurse desires to improve time management skills. What action should the nurse take first? a) Make a to-do list. b) Use a daily worksheet. c) Complete a time inventory. d) Focus on completing the most difficult tasks first.

c) Complete a time inventory.

A patient dying of heart failure has changed his choice about his end-of-life treatment measures several times. He says, "I just can't make up my mind about it. Which nursing diagnosis is most appropriate for this patient? a) Deficient Knowledge b) Spiritual Distress c) Decisional Conflict d) Death Anxiety

c) Decisional Conflict

A nursing student is having difficulty understanding the role of nursing diagnosis and setting client goals. What should the instructor explain to this student? A) Client goals are used to coordinate care activities. B) Client goals ensure cost-effective care is provided. C) Diagnosis is the basis for planning client-centered goals. D) Diagnosis ensures the correct treatment is prescribed for the client.

c) Diagnosis is the basis for planning client-centered goals.

An expert nurse feels confident in the role as a clinician on the unit, enjoys the work, and feels in charge of career direction. Which leadership state is the nurse experiencing? a) Power-based authority b) Effective management skills c) Empowerment in the role d) Followership skills

c) Empowerment in the role

The nurse is asking the patient reflective, clarifying questions to help the patient identify purpose in life. Which nursing intervention does this action implement? a) Spiritual Support b) Self-Esteem Enhancement c) Exploring Meaning d) Hope Inspiration

c) Exploring Meaning

How should the nurse respond to a family immediately after a patient dies? a) Ask the family to leave the patient's room so postmortem care can be performed. b) Leave tubes and IV lines in place until the family has the opportunity to view the body. c) Express sympathy to the family (e.g., " I am sorry for your loss ́ d) Tell the family that they will have limited time with their loved one.

c) Express sympathy to the family (e.g., " I am sorry for your loss ́

The nurse manager desires to use the SOAR process to improve the care area performance. On what should the manager focus when using SOAR for this analysis? a) Results b) Strengths c) Inspiration d) Opportunities

c) Inspiration

Which explains why it is important to have the correct etiology for a nursing diagnosis? A) It is the cause of the problem. B) It cannot always be observed. C) It directs nursing care. D) It is an inference.

c) It directs nursing care.

An organization changed the documentation system a month ago; however, one staff nurse has missed every training session and continues to use the previous documentation system. What is this nurse demonstrating? a) Unfreezing b) Active resistance c) Passive resistance d) Comfort zone

c) Passive resistance

The nurse with 15 years of experience and 7 years of employment with an organization will provide guidance and advice to a new graduate nurse. What is the role of the experienced nurse? a) Mentor b) Manager c) Preceptor d) Leader

c) Preceptor

Which intervention is appropriate for a client receiving palliative care? Select all that apply. a) Surgically inserting a device to decrease the workload of the heart in a patient awaiting heart transplantation b) Administering IV dopamine to raise blood pressure of a patient with end-stage lung cancer c) Providing moisturizing eye drops to an unconscious patient whose eyes are dry d) Administering a medication to relieve the nausea of a patient with end-stage leukemia e) Establishing an advance directive that addresses the next 6 months of care

c) Providing moisturizing eye drops to an unconscious patient whose eyes are dry d) Administering a medication to relieve the nausea of a patient with end-stage leukemia

The nurse is assigned to care for a client with Guillain-Barré syndrome; however, the nurse does not understand the disorder. What is the best way for the nurse to address this gap in knowledge? A) Talk with the family. B) Complete the client assessment. C) Research and read about the disorder. D) Ask another nurse to explain the disorder.

c) Research and read about the disorder.

The mother of a preschool child dies suddenly of a ruptured cerebral aneurysm. What recommendation should the nurse make to the family regarding how to most therapeutically care for the child? a) Take the child to the funeral even if he is frightened. b) Notify the physician immediately if the child shows signs of regression. c) Spend as much time as possible with the child. d) Provide distraction whenever the child begins to express feelings of sadness.

c) Spend as much time as possible with the child.

A patient with terminal cancer requires increasing doses of an opioid pain medication to obtain relief from pain. This patient is exhibiting signs of drug: a) Abuse b) Misuse c) Tolerance d) Dependence

c) Tolerance

A nursing assistive personnel (NAP) with limited direct care experience is assigned to help on a care area. Which task should the nurse safely delegate to the NAP to complete? a) Feeding b) Ambulating c) Transporting d) Vital sign measurement

c) Transporting

A patient has difficulty taking liquid medications from a cup. How should the nurse administer the medications? a) Request that the prescriber change the order to the IV route. b) Administer the medication by the IM route. c) Use a needleless syringe to place the medication in the side of the mouth. d) Add the dose to a small amount of food or beverage to facilitate swallowing.

c) Use a needleless syringe to place the medication in the side of the mouth.

Which intervention should be included in the plan of care for a patient in the end-stage death process? a) Encourage the patient to accept as much help as possible. b) Avoid administering laxatives. c) Wet the lips and mouth frequently. d) Administer pain medication on an as-needed basis.

c) Wet the lips and mouth frequently.

A young adult calls the clinic to ask for a prescription for "that new flu drug." He says he has had the flu for almost 4 days and just heard about a drug that can reduce the symptoms. What is the nurse's best response to his request? a. "Now that you've had the flu, you will need a booster vaccination, not the antiviral drug." b. "We will need to do a blood test to verify that you actually have the flu." c. "Drug therapy should be started within 2 days of symptom onset, not 4 days." d. "We'll get you prescription. As long as you start treatment within the next 24 hours, the drug should be effective."

c. "Drug therapy should be started within 2 days of symptom onset, not 4 days."

The nurse is reviewing instructions for a patient with type 2 diabetes who also takes insulin injections as part of the therapy. The nurse asks the patient, "What should you do if your fasting blood glucose is 47 mg/dL?" Which response by the patient reflects a correct understanding of insulin therapy? a. "I will call my doctor right away." b. "I will give myself the regular insulin." c. "I will take an oral form of glucose." d. "I will rest until the symptoms pass."

c. "I will take an oral form of glucose."

When given an intravenous medication, the patient says to the nurse, "I usually take pills. Why does this medication have to be given in the arm?" What is the nurse's best answer? a. "The medication will cause fewer adverse effects when given intravenously." b. "The intravenous medication will have delayed absorption into the body's tissues." c. "The action of the medication will begin sooner when given intravenously." d. "There is a lower chance of allergic reactions when drugs are given intravenously."

c. "The action of the medication will begin sooner when given intravenously."

The nurse is assessing a patient for contraindications to drug therapy with acetaminophen (Tylenol). Which patient should not receive acetaminophen? a. A patient with a fever of 101° F (38.3° C) b. A patient who is complaining of a mild headache c. A patient with a history of liver disease d. A patient with a history of peptic ulcer disease

c. A patient with a history of liver disease

A patient has received a prescription for a 2-week course of antifungal suppositories for a vaginal yeast infection. She asks the nurse if there is an alternative to this medication, saying, "I don't want to do this for 2 weeks!" Which is a possibility in this situation? a. A single dose of a vaginal antifungal cream. b. A one-time infusion of amphotericin B. c. A single dose of a fluconazole (Diflucan) oral tablet. d. There is no better alternative to the suppositories.

c. A single dose of a fluconazole (Diflucan) oral tablet.

A 6-year-old child who has chickenpox also has a fever of 102.9° F (39.4° C). The child's mother asks the nurse if she should use aspirin to reduce the fever. What is the best response by the nurse? a. It's best to wait to see if the fever gets worse. b. You can use the aspirin, but watch for worsening symptoms. c. Acetaminophen (Tylenol) should be used to reduce his fever, not aspirin. d. You can use aspirin, but be sure to follow the instructions on the bottle.

c. Acetaminophen (Tylenol) should be used to reduce his fever, not aspirin.

A patient in the emergency department was showing signs of hypoglycemia and had a fingerstick glucose level of 34 mg/dL. The patient has just become unconscious. What is the nurse's next action? a. Have the patient eat glucose tablets. b. Have the patient consume fruit juice, a nondiet soft drink, or crackers. c. Administer intravenous glucose (50% dextrose). d. Call the lab to order a fasting blood glucose level.

c. Administer intravenous glucose (50% dextrose).

A 78-year-old patient is in the recovery room after having a lengthy surgery on his hip. As he is gradually awakening, he requests pain medication. Within 10 minutes after receiving a dose of morphine sulfate, he is very lethargic and his respirations are shallow, with a rate of 7 per minute. The nurse prepares for which priority action at this time? a. Assessment of the patient's pain level b. Immediate intubation and artificial ventilation c. Administration of naloxone (Narcan) d. Close observation of signs of opioid tolerance

c. Administration of naloxone (Narcan)

Which situation is the most conducive to conducting a successful interview of an older adult client (alert and oriented) whose spouse and two children are in the hospital room visiting and watching television? A) Provide enough chairs so the family and you are able to sit facing the client. B) Introduce yourself and ask Dear, what name do you prefer to go by ? before asking any questions. C) After the family leaves, ask the client if she is comfortable and willing to answer a few questions. D) Ask the client if you can talk with her while her family is watching the television.

c. After the family leaves, ask the client if she is comfortable and willing to answer a few questions.

A 75-year-old woman has been given a nonsteroidal anti-inflammatory drug (an NSAID for the treatment of rheumatoid arthritis. The nurse is reviewing the patient's medication history and notes that which types of medications could have an interaction with the NSAID? (Select all that apply.) a. Antibiotics b. Decongestants c. Anticoagulants d. Beta blockers e. Diuretics f. Corticosteroids

c. Anticoagulants e. Diuretics f. Corticosteroids

Which is a disadvantage of paper health records? A) Assist collaboration B) Provide cautionary reminders C) Are sometimes illegible D) Serve as a resource

c. Are sometimes illegible

A client who cannot manage a patient-controlled analgesia pump is prescribed morphine 4 mg intravenously q 1-hour PRN pain. When should the nurse administer the medication? A) Every hour around the clock B) Immediately after taking off the order C) As needed, but not more than once per hour D) 1 hour after the last administered dose

c. As needed, but not more than once per hour

The patient shows the necessary defining characteristics, and the nurse diagnoses Decisional Conflict related to unclear personal values and beliefs. Which essential action should the nurse take to help ensure the accuracy of this diagnosis? A) Ask a more experienced nurse to confirm it. B) Have a social worker interview the patient. C) Ask the patient to confirm the diagnosis. D) Read about Decisional Conflict in the NANDA-I handbook.

c. Ask the patient to confirm the diagnosis.

A patient who is diagnosed with shingles is taking topical acyclovir, and the nurse is providing instructions about adverse effects. The nurse will discuss which adverse effects of topical acyclovir therapy? a. Insomnia and nervousness b. Temporary swelling and rash c. Burning when applied d. This medication has no adverse effects.

c. Burning when applied

A patient is infected by invasive aspergillosis, and the medical history reveals that the patient has not been able to tolerate several antifungal drugs. The nurse anticipates an order for which medication to treat this infection? a. Fluconazole (Diflucan) b. Micafungin (Mycamine) c. Caspofungin (Cancidas) d. Nystatin (Mycostatin)

c. Caspofungin (Cancidas)

When reviewing the allergy history of a patient, the nurse notes that the patient is allergic penicillin. Based on this finding, the nurse would question an order for which class of antibiotics? a. Tetracyclines b. Sulfonamides c. Cephalosporins d. Quinolones

c. Cephalosporins

A patient is receiving an infusion of fresh frozen plasma. Based on this order, the nurse interprets that this patient has which condition? a. Hypovolemic shock b. Anemia c. Coagulation disorder d. Previous transfusion reaction

c. Coagulation disorder

A patient is receiving cidofovir (Vistide) as part of treatment for a viral infection, and the nurse is preparing to administer probenecid, which is also ordered. Which is the rationale for administering probenecid along with the cidofovir treatment? a. Probenecid has a synergistic effect when given with cidofovir, thus making the antiviral medication more effective. b. The probenecid also prevents replication of the virus. c. Concurrent drug therapy with probenecid reduces the nephrotoxicity of the cidofovir. d. The probenecid reduces the adverse gastrointestinal effects of the cidofovir.

c. Concurrent drug therapy with probenecid reduces the nephrotoxicity of the cidofovir.

A patient who has type 2 diabetes is scheduled for an oral endoscopy and has been NPO (nothing by mouth) since midnight. What is the best action by the nurse regarding the administration of her oral antidiabetic drugs? a. Administer half the original dose. b. Withhold all medications as ordered. c. Contact the prescriber for further orders. d. Give the medication with a sip of water.

c. Contact the prescriber for further orders.

The medication order reads, "Give ondansetron (Zofran) 4 mg, 30 minutes before beginning chemotherapy to prevent nausea." The nurse notes that the route is missing from the order. What is the nurse's best action? a. Give the medication intravenously because the patient might vomit. b. Give the medication orally because the tablets are available in 4-mg doses. c. Contact the prescriber to clarify the route of the medication ordered. d. Hold the medication until the prescriber returns to make rounds.

c. Contact the prescriber to clarify the route of the medication ordered.

A patient is suffering from tendonitis of the knee. The nurse is reviewing the patient's medication administration record and medication is most appropriate for this type of pain? a. Antidepressant b. Anticonvulsant c. Corticosteroid d. Local anesthesia

c. Corticosteroid

A hospital uses a source-oriented medical record. What is a major disadvantage of this charting system? A) It involves a cooperative effort among various disciplines. B) The system requires diligence in maintaining a current problem list. C) Data may be fragmented and scattered throughout the chart. D) It allows the nurse to provide information in an unorganized manner.

c. Data may be fragmented and scattered throughout the chart

The nurse is monitoring for therapeutic results of antibiotic therapy in a patient with an infection. Which laboratory value would indicate therapeutic effectiveness of this therapy? a. Increased red blood cell count b. Increased hemoglobin level c. Decreased white blood cell count d. Decreased platelet count

c. Decreased white blood cell count

How are critical pathways and standardized nursing care plans similar? A) Specify daily, or even hourly, outcomes and interventions B) Prescribe minimal care needed to meet recommended lengths of stay C) Describe care common to all patients with a certain condition or situation D) Emphasize medical problems and interventions

c. Describe care common to all patients with a certain condition or situation

Which drugs would be affected by the first-pass effect? (Select all that apply.) a. Morphine given by IV push injection b. Sublingual nitroglycerin tablets c. Diphenhydramine (Benadryl) elixir d. Levothyroxine (Synthroid) tablets e. Transdermal nicotine patches f. Esomeprazole (Nexium) capsules g.Penicillin given by IV piggyback infusion

c. Diphenhydramine (Benadryl) elixir d. Levothyroxine (Synthroid) tablets f. Esomeprazole (Nexium) capsules

A patient is to receive acetylcysteine (Mucomyst) as part of the treatment for an acetaminophen (Tylenol) overdose. Which action by the nurse is appropriate when giving this medication? a. Giving the medication undiluted for full effect b. Avoiding the use of a straw when giving this medication c. Disguising the flavor with soda or flavored water d. Preparing to give this medication via a nebulizer

c. Disguising the flavor with soda or flavored water

Which is the reason why nondirective interviewing is a useful technique? A) Allows the nurse to have control of the interview B) Is an efficient way to interview a patient C) Facilitates open communication D) Helps focus patients who are anxious

c. Facilitates open communication

When monitoring a patient's response to oral antidiabetic drugs, the nurse knows that which laboratory result would indicate a therapeutic response? а. Random blood glucose level 180 mg/dL b. Blood glucose level of 50 mg/dL after meals c. Fasting blood glucose level between 92 mg/dL d. Evening blood glucose level below 80 mg/dL

c. Fasting blood glucose level between 92 mg/dL

During an infusion of albumin, the nurse monitors the patient closely for the development of which adverse effect? a. Hypernatremia b. Fluid volume deficit c. Fluid volume overload d. Transfusion reaction

c. Fluid volume overload

A 38-year-old man has come into the urgent care center with severe hip pain after falling from a ladder at work. He says he has taken several pain pills over the past few hours but cannot remember how many he has taken. He hands the nurse an empty bottle of acetaminophen (Tylenol). The nurse is aware that the most serious toxic effect of acute acetaminophen overdose is which condition? a. Tachycardia b. Central nervous system depression c. Hepatic necrosis d. Nephropathy

c. Hepatic necrosis

A sanitation worker has experienced a needle stick by a contaminated needle that was placed in a trash can. The employee health nurse expects that which drug will be used to provide passive immunity to hepatitis B infection? a. Haemophilus influenzae type b (Hib) b. Varicella virus vaccine (Varivax) c. Hepatitis B immunoglobulin (BayHep B) d. Hepatitis B virus vaccine (inactivated) (Recombivax HB)

c. Hepatitis B immunoglobulin (BayHep B)

The nurse checks the patient's laboratory work prior to administering a dose of vancomycin (Vancocin) and finds that the trough vancomycin level is 24 mcg/mL. What will the nurse do next? a. Administer the vancomycin as ordered. b. Hold the drug, and administer 4 hours later. c. Hold the drug, and notify the prescriber. d. Repeat the test to verify results.

c. Hold the drug, and notify the prescriber.

A patient will be discharged with a 1-week supply of an opioid analgesic for pain management after abdominal surgery. The nurse will include which information in the teaching plan? a. How to prevent dehydration due to diarrhea b. The importance of taking the drug only when the pain becomes severe c. How to prevent constipation d. The importance of taking the drug on an empty stomach

c. How to prevent constipation

Which nursing diagnosis is written in the correct format when using the NANDA-I taxonomy? A) Bowel Obstruction related to recent abdominal surgery A.M.B. nausea, vomiting, and abdominal pain B) Inability to Ingest Food related to imbalanced nutrition: less than body requirements A.M.B. inadequate food intake, weight less than 20% under ideal body weight C) Impaired Skin Integrity related to physical immobility A.M.B. skin tear over sacral area D) Caregiver Role Strain related to alienation from family and friends A.M.B. 24-hour care responsibilities

c. Impaired Skin Integrity related to physical immobility A.M.B. skin tear over sacral area

Which nursing diagnosis is written in the correct format? A) Imbalanced Nutrition: Less than Body Requirements related to body weight less than 20% under ideal weight B) Ineffective Airway Clearance related to increased respiratory rate and irregular rhythm C) Impaired Swallowing related to absent gag reflex D) Excess Fluid Volume related to 3 lb weight gain in 24 hours

c. Impaired swallowing related to absent gag reflex

A patient comes to the urgent care clinic because he stepped on a rusty nail. Which type of assessment should the nurse perform when collecting data from this patient? A) Comprehensive B) Ongoing C) Initial focused D) Special needs

c. Initial Focused

Which is an example of an active listening behavior? A) Taking frequent notes B) Asking for more details C) Leaning toward the patient D) Sitting with legs crossed

c. Leaning toward the patient

For which cultural group must the health care provider respect the value placed on preserving harmony with nature and the belief that disease is a result of ill spirits? a. Hispanics b. Asian Americans c. Native Americans d. African Americans

c. Native Americans

The nurse is reviewing principles of immunization. What type of immunization occurs when antibodies pass from mother to infant during breastfeeding or through the placenta during pregnancy? a. Artificial active immunization b. Attenuating immunization c. Natural passive immunization d. Artificial passive immunization

c. Natural passive immunization

A patient is taking the nonsteroidal anti-inflammatory drug indomethacin (Indocin) as treatment for pericarditis. The nurse will teach the patient to watch for which adverse effect? a. Tachycardia b. Nervousness c. Nausea and vomiting d. Dizziness

c. Nausea and vomiting

Which is the most important reason for nurses to be critical thinkers? A) Nurses need to follow policies and procedures. B) Nurses work with other healthcare team members. C) Nurses care for clients who have multiple health problems. D) Nurses have to be flexible and work variable schedules.

c. Nurses care for clients who have multiple health problems

The nurse is assigned to the care of the following patients. In planning nursing care, the nurse knows to use touch cautiously when communicating with which patient? A) Middle-aged woman just diagnosed with terminal lung cancer B) Middle-aged man experiencing the acute phase of myocardial infarction (MI) C) Older adult with a history of dementia admitted for dehydration D) Young adult in the rehabilitative phase after arthroscopic surgery

c. Older adult with a history of dementia admitted for dehydration

A patient who is severely anemic also has acute heart failure with severe edema due to fluid overload. The prescriber wants to raise the patient's hemoglobin and hematocrit levels. The nurse anticipates that the patient will receive which blood product? a. Fresh frozen plasma b. Albumin c. Packed red blood cells (PRBCS) d. Whole blood

c. Packed red blood cells (PRBCS)

A patient with a history of hypertension and rheumatoid arthritis is admitted for surgery for colon cancer. Which integrated plan of care (IPOC) would be most appropriate for the nurse to implement? A) Hypertension B) Rheumatoid arthritis C) Postoperative colon resection D) Follow all three plans

c. Postoperative Colon Resection

A patient with a long-term intravenous catheter is going home. The nurse knows that if he is allergic to seafood, which antiseptic agent is contraindicated? a. Chlorhexidine gluconate (Hibiclens) b. Hydrogen peroxide c. Povidone-iodine (Betadine) d. Isopropyl alcohol

c. Povidone-iodine (Betadine)

The nurse is assessing a patient who has been admitted to the emergency department for a possible opioid overdose. Which assessment finding is characteristic of an opioid drug overdose? a. Dilated pupils b. Restlessness c. Respiration rate of 6 breaths/min d. Heart rate of 55 beats/min

c. Respiration rate of 6 breaths/min

A patient is receiving his third intravenous dose of a penicillin drug. He calls the nurse to report that he is feeling "anxious" and is having trouble breathing. What will the nurse do first? a. Notify the prescriber. b. Take the patient's vital signs. c. Stop the antibiotic infusion. d. Check for allergies.

c. Stop the antibiotic infusion.

A patient has a urinary tract infection. The nurse knows that which class of drugs is especially useful for such infections? a. Macrolides b. Carbapenems c. Sulfonamides d. Tetracyclines

c. Sulfonamides

The nurse is administering one of the lipid formulations of amphotericin B. When giving this drug, which concept is important to remember? a. The lipid formulations may be given in oral form. b. The doses are much lower than the doses of the older drugs. c. The lipid formulations are associated with fewer adverse effects than the older drugs. d. There is no difference in cost between the newer and older forms.

c. The lipid formulations are associated with fewer adverse effects than the older drugs.

The nurse is monitoring a patient who has been on antibiotic therapy for 2 weeks. Today the patient tells the nurse that he has had watery diarrhea since the day before and is having abdominal cramps. His oral temperature is 101° F (38.3° C). Based on these findings, which conclusion will the nurse draw? a. The patient's original infection has not responded to the antibiotic therapy. b. The patient is showing typical adverse effects of antibiotic therapy. c. The patient needs to be tested for Clostridium difficile infection. d. The patient will need to take a different antibiotic.

c. The patient needs to be tested for Clostridium difficile infection.

The nurse is performing an assessment of a newly admitted patient. Which is an example of subjective data? a. Blood pressure 158/96 mm Hg b. Weight 255 pounds c. The patient reports that he uses the herbal product ginkgo. d. The patient's laboratory work includes a complete blood count and urinalysis.

c. The patient reports that he uses the herbal product ginkgo.

When the nurse considers the timing of a drug dose, which factor is appropriate to consider when deciding when to give a drug? a. The patient's ability to swallow b. The patient's height c. The patient's last meal d. The patient's allergies

c. The patient's last meal

The nurse is reviewing pharmacology terms for a group of newly graduated nurses. Which sentence defines a drug's half-life? a. The time it takes for the drug to cause half of its therapeutic response b. The time it takes for one half of the original amount of a drug to reach the target cells c. The time it takes for one half of the original amount of a drug to be removed from the body d. The time it takes for one half of the original amount of a drug to be absorbed into the circulation

c. The time it takes for one half of the original amount of a drug to be removed from the body

Which statement is true for goals/outcomes for collaborative problems? A) They are monitored only by other disciplines. B) They are usually sensitive to nursing interventions. C) They state that a complication will not occur. D) They state only broad performance criteria.

c. They state that a complication will not occur.

When a patient is on aminoglycoside therapy, the nurse will monitor the patient for which indicators of potential toxicity? a. Fever b. White blood cell count of 8000 cells/mm c. Tinnitus and dizziness d. Decreased blood urea nitrogen (BUN) levels

c. Tinnitus and dizziness

Which is an example of an open-ended question? A) Have you had surgery before? B) When was your last menstrual period? C) What happens when you have a headache? D) Do you have a family history of heart disease?

c. What happens when you have a headache?

When teaching about hypoglycemia, the nurse will make sure that the patient is aware of the early signs of hypoglycemia, including: a. hypothermia and seizures. b. nausea and diarrhea. c. confusion and sweating. d. fruity, acetone odor to the breath.

c. confusion and sweating.

Which is an example of practical knowledge? A) The tricuspid valve is between the right atrium and ventricle of the heart. B) The pancreas does not produce enough insulin in type 1 diabetes. C) When assessing the abdomen, you should auscultate before palpating. D) Research shows pain medication given intravenously acts faster than by other routes.

c. when assessing the abdomen, you should auscultate before palpating

During a blood transfusion, the patient begins to have chills and back pain. What is the nurse's priority action? a.)Observe for other symptoms. b.) Slow the infusion rate of the blood. c.) Discontinue the infusion immediately, and notify the prescriber. d.) Tell the patient that these symptoms are a normal reaction to the blood product.

c.) Discontinue the infusion immediately, and notify the prescriber.

When given a scheduled morning medication , the patient states , " I haven't seen that pill before . Are you sure it's correct ? " The nurse checks the medication administration record and verifies that it is listed . Which is the nurse's best response ? a . " It's listed here on the medication sheet , so you should take it . " b . " Go ahead and take it , and then I'll check with your doctor about it . " c. " It wouldn't be listed here if it were not ordered for you ! " d " Let me check on the order first before you take it . "

d " Let me check on the order first before you take it . "

A patient is to receive a penicillin intramuscular ( IM ) injection in the ventrogluteal site . The nurse will use which angle for the needle insertion ? a . 15 degrees b . 45 degrees . c . 60 degrees d . 90 degrees

d . 90 degrees

A patient is receiving eyedrops that contain a beta - blocker medication . The nurse will use what method to reduce systemic effects after administering the eyedrops ? a . Wiping off excess liquid immediately after instilling the drops b . Having the patient close the eye tightly after the drops are instilled c . Having the patient try to keep the eye open for 30 seconds after the drops are instilled d . Applying gentle pressure to the patient's nasolacrimal duct for 30 to 60 seconds after instilling the drops

d . Applying gentle pressure to the patient's nasolacrimal duct for 30 to 60 seconds after instilling the drops

1. Before administering any medication , what is the nurse's priority action regarding patient safety ? a . Verifying orders with another nurse b . Documenting the medications given c . Counting medications in the medication cart drawers d . Checking the patient's identification using two identifiers

d . Checking the patient's identification using two identifiers

When giving medications , the nurse will use Standard Precautions , which include what action ? a . Bending the needle to prevent reuse b . Recapping needles to prevent needle sticks c . Discarding all syringes and needles in the trash can d . Discarding all syringes and needles in a puncture - resistant container

d . Discarding all syringes and needles in a puncture - resistant container

The nurse is about to give a rectal suppository to a patient . Which technique would facilitate the administration and absorption of the rectal suppository ? a . Having the patient lie on his or her right side , unless contraindicated b . Having the patient hold his or her breath during insertion of the medication c . Lubricating the suppository with a small amount of petroleum - based lubricant before insertion d . Encouraging the patient to lie on his or her left side for 15 to 20 minutes after insertion

d . Encouraging the patient to lie on his or her left side for 15 to 20 minutes after insertion

The nurse is giving medications through a percutaneous endoscopic gastrostomy ( PEG ) tube . Which technique is correct ? a . Administering the medications using a 3 - mL medication syringe b . Applying firm pressure on the syringe's piston to infuse the medication c . Flushing the tubing with 30 mL of saline after the medication has been given d . Using the barrel of the syringe , allowing the medication to flow via gravity into the tube

d . Using the barrel of the syringe , allowing the medication to flow via gravity into the tube

The nurse has an order to administer an intramuscular ( IM ) immunization to a 2 - month - old child . Which site is considered the best choice for this injection ? a . Deltoid b . Dorsogluteal c . Ventrogluteal d . Vastus lateralis

d . Vastus lateralis

The nurse is measuring 4 mL of a liquid cough elixir for a child . Which method is most appropriate ? a . Using a teaspoon to measure and administer b . Holding the medication cup at eye level and filling it to the desired level c . Withdrawing the elixir from the container using a syringe without a needle attached d . Withdrawing the elixir from the container using a calibrated oral syringe

d . Withdrawing the elixir from the container using a calibrated oral syringe

A new graduate asks the manager to explain the purpose of full-spectrum nursing. How should the manager respond? A) Provide care in the most cost-effective manner possible. B) Minimize the use of overtime when providing client care. C) Adhere to federal and state regulations when providing care. D) Achieve safe, effective care and promote good patient outcomes.

d) Achieve safe, effective care and promote good patient outcomes.

The nurse is a Christian. She is caring for a Jewish patient who has asked her to offer a prayer at the bedside. The nurse feels comfortable doing so. Which action by the nurse is appropriate? a) Offer a prayer for healing using the nurse's usual words and format. b) Begin the prayer with the "Jehovah's God" as she always does while avoiding the name of Jesus. c) Avoid saying any name for the Supreme Being while praying and quote an Old Testament Bible scripture as the prayer. d) Ask whether the expectation is to pray for the patient or with the patient.

d) Ask whether the expectation is to pray for the patient or with the patient.

At a recent nurse staff meeting, the chief nursing officer (CNO) announced that all nursing staff would work 12-hour shifts on a day-night rotation schedule that would alternate every 6 weeks. This decision was made to solve discord between the nurses who work the different shifts in order for all staff to appreciate the different job responsibilities. What type of leadership is the CNO displaying? a) Management b) Laissez-faire c) Democratic d) Authoritarian

d) Authoritarian

The managers of the physical therapy and nursing departments are having conflict about who is responsible for transporting patients for rehabilitation. All attempts at conflict resolution have failed. What is the next step the managers should take? a) Inform the nurses that they must take the patients to and from therapy. b) Inform the therapists that they must take the patients to and from therapy. c) Ask the hospital administrator to make an unbiased decision. d) Begin informal negotiations between the two departments.

d) Begin informal negotiations between the two departments.

The nurse accidentally provides a patient with 10 mg of warfarin instead of 5 mg as prescribed. Which action should the nurse take? a) No action is necessary because an extra 5 mg of warfarin is not harmful. b) Call the prescriber and ask her to change the order to 10 mg. c) Document on the chart that the drug was given and indicate the drug was given in error. d) Complete an incident report according to the facility's policy.

d) Complete an incident report according to the facility's policy.

The nurse notes that a risk management nurse reviews the medical records of several clients on a care area. For what purpose is this review being performed? A) Calculating the cost of care B) Monitoring health status over time C) Gathering data for clinical research D) Determining whether interventions were appropriate

d) Determining whether interventions were appropriate

Which is the most important reason to develop a definition of nursing? A) Recruit more informed people into the nursing profession B) Evaluate the degree of role satisfaction C) Dispel the stereotypical images of nurses and nursing D) Differentiate nursing activities from those of other health professionals

d) Differentiate nursing activities from those of other health professionals

Which outcome statement contains the best example of performance criteria? A) Turns self in bed frequently while awake B) Understands how to use crutches by day 2 C) States that pain is decreased after being medicated D) Eats 75% of each meal without complaint of nausea

d) Eats 75% of each meal without complaint of nausea

What emotional response is typical during Rando's confrontation phase of the grieving process? a) Anger and bargaining b) Shock with disbelief c) Denial d) Emotional upset

d) Emotional upset

Which nursing intervention should be included in the plan of care for a patient dying of cancer? a) Encourage at least one family member to remain at the bedside at all times. b) Follow up with other healthcare team members during weekly meetings. c) Avoid discussing the dying process with family (to reduce sadness). d) Encourage family members to participate in care of the patient when possible.

d) Encourage family members to participate in care of the patient when possible.

The nurse utilizes the computer to create a care plan for a client. What is an advantage of the nurse using this approach for planning care? A) Limits intuition to reduce errors B) Encourages a linear thinking process C) Requires minimal expertise to plan care D) Ensures a variety of interventions are selected

d) Ensures a variety of interventions are selected

Which action should the nurse take immediately after administering a medication through a nasogastric tube? a) Verify correct nasogastric tube placement in the stomach. b) Auscultate the abdomen for presence of bowel sounds. c) Immediately administer the next prescribed medication. d) Flush the tube with water using a needleless syringe.

d) Flush the tube with water using a needleless syringe

Which action should the nurse take to relax the vastus lateralis muscle before administering an intramuscular injection into the site? a) Apply a warm compress. b) Massage the site in a circular motion. c) Apply a soothing lotion. d) Have the client assume a sitting position.

d) Have the client assume a sitting position.

The nurse manager is completing a SWOT analysis on personal attributes. What should the nurse include when focusing on the W of the analysis? a) Time constraints b) Work experience c) Changes in healthcare d) Inconsistent time management skills

d) Inconsistent time management skills

The nurse plans to attend a seminar to learn how to become a legal nurse consultant because this has been a long-standing interest. What is the best way to describe this nurse's decision? a) Strategy b) Outcome c) Intervention d) Long-term goal

d) Long-term goal

During a health history, a patient whose wife died unexpectedly 6 months ago in a motor vehicle accident admits to drinking at least six whiskeys every night before going to bed. Which type of grief does this best illustrate? a) Delayed b) Chronic c) Disenfranchised d) Masked

d) Masked

During a clinic interview, a client states experiencing dizziness upon standing. Which nursing action is appropriate for the nurse to implement? A) Ask the client when in the day dizziness occurs. B) Help the client to assume a recumbent position. C) Measure both heart rate and blood pressure with the client standing. D) Measure vital signs with the client supine, sitting, and standing.

d) Measure vital signs with the client supine, sitting, and standing.

An older client who became a widower a year ago reports beginning to adjust to life alone. According to John Bowlby, which stage of grief does this comment most likely indicate? a) Shock and numbness b) Yearning and searching c) Disorganization and despair d) Reorganization

d) Reorganization

A newly admitted patient states not being concerned about diagnostic tests but is shaky and tearful and does not maintain eye contact. When conducting a conversation with the client, what should the nurse encourage the client to do? A) Explain the reason for incongruent statements B) Engage in diversional activities to cope with stress C) Express concerns to the primary care provider D) Discuss concerns and fears with the nurse

d) discuss concerns and fears with nurse

The nurse begins discharge planning for an older adult with left-side weakness. Which action is most important in ensuring that the discharge plan is successful? A) Start planning at admission B) Involve the family members C) Involve the multidisciplinary team D) Get patient input when making the plan

d) get patient input when making the plan

Which are the rights of medication administration? a) right patient, right room, right drug, right route, right dose, and right time. b) right drug, right dose, right route, right time, right physician, and right documentation. c) right patient, right drug, right route, right time, right documentation, and right equipment. d) right patient, right drug, right dose, right route, right time, and right documentation.

d) right patient, right drug, right dose, right route, right time, and right documentation.

The nurse is reviewing instructions for vaginal antifungal drugs with a patient. Which statement by the nurse is an appropriate instruction regarding these drugs? a. "The medication can be stopped when your symptoms are relieved." b. "Discontinue this medication if menstruation begins." c. "Daily douching is part of the treatment for vaginal fungal infections." d. "Abstain from sexual intercourse until the treatment has been completed and the infection has resolved."

d. "Abstain from sexual intercourse until the treatment has been completed and the infection has resolved."

The nurse is teaching a patient who is taking colchicine for the treatment of gout. Which instruction will the nurse include during the teaching session? a. Fluids should be restricted while on colchicine therapy. b. Take colchicine with meals. c. The drug will be discontinued when symptoms are reduced. d. Call your doctor if you have increased pain or blood in the urine.

d. "Call your doctor if you have increased pain or blood in the urine."

During drug therapy with a tetracycline antibiotic, a patient complains of some nausea and decreased appetite. Which statement is the nurse's best advice to the patient? a. "Take it with cheese and crackers or yogurt." b. "Take each dose with a glass of milk." c. "Take an antacid with each dose as needed." d. "Drink a full glass of water with each dose."

d. "Drink a full glass of water with each dose."

The nurse is teaching a review class to nurses about diabetes mellitus. Which statement by the nurse is correct? a. "Patients with type 2 diabetes will never need insulin." b. "Oral antidiabetic drugs are safe for use during pregnancy." c. "Pediatric patients cannot take insulin," d. "Insulin therapy is possible during pregnancy if managed carefully."

d. "Insulin therapy is possible during pregnancy if managed carefully."

A patient who has a history of coronary artery disease has been instructed to take one 81-mg aspirin tablet a day. The patient asks about the purpose of this aspirin. Which response by the nurse is correct? a. Aspirin is given reduce anxiety. b. It helps to reduce inflammation. c. Aspirin is given to relieve pain. d. It will help to prevent clot formation.

d. "It will help to prevent clot formation."

A patient is in an urgent care center and is receiving treatment for mild hyponatremia after spending several hours doing gardening work in the heat of the day. The nurse expects that which drug therapy will be used to treat this condition? a.) Oral supplementation of fluids b.) Intravenous bolus of lactated Ringer's solution c.) Normal saline infusion, administered slowly d. )Oral administration of sodium chloride tablets

d. )Oral administration of sodium chloride tablets

The patient is complaining of a headache and asks the nurse which over-the-counter medication form would work the fastest to help reduce the pain. Which medication form will the nurse suggest? a. A capsule b. A tablet c. An enteric-coated tablet d. A powder

d. A powder

The nurse is writing a nursing diagnosis for a plan of care for a patient who has been newly diagnosed with type 2 diabetes. Which statement reflects the correct format for a nursing diagnosis? a. Anxiety b. Anxiety related to new drug therapy c. Anxiety related to anxious feelings about drug therapy, as evidenced by statements such as "I'm upset about having to test my blood sugars." d. Anxiety related to new drug therapy, as evidenced by statements such as "I'm upset about having to test my blood sugars."

d. Anxiety related to new drug therapy, as evidenced by statements such as "I'm upset about having to test my blood sugars."

An older adult patient is diagnosed with type 1 diabetes mellitus (DM). The patient can perform self-care activities but needs help with shopping and meal preparation as well as with blood glucose monitoring and insulin administration. Which type of healthcare facility would be most appropriate for this patient? A) Acute care facility B) Ambulatory care facility C) Extended care facility D) Assisted living facility

d. Assisted Living Facility

The nurse is preparing an infusion of amphotericin B for a patient who has a severe fungal infection. Which intervention is appropriate regarding the potential adverse effects of amphotericin B? a. Discontinuing the infusion immediately if fever, chills, or nausea occur b. Gradually increasing the infusion rate until the expected adverse effects occur с. If fever, chills, or nausea occur during the infusion, administering medications to treat the symptoms d. Before beginning the infusion, administering an antipyretic and an antiemetic drug

d. Before beginning the infusion, administering an antipyretic and an antiemetic drug

A patient is admitted with a fever of 102,8% F (39.3 C), origin unknown. Assessment reveals cloudy, foul-smelling urine that is dark amber in color. Orders have just been written to obtain stat urine and blood cultures and to administer an antibiotic intravenously. The nurse will complete these orders in which sequence? a. Blood culture, antibiotic dose, urine culture b. Urine culture, antibiotic dose, blood culture c. Antibiotic dose, blood and urine cultures d. Blood and urine cultures, antibiotic dose

d. Blood and urine cultures, antibiotic dose

4. A patient who is HIV- positive has been receiving medication therapy that includes zidovudine (Retrovir). However, the prescriber has decided to stop the zidovudine because of its dose- limiting adverse effect. Which of these conditions is the dose-limiting adverse effect of zidovudine therapy? a. Retinitis b. Renal toxicity c. Hepatotoxicity d. Bone marrow suppression

d. Bone marrow suppression

A patient has been diagnosed with carbapenemase-resistant Enterobacteriaceae (CRE). The nurse expects to see orders for which drug? a. Dapsone (Cubicin), a miscellaneous antibiotic b. Ciprofloxacin (Cipro), a quinolone c. Linezolid (Zyvox), an oxazolidinone d. Colistimethate sodium (Coly-Mycin), a polypeptide antibiotic

d. Colistimethate sodium (Coly-Mycin), a polypeptide antibiotic

A patient has used enteric aspirin for several years as treatment for osteoarthritis. However, the symptoms are now worse and she is given a prescription for a nonsteroidal anti-inflammatory drug and misoprostol (Cytotec). The patient asks the nurse, ―Why am I now taking two pills for arthritis?‖ What is the nurse's best response? a. Cytotec will also reduce the symptoms of your arthritis. b. Cytotec helps the action of the NSAID so that it will work better. c. Cytotec reduces the mucous secretions in the stomach, which reduces gastric irritation. d. Cytotec may help to prevent gastric ulcers that may occur in patients taking NSAIDs.

d. Cytotec may help to prevent gastric ulcers that may occur in patients taking NSAIDs.

When making a diagnosis using NANDA-I, which provides support for the diagnostic label chosen by the nurse? A) Etiology B) Related factors C) Diagnostic label D) Defining characteristics

d. Defining characteristics

The nurse is preparing to use an antiseptic. Which statement is correct regarding how antiseptics differ from disinfectants? a. Antiseptics are used to sterilize surgical equipment. b. Disinfectants are used as preoperative skin preparation. c. Antiseptics are used only on living tissue to kill microorganisms. d. Disinfectants are used only on nonliving objects to destroy organisms.

d. Disinfectants are used only on nonliving objects to destroy organisms.

When should the nurse make systematic observations about a patient? A) When the patient has specific complaints B) With the first assessment of the shift C) Each time the nurse gives medications to the patient D) Each time the nurse interacts with the patient

d. Each time the nurse interacts with the patient

The nurse has been monitoring the patient's progress on a new drug regimen since the first dose and documenting the patient's therapeutic response to the medication. Which phase of the nursing process do these actions illustrate? a. Nursing diagnosis b. Planning c. Implementation d. Evaluation

d. Evaluation

The nurse is administering medications to the patient who is in renal failure resulting from end- stage renal disease. The nurse is aware that patients with kidney failure would most likely have problems with which pharmacokinetic phase? a. Absorption b. Distribution c. Metabolism d. Excretion

d. Excretion

Two patients arrive at the clinic; one is a young boy with sickle cell anemia, and another is a 57- year-old woman with early stages of Hodgkin's disease. The nurse notices that both patients need the same vaccine. What vaccine would that be? a. Varicella virus vaccine (Varivax) b. Herpes zoster vaccine (Zostavax) c. Hepatitis B virus vaccine, inactivated (Recombivax HB) d. Haemophilus influenzae type b (Hib) vaccine

d. Haemophilus influenzae type b (Hib) vaccine

What is a benefit of standardized care plans? A) Specify patient outcomes for each day B) Include both medical and nursing orders C) Apply to every patient on a particular unit D) Help ensure that important interventions are not overlooked

d. Help ensure that important interventions are not overlooked

A mother brings her toddler into the emergency department and tells the nurse that she thinks the toddler has eaten an entire bottle of chewable aspirin tablets. The nurse will assess for which most common signs of salicylate intoxication in children? a. Photosensitivity and nervousness b. Tinnitus and hearing loss c. Acute gastrointestinal bleeding d. Hyperventilation and drowsiness

d. Hyperventilation and drowsiness

The nurse is administering a vancomycin (Vancocin) infusion. Which measure is appropriate for the nurse to implement in order to reduce complications that may occur with this drug's administration? a. Monitoring blood pressure for hypertension during the infusion b. Discontinuing the drug immediately if red man syndrome occurs c. Restricting fluids during vancomycin therapy d. Infusing the drug over at least 1 hour

d. Infusing the drug over at least 1 hour

The nurse is reviewing the medication orders for a patient who will be receiving gentamicin therapy. Which other medication or medication class, if ordered, would be a potential interaction concern? a. Calcium channel blockers b. Phenytoin c. Proton pump inhibitors d. Loop diuretics

d. Loop diuretics

An 18-year-old basketball player fell and twisted his ankle during a game. The nurse will expect to administer which type of analgesic? a. Synthetic opioid, such as meperidine (Demerol) b. Opium alkaloid, such as morphine sulfate c. Opioid antagonist, such as naloxone HCL (Narcan) d. Nonopioid analgesic, such as indomethacin (Indocin)

d. Nonopioid analgesic, such as indomethacin (Indocin)

The nurse administering pain medication every 4 hours is an example of which aspect of patient care? A) Assessment data B) Nursing diagnosis C) Patient outcome D) Nursing intervention

d. Nursing intervention

When monitoring the patient receiving an intravenous infusion to reduce blood pressure, the nurse notes that the patient's blood pressure is extremely low, and the patient is lethargic and difficult to awaken. This would be classified as which type of adverse drug reaction? a. Adverse effect b. Allergic reaction c. Idiosyncratic reaction d. Pharmacologic reaction

d. Pharmacologic reaction

An infant has been hospitalized with a severe lung infection caused by the respiratory syncytial virus (RSV) and will be receiving medication via the inhalation route. The nurse expects which drug to be used? a. Acyclovir (Zovirax) b. Ganciclovir (Cytovene) c. Amantadine (Symmetrel) d. Ribavirin (Virazole)

d. Ribavirin (Virazole)

The nurse is teaching patients about self-injection of insulin. Which statement is true regarding injection sites? а. Avoid the abdomen because absorption there is irregular. b. Choose a different site at random for each injection c. Give the injection in the same area each time. d. Rotate sites within the same location for about 1 week before rotating to a new location.

d. Rotate sites within the same location for about 1 week before rotating to a new location.

What is the role of the Joint Commission in regard to patient assessment? The Joint Commission A) States what assessments are collected by individuals with different credentials B) Regulates the time frames for when assessments should be completed C) Identifies how data are to be collected and documented D) Sets standards for what and when to assess the patient

d. Sets standards for what and when to assess the patient

A 79-year-old patient is receiving a quinolone as treatment for a complicated incision infection. The nurse will monitor for which adverse effect that is associated with these drugs? a. Neuralgia b. Double vision c. Hypotension d. Tendonitis and tendon rupture

d. Tendonitis and tendon rupture

When reviewing the mechanism of action of a specific drug, the nurse reads that the drug works by selective enzyme interaction. Which of these processes describes selective enzyme interaction? a. The drug alters cell membrane permeability. b. The drug's effectiveness within the cell walls oř the target tissue is enhanced. c. The drug is attracted to a receptor on the cell wall, preventing an enzyme from binding to that receptor. d. The drug binds to an enzyme molecule and inhibits or enhances the enzyme's action with the normal target cell. phase?

d. The drug binds to an enzyme molecule and inhibits or enhances the enzyme's action with the normal target cell. phase?

When administering a new medication to a patient, the nurse reads that it is highly protein bound. Assuming that the patient's albumin levels are normal, the nurse would expect which result, as compared to a medication that is not highly protein bound? a. Renal excretion will be faster. b. The drug will be metabolized quickly. c. The duration of action of the medication will be shorter. d. The duration of action of the medication will be longer.

d. The duration of action of the medication will be longer.

The nurse is assessing an elderly Hispanic woman who is being treated for hypertension. During the assessment, what is important for the nurse to remember about cultural aspects? a. The patient should be discouraged from using folk remedies and rituals. b. The nurse will expect the patient to value protective bracelets and "root workers" as healers. c. The nurse will remember that the balance among body, mind, and environment is important for this patient's health beliefs. d. The nurse's assessment needs to include gathering information regarding religious practices and beliefs regarding medication, treatment, and healing.

d. The nurse's assessment needs to include gathering information regarding religious practices and beliefs regarding medication, treatment, and healing.

A patient has been selected as a potential recipient of an experimental drug for heart failure. The nurse knows that when informed consent has been obtained, it indicates which of these? a. The patient has been informed of the possible benefits of the new therapy. b. The patient will be informed of the details of the study as the research continues. c. The patient will receive the actual drug during the experiment. d. The patient has had the study's purpose, procedures, and the risks involved explained to him.

d. The patient has had the study's purpose, procedures, and the risks involved explained to him.

The nurse is assigned to a patient who is newly diagnosed with type 1 diabetes mellitus. Which statement best illustrates an outcome criterion for this patient? a. The patient will follow instructions. b. The patient will not experience complications. c. The patient will adhere to the new insulin treatment regimen. d. The patient will demonstrate correct blood glucose testing technique.

d. The patient will demonstrate correct blood glucose testing technique.

When administering drugs, the nurse remembers that the duration of action of a drug is defined as which of these? a. The time it takes for a drug to elicit a therapeutic response b. The amount of time needed to remove a drug from circulation c. The time it takes for a drug to achieve its maximum therapeutic response d. The time period at which a drug's concentration is sufficient to cause a therapeutic response

d. The time period at which a drug's concentration is sufficient to cause a therapeutic response

The nurse is reviewing a list of scheduled drugs and notes that Schedule C-I drugs are not on the list. Which is a characteristic of Schedule C-I drugs? a. No refills are permitted. b. They may be obtained over-the-counter with a signature. c. They are available only by written prescription. d. They are used only with approved protocols.

d. They are used only with approved protocols.

The drug nalbuphine (Nubain) is an agonist-antagonist (partial agonist). The nurse understands that which is a characteristic of partial agonists? a. They have anti-inflammatory effects. b. They are given to reverse the effects of opiates. c. They have a higher potency than agonists. d. They have a lower dependency potential than agonists.

d. They have a lower dependency potential than agonists.

A patient is taking a combination of antiviral drugs as treatment for early stages ofa viral infection. While discussing the drug therapy, the patient asks the nurse if the drugs will kill the virus. When answering, the nurse keeps in mind which fact about antiviral drugs? a. They are given for palliative reasons only. b. They will be effective as long as the patient is not exposed to the virus again. c. They can be given in large enough doses to eradicate the virus without harming the body's healthy cells. d. They may also kill healthy cells while killing viruses.

d. They may also kill healthy cells while killing viruses.

During discharge patient teaching, the nurse reviews prescriptions with a patient. Which statement is correct about refills for an analgesic that is classified as Schedule C-III? a. No prescription refills are permitted. b. Refills are allowed only by written prescription. c. The patient may have no more than five refills in a 6-month period. d. Written prescriptions expire in 12 months.

d. Written prescriptions expire in 12 months.

A patient arrives at the urgent care center complaining of leg pain after a fall when rock climbing. The x-rays show no broken bones, but he has a large bruise on his thigh. The patient says he drives a truck and does not want to take anything strong because he needs to stay awake. Which statement by the nurse is most appropriate? a. It would be best for you not to take anything if you are planning to drive your truck b. We will discuss with your doctor about taking an opioid because that would work best for your pain. c. You can take acetaminophen, also known as Tylenol, for pain, but no more than 1000 mg per day. d. You can take acetaminophen, also known as Tylenol, for pain, but no more than 3000 mg per day.

d. You can take acetaminophen, also known as Tylenol, for pain, but no more than 3000 mg per day.

Why is it important to obtain information about nutritional and herbal supplements as well as about complementary and alternative therapies? A) To determine what type of therapies are acceptable to the client B) To identify whether the client has a nutrition deficiency C) To help you to understand cultural and spiritual beliefs D) To identify potential interaction with prescribed medication and therapies

d. to identify potential interaction with prescribed medication and therapies

The nurse is working with a graduate nurse to prepare an intravenous dose of potassium. Which statement by the graduate nurse reflects a need for further teaching? a.) "We will need to monitor this infusion closely." b.) "The infusion rate should not go over 10 mEq/hr." c.) "The intravenous potassium will be diluted before we give it." d.) "The intravenous potassium dose will be given undiluted."

d.) "The intravenous potassium dose will be given undiluted."

When monitoring a patient for signs of hypokalemia, the nurse looks for what early sign? a.) Seizures b.) Cardiac dysrhythmias c.) Diarrhea d.) Muscle weakness

d.) Muscle weakness

A patient is taking a sulfonylurea medication for new-onset type 2 diabetes mellitus. When reviewing potential adverse effects during patient teaching, the nurse will include information about which of these effects? (Select all that apply.) а. Нурoglycemia b. Nausea c. Diarrhea d. Weight gain e. Peripheral edema

а. Нурoglycemia b. Nausea d. Weight gain


Conjuntos de estudio relacionados

سؤال وجواب في التاريخ الاسلامي

View Set

personal finances module 1 study guide

View Set

404 Chapter Questions 10, 11, and 12

View Set

U.S. History II Final Exam Part II

View Set

Intro to Social Psych - Ch 11: Prosocial Behavior

View Set

6. Táplálkozás és élelmezés egészségtan

View Set

Management of Patients with Chest and Lower Respiratory Tract Disorders

View Set

PN NCLEX 6th Edition-Pharmacology/Oncology

View Set